Jump to content

Wikipedia:Reference desk/Science

From Wikipedia, the free encyclopedia

This is an old revision of this page, as edited by JohnAspinall (talk | contribs) at 04:54, 17 February 2008 (→‎How much elecricity does an solar panal of a sq mts produce.). The present address (URL) is a permanent link to this revision, which may differ significantly from the current revision.

Welcome to the science section
of the Wikipedia reference desk.
Select a section:
  • [[:|{{{1}}}]]
Want a faster answer?

Main page: Help searching Wikipedia

   

How can I get my question answered?

  • Select the section of the desk that best fits the general topic of your question (see the navigation column to the right).
  • Post your question to only one section, providing a short header that gives the topic of your question.
  • Type '~~~~' (that is, four tilde characters) at the end – this signs and dates your contribution so we know who wrote what and when.
  • Don't post personal contact information – it will be removed. Any answers will be provided here.
  • Please be as specific as possible, and include all relevant context – the usefulness of answers may depend on the context.
  • Note:
    • We don't answer (and may remove) questions that require medical diagnosis or legal advice.
    • We don't answer requests for opinions, predictions or debate.
    • We don't do your homework for you, though we'll help you past the stuck point.
    • We don't conduct original research or provide a free source of ideas, but we'll help you find information you need.



How do I answer a question?

Main page: Wikipedia:Reference desk/Guidelines

  • The best answers address the question directly, and back up facts with wikilinks and links to sources. Do not edit others' comments and do not give any medical or legal advice.
See also:


February 11

Melting points of ice in cold

We're experiencing bitterly cold windchills where I live, and it got me thinking: when salt is applied to ice, it removes the ice because it lowers the melting point. So let's say that I have some calcium chloride salt that is effective in temperatures as low as -25°F. Now let's also suppose that the temperature outside is 0°F, but the windchill is -40°F. Will the salt work? --Emery (talk) 01:43, 11 February 2008 (UTC)[reply]

As far as I know, it shouldn't make it stop working. As you can tell from reading the article, wind chill is merely an apparent change in temperature, caused by changing the rate of heat transfer. It does not actually lower the temperature of a substance any more than still air would, it just does it faster (it only lowers your body temperature further than still air because cooling air has to fight with your body's heat-generating metabolism). Someguy1221 (talk) 01:56, 11 February 2008 (UTC)[reply]
He stole my answer! Just kidding. Someguy is absolutely correct. Wisdom89 (talk) 02:02, 11 February 2008 (UTC)[reply]
To say the same thing another way: wind chill does not describe a temperature; it refer to a temperature as an expressive way of describing a rate of heat loss from your body. --Anon, 03:40 UTC, Feb. 11.
"Perceived cold" is a good way of describing it methinks. Wisdom89 (talk) 03:41, 11 February 2008 (UTC)[reply]

Thank you very much, I figured as such but I just wanted to make sure. :) Emery (talk) 23:43, 12 February 2008 (UTC)[reply]

Orbits

Assuming you have a spaceship with an engine that can provide one gravity of continuous thrust, what's the minimum orbital period you can manage for orbiting Earth? --12.169.167.154 (talk) 01:51, 11 February 2008 (UTC)[reply]

Unfortunately, I don't think the question is well-formed. If you launch your spaceship from a point above the Earth's surface (so that the gravitational acceleration is slightly less than g) you will accelerate at an increasing rate, past escape velocity, and break any orbit. As long as you can produce continuous thrust, in fact, you will always be able to break out of any orbit - orbits only require an initial velocity. Anyway, you might be able to find what you're looking for if you note that for stable orbit, Gravitational acceleration must equal centripetal acceleration, or and the period . SamuelRiv (talk) 03:04, 11 February 2008 (UTC)[reply]

The question makes sense if you realize that it's not asking about an orbit in the usual sense of a freefall path, but one where the spacecraft is thrusting continuously downward, but is moving fast enough that it does not collide with the Earth. I have seen the term forced orbit used for this, although it doesn't seem to be known to Wikipedia.

As Samuel noted, in a conventional orbit . Assuming that the orbit is not so high above ground level that diminution of gravity is a significant factor, the left-hand side is simply equal to 1 gee. So if we thrust continuously downward at 1 gee, we double the total acceleration and therefore double the value of . So the speed is greater by a factor of and the period is smaller by the same factor. The answer is just about an hour. For a more accurate number, do the math based on a realistic value of .

--Anonymous, safely at ground level, 03:52 UTC, February 11, 2008.

Ah, I see it now. Anon is correct, so we have or , which will give minimum period for minimum r, so we must orbit right at the surface of the Earth, and we get a period of about a minute (87s), if I did this right. SamuelRiv (talk) 18:22, 13 February 2008 (UTC)[reply]
Well, I said "about an hour" (i.e. 1/sqrt(2) times the minimum period for an ordinary satellite), so one of us isn't "correct". I don't have time right now to look for the error. --Anon, 20:05 UTC, Feb. 13.
Okay, since no one else filled in, I'm coming back to this now. In short, I was right. Dimensional algebra confirms that is impossible: it would have units of s/m, not m/s. Samuel's first equation is correct and equivalent to what I said before. But then the period is given by , which is indeed about an hour.
More precisely, we are dealing with two different acceleration components: one due to the spaceship's power and one due to the Earth's gravity. The first is fixed at 1 gee, while the latter obviously depends on the ship's altitude. Taking the Earth's radius to be 6380 km and the lowest possible orbit to be 160 km above ground, we have r = 6540 km and the Earth's gravity at that height is (6380/6540)² = 0.95167 gee. So the total centripetal acceleration is a = 1.95167 gee = 19.1393 m/s². Replacing with the more general expression and substituting these values, we get T = 3,673 seconds or say 61.2 minutes. Okay?
--Anonymous, 07:45 UTC, February 17, 2008.
Subsidiary question for anyone interested: could one do even better by using a polar orbit with the perigee over one pole and the apogee over the other? Since the Earth's polar radius is shorter, this would allow a perigee less than 6540 km, but what apogee would be required? You may assume that the 160 km minimum height is measured from sea level and applies at all points in the orbit -- but this means that the curve which the orbit must stay above is not exactly elliptical, so things get tricky.
--Anon, 07:55 UTC, Feb. 17.
Another afterthought: the 160 km figure may actually be too high for this specific problem. If the spacecraft can thrust continuously, then it could vector that thrust to an ever so slightly forward angle and and so overcome a small amount of atmospheric drag. Of course at some point you get into serious frictional heating: what you want is an altitude just high enough that that won't happen. I can't find any information on how high that would be. But if, for example, a circular forced orbit is possible at r = 6480 km (100 km above sea level) instead of 6540 km, then by the formula I gave above, the period would be 3,628 seconds or almost a minute less.  ::::--Anon, 08:16 UTC, Feb. 17.

Force+chair

1. I can lift my own weight. (empirically tested at gym with weight machine)

2. I can lift the weight of the folding chair in my room. (again, tested it)

3. I can lift the chair's weight combined with my own (tested at gym again)

4. I can not lift the chair while standing on it.


Why? I understand that if I did manage it, I would be levitating, and I think it has something to do with...by lifting the chair, I'm pushing down ONTO it with an equal amount of pressure from my legs, neutralizing the lift. Is that right?

Thanks76.118.184.59 (talk) 02:23, 11 February 2008 (UTC)[reply]

Yes. Because the amount of force with which you lift the chair is equal to the amount of force with which your feet press against the chair, the net force on the chair and you is zero, and you don't move. Wrad (talk) 02:32, 11 February 2008 (UTC)[reply]
But see cartoon physics. --Milkbreath (talk) 04:11, 11 February 2008 (UTC)[reply]
"Perceived cold" is a good way of describing it methinks. Wisdom89 (talk) 03:41, 11 February 2008 (UTC)[reply]
You meant to put that under #Melting points of ice in cold, right? --Anon, 03:55 UTC, Feb. 11.
heh, yep, slip up - thanks for pointing it out. Wisdom89 (talk) 04:57, 11 February 2008 (UTC)[reply]
You could always use a pulley to reverse the direction of the applied force.--Shantavira|feed me 08:32, 11 February 2008 (UTC)[reply]
Yea, but the pulley needs to be attached to something which is fixed relative to the ground, and that thing is in effect holding up both you and the chair. --Spoon! (talk) 13:15, 11 February 2008 (UTC)[reply]
I have always found such levitation quite easy- while I am dreaming. Edison (talk) 03:23, 12 February 2008 (UTC)[reply]

What is Adjunctive Therapy and how does it work?

What is Adjunctive therapy and how does it work? Also, how does it apply to animals, especially exotics, and how does it effect them? —Preceding unsigned comment added by SBlondie8967 (talkcontribs) 04:49, 11 February 2008 (UTC)[reply]

Adjunctive therapy, according to the National Institute of Health is "Another treatment used together with the primary treatment. Its purpose is to assist the primary treatment." [1] For example: giving glycoprotein IIb/IIIa inhibitors to someone who has had angioplasty. The angioplasty is the primary treatment, but the IIb/IIIa inhibitors help improve the effectiveness of the primary treatment and are thus considered adjunctive therapy [2]. I'm not sure how to respond to your second question about animals, as I imagine that it would be the same or very similar is in humans. (EhJJ)TALK 19:25, 11 February 2008 (UTC)[reply]

Can you think of any major new organs or adaptive behaviours which evolution might throw up in the next few million years?

Some researchers believe that the Cambrian explosion may have been due to the sudden development of vision. Organs for hearing similarly affected life on earth in profound ways. There are now countless methods whereby creatures walk, fly, mate, live and so on. But are there any other possible major developments that we could second-guess and say: “Here is another organ or sense or way of getting around or maintaining an existence that evolution has not yet found”? Can you think of anything, or do you think that basically nature has no more tricks up her sleeve. Myles325a (talk) 05:56, 11 February 2008 (UTC)[reply]

I doubt there will be anything that will create a new Cambrian explosion, but the ability to detect harmful radiation might be useful after humans have irradiated the planet. — Ƶ§œš¹ [aɪm ˈfɻɛ̃ⁿdˡi] 06:54, 11 February 2008 (UTC)[reply]
Evolution will continue so long as life exists on this planet. Nature is certainly not done. In my opinion it's really not feasible to infer what other adaptations, phenotypic changes, or speciation events might occur in the next million years. Humans most likely will not survive that long. However, if forced to make some kind of a projection, human beings will not be around to observe any major changes. Allelic frequency changes will continue to occur, and new species of plants will continue to emerge, but other than that, I doubt we'd see much. Wisdom89 (T / C) 07:52, 11 February 2008 (UTC)[reply]
For either genetic drift or natural selection to result in anything you would really consider "evolution," there would probably need to be some manner of reproductive isolation to arise (stir and let cool for 3000 generations). I don't see this as likely in a stable world. Someguy1221 (talk) 08:03, 11 February 2008 (UTC)[reply]
An organ capable of breaking down and expelling radioactive material/heavy metals from the body would be a useful adaptation - as would the ability to emit EMP (to defend oneself against those pesky wild, predatory nanobots). --Kurt Shaped Box (talk) 08:55, 11 February 2008 (UTC)[reply]
Who is to say that the nanobots won't evolve resistance to EMP? Advanced nanobots can evolve in just one generation, but nature takes a much longer time. Let's hope we don't invent anything like that. 206.252.74.48 (talk) 14:37, 11 February 2008 (UTC)[reply]
Is this a reference to Star Trek the Next Generation by any chance? Wisdom89 (T / C) 20:48, 11 February 2008 (UTC)[reply]
I don't watch Star Trek, so no. Nearly every scientific thing ever discovered seems to be featured on that show, so it's hardly a coincidence. 206.252.74.48 (talk) 16:59, 12 February 2008 (UTC)[reply]

WHEELS!!

Eventually humanity will adapt to pavement, as rollerbladers have done... Wheels FTW! —Preceding unsigned comment added by 79.122.115.170 (talk) 08:40, 11 February 2008 (UTC)[reply]

I saw the future on TV once. It was land squid. --Kurt Shaped Box (talk) 08:48, 11 February 2008 (UTC)[reply]

Kurt Vonnegut's Galápagos posits that the best adaptation for everyone would be for the human brain size to shrink back down to a less troublesome size. --Sean 14:37, 11 February 2008 (UTC)[reply]

But there is the problem of biological wheels being nigh impossible. But there is a clever work-around in the book The Amber Spyglass. I think humans are more likely to develop cup-holders and storage pouches for snacks. We may also eventually secrete petroleum, which will solve all our problems, right? 206.252.74.48 (talk) 16:37, 11 February 2008 (UTC)[reply]

Biological wheels are nowhere near impossible- they're quite common, albeit on a small scale. See Flagellum. Friday (talk) 16:52, 11 February 2008 (UTC)[reply]

Anyway, it's not the wheels that are the hard part, but the axles! :) --Sean 18:27, 11 February 2008 (UTC)[reply]
Not really, see the structure of ATP synthase. It has a protein axle (gamma subunit) that connects the two wheels (F0 rotary motor and the F1 catalytic core) One difference is that the wheels are stationary and the axle spins but the general idea is there from a structural perspective. David D. (Talk) 22:23, 11 February 2008 (UTC)[reply]

AIDS

Are we any closer to a cure for AIDS than we were 25 years ago? —Preceding unsigned comment added by 195.188.208.251 (talk) 10:29, 11 February 2008 (UTC)[reply]

That's a very possible maybe. There have been some promising studies, although no absolute proof that an HIV cure or vaccine has been developed yet. HIV vaccine describes some of the research in this area, though I'll admit it's not the greatest article. There is also this research program that has (apparently) successfully cured HIV infected mice during the ~40 day duration of the animal testing. Interestingly, that press release completely leaves out about two thirds of what that treatment entails, although those two thirds may have been thought up only after that very old press release was made. There have also been treatments tested in humans that successfully lowered the viral load of infected individuals, though the true impact of such treatments is yet to be seen (I also don't have a link for the treatment I'm thinking of, offhand). Someguy1221 (talk) 11:03, 11 February 2008 (UTC)[reply]
I saw some information about this at this link --Dweller (talk) 13:35, 11 February 2008 (UTC)[reply]

Military Tattoos

I don't know if this is a big public misconseption but I think it is a common thing for military units to all get tattoos. And, at least here in Aus, you cannot donate blood if you have tattoos. And I would suppose that militaries around the world would use blood transfusions very regulary.

So...can people in the military donate blood? I have seen this happen in movies and such (Pearl Harbour and Band of Brothers comes to mind) but do they (now) enforce the same rule of no donating if you have tattoos? If not why not? (I can't think that where they get their tattoos from is any more sanitary than the general populution). If they do not allow the donating where do they get the blood from?

Shniken1 (talk) 12:44, 11 February 2008 (UTC)[reply]

Why wouldn't they let you donate blood if you have tattoos? I have never heard of any such rule in the U.S., so I think this is unique to where you are from. 206.252.74.48 (talk) 14:33, 11 February 2008 (UTC)[reply]
The UK National Blood service also has restrictions based on tattoos and piercings [3]. It's possible (probable, in my opinion) that the military have their own rules regarding who can donate. Travel to malarial areas would also be an issue affecting the military. 81.174.226.229 (talk) 14:44, 11 February 2008 (UTC)[reply]
From the American Red Cross ([4]) guidelines for deferral of donations:
Wait 12 months after a tattoo if the tattoo was applied in a state that does not regulate tattoo facilities. This requirement is related to concerns about hepatitis....
Acceptable if the tattoo was applied by a state-regulated entity using sterile needles and ink that is not reused. Only a few states currently regulate tattoo facilities, so most donors with tattoos must wait 12 months after tattoo application before donating blood. You should discuss your particular situation with the health historian at the time of donation.
I have seen other jurisdictions with a similar deferral period (12 months) following a tattoo or piercing. As noted, the chief concern is about diseases – particularly strains of hepatitis – that may be transmitted via improperly-cleaned piercing or tattooing equipment.
In wartime, and while experiencing severe blood shortages, it is conceivable that blood-collecting agencies might relax some of their deferral criteria. I cannot comment on how or when this may have occurred in the past. TenOfAllTrades(talk) 14:57, 11 February 2008 (UTC)[reply]

RD stress

(See my associated post at the talk page) How often do periods of RD angst occur and how long do they tend to last? I'm depending on you bespectacled intelligent folk to derive some cast-iron objective criteria for determining when these periods are, and then generating some lovely charts and graphs to map it. To help you, I'd suggest that posts to the talk page proliferate at these periods. I'm looking forward to some very informative and comforting responses. If you do really well, you might even prompt me to reestablish the neglected thread of the week award. --Dweller (talk) 13:05, 11 February 2008 (UTC)[reply]

You'd better re-establish that reward. In a bizarre coincidence I just discovered it today and was wondering if it will ever come back. 206.252.74.48 (talk) 16:23, 11 February 2008 (UTC)[reply]

One thing I agree on.. looking at the dreaded 'discussion' page is a sure sign that things are getting bad or going to get bad soon. The thing has 'bad vibes'>87.102.79.203 (talk) 19:15, 11 February 2008 (UTC)[reply]

It seems to me that reference-desk stress is something that resides in the individual, not the desk itself. My personal motto for years (after a brush with severe depression caused by stress at work) is "avoid all stress". Other than a brief fling with a medical advice question, I have simply ignored those questions that look like they are going to cause strife. After all, if we all ignored such questions (whether innocent or trollish), stress for all would be avoided. Another general rule is never try to answer a question that you really don't know much about, especially if you know some other reference-desk regular knows more that you do. I'm certainly not advocating a humorless reference desk, but after reading the desk for a while, it is perfectly obvious what kind of humor is acceptable and what is not. Indeed I love the clever puns and reductio-ad-absurdum arguments that some of you are so good at. Also, Dweller, I've missed you--I really enjoyed your thread of the week picks!--Eriastrum (talk) 18:55, 13 February 2008 (UTC)[reply]
I second just about everything Eriastrum wrote, especially the last sentence. Having Dweller dangle the award over our heads makes it tempting indeed to crawl through wikidashboards and stress analysis, but I'm afraid that re-reading the screaming contests in the archives might cause me too much stress for my own good. :-) Have mercy, Dweller, and give us our prized prize back. ---Sluzzelin talk 13:19, 14 February 2008 (UTC)[reply]

Looking for northern lights

I have vacation time at the end of August. I decided that I'd like to use it to go see the northern lights. Any suggestions on where to go? I'd want to go to a rather metropolitan area. Tromsø appears to work - but I don't know if it is far enough north. -- kainaw 13:32, 11 February 2008 (UTC)[reply]

Have you been keeping an eye on http://www.spaceweather.com/ ? There's a wealth of information there, and the site will send you an email (for free) or a phone call (for cash) every time something interesting is happening in the sky. TenOfAllTrades(talk) 15:05, 11 February 2008 (UTC)[reply]
One thing to keep in mind is that it will be summer up there, and thus the sun will be shining for most of the night, effectively making the northern lights impossible to see unless you stay up very late. Being in a city won't help much either, as city lights caan block all but the most startling northern lights. If you're lucky and you do your homework, though, you might be able to see them. Wrad (talk) 15:15, 11 February 2008 (UTC)[reply]
Tromsø in August is absolutely beautiful, but it's not the place (or time) to go to for watching polar lights - at that latitude, the sun sets for an hour or two in August, but you don't get to see a dark night sky - it's more like a prolonged dusk immediately followed by the dawn. If you can brave the weather, visit Tromsø in December or January for spectacular polar light displays. In August, you probably have better chances at Patagonia (although I don't know if that is far enough to the south) -- Ferkelparade π 15:30, 11 February 2008 (UTC)[reply]
Basically, seeing the northern lights is all about probabilities, and by going in the summer and staying in a city, you are drastically decreasing those probabilities, approaching impossibility. Wrad (talk) 15:35, 11 February 2008 (UTC)[reply]
Thanks. That all makes a lot of sense. I have seen the lights in Norway before (Bardufuss is the phonetic spelling of whatever city I was in - I don't know the Norwegian name). That was in February and I was on top of a mountain above the snowline. I guess my best bet is to try to head south instead of north. I've noticed on astronomy sites that you can see the band of the Milky Way in August if you head south as well. Now, it is just a matter of deciding where to head to. -- kainaw 16:06, 11 February 2008 (UTC)[reply]

Two points. First, in terms of being where the aurora is, it's not your latitude that matters, but your position in relation to the magnetic pole. Therefore the aurora borealis can be seen farther south in Canada than in Norway. This particularly matters in summer since the lower latitude in Canada means you get more darkness at night. I haven't looked into the question of where in Canada is best to go, and for the southern hemisphere I have no idea. I also note that if you get too close to the magnetic pole, the aurora dimishes again (hence the term "auroral oval"); but this isn't likely to be a problem in practicce.

Second, we are currently nearer the bottom of the solar cycle (2006) than the top (about 2012). There should be much better auroras available if you wait a few years.

--Anonymous, 22:22:22 UTC, February 11, 2008.

Should I be the smart-ass who points out that you can't see the Northern Lights from the Southern hemisphere? No, I shouldn't, carry on. Mad031683 (talk) 16:56, 12 February 2008 (UTC)[reply]
Hi. By the way, during occasional intense solar storms, the northern lights can reach pretty far south. One time, a few years ago, I believe it reached down to southern Ontario (but it was cloudy. By the way, if you're lucky and the sky is clear and you're in the place you were mentioning at the time, on August 1, there is a 75% solar eclipse at that location. Hope this helps. Thanks. ~AH1(TCU) 22:47, 12 February 2008 (UTC)[reply]

re:thermoacoustic refrigerator

as mentioned earlier im working on thermoacoustic refrigerator for demonstration.in my final experiment my results were not that good as mentioned in the article.rather i m unable to interpret the results please help:

1. room temperature:25 cel {these are temperatures of cold end of stack) 2.temperature inside the resonator:27 cel 3.temperature reduced by 1 cel in 1 min at 385Hz freq and length of wave was 92: 26 cel 4.temperature reduced by 1 cel in 1 min at 385 Hz freq and length of wave was 92:25.5 cel 5.further temperature became constant at 25.5 cel for next 10 mins.

is the temperature reduction due to the stack.or it is due to a push of external air(at 25 cel ) into the resonator. ive used the function generator: http://www.edn.com/contents/images/di2409setup.exe

kindly help, regards sam Reveal.mystery (talk) —Preceding comment was added at 14:03, 11 February 2008 (UTC)[reply]

Vacuum in space

Who figured-out/proved that space is mostly empty between physical objects? In particular, that "the heavens" are not full of air like our own atmosphere? Note that I am specifically *not* talking about luminiferous aether or any such medium for EM radiation; just air. Thanks. --Sean 14:29, 11 February 2008 (UTC)[reply]

I don't know that this discovery can be attributed to a single person. In the early 17th century Francis Godwin wrote The Man in the Moone in which his hero travels to the Moon in a chariot towed by trained geese - so Godwin clearly believed that interplanetary space was filled with air [5]. By the late 17th century Robert Boyle was experimenting with the properties of a man-made vacumm and conjecturing that the interstellar medium could be a similar vacuum. I imagine that an evolving understanding of the implications of Newton's law of universal gravitation and Newton's laws of motion in the late 17th/early 18th centuries convinced scientists that interplanetary space must be mostly empty, otherwise the planets would quickly lose energy and spiral in towards the Sun. I don't know whether Newton himself ever explicitly pointed this out. Gandalf61 (talk) 16:31, 11 February 2008 (UTC)[reply]
Before that, Aristotle (and those who followed him) would not have countenanced the idea that the heavens might be full of air. For him, the cœlestial realm was utterly and profoundly different from the terrestrial/sublunary, cœlestial objects being made of æther. Algebraist 17:09, 11 February 2008 (UTC)[reply]
Bœnus pœints for þe use of archæic ligature! SamuelRiv (talk) 21:32, 11 February 2008 (UTC)[reply]
Um, doesn't that change the pronounciations completely? Such as it being pronounced "bee-nus peeints"? —Preceding unsigned comment added by 206.252.74.48 (talk) 21:56, 11 February 2008 (UTC)[reply]
Depends what century you're in. Originally, œ had the value [oi̯], but this had shifted to [e] by the middle ages. But what do I do with these bonus points? Is there a prize of some sort? Algebraist 23:42, 11 February 2008 (UTC)[reply]
The discovery of the barometric formula was a clue as well. Icek (talk) 15:47, 12 February 2008 (UTC)[reply]

plastic beads

How are solid plastic beads made? —Preceding unsigned comment added by 71.100.0.31 (talk) 15:36, 11 February 2008 (UTC)[reply]

This may be of use (http://www.wisegeek.com/what-is-plastic-extrusion.htm) perhaps? ny156uk (talk) 19:04, 11 February 2008 (UTC)[reply]

The material is probably sprayed into a cooling medium. The size of the droplets controls the size of the produced beads.
Atlant (talk) 20:14, 11 February 2008 (UTC)[reply]
But how do they get the eyes for the thread? David D. (Talk) 22:29, 11 February 2008 (UTC)[reply]
Well, I agree that any that are basically prismatic are probably extruded. Pierced spheres, I don't know about; I could imagine compression molding followed by tumbling for flash removal.
Atlant (talk) 23:36, 11 February 2008 (UTC)[reply]

Solid beads are different from pierced beads. pierced beads are extruded pipe or tubing that is cut and then heated until the edges melt and disappear. The spray thing with molten plastic would be like how lead shot is made by pouring liquid lead into a container that has holes at the top of a tower and as the droplets fall they turn into spheres and solidify before hitting the ground. Maybe plastic is done the same way. 71.100.0.31 (talk) 01:27, 12 February 2008 (UTC)[reply]

Big-Bonedness

Is there such as condition as being 'big-boned'? --Ouzo (talk) 15:57, 11 February 2008 (UTC)[reply]

Yes, ICD9 278 covers the many variations of obesity. If you are referring to conditions in which the bones themselves are deformed and larger than normal, see the ICD9 710 to 739 range. -- kainaw 16:11, 11 February 2008 (UTC)[reply]
Thanks for the link. ICD-731 listed as Osteitis deformans describes what I'm looking for (articled as Paget's disease of bone). --Ouzo (talk) 20:46, 11 February 2008 (UTC)[reply]
Just out of curiosity, in what way does Paget disease of bone meet your criteria of big-bonedness? I would've thought that something like acromegaly would have been more up your alley. --David Iberri (talk) 03:45, 13 February 2008 (UTC)[reply]

Chemistry helper software

Hey everybody. I'm looking for some kind of (simple!) chemistry software, preferably for Mac OS X or Linux. I'm not quite sure what functionality I need, except I need it to help me draw Lewis structure diagrams. By 'help' I don't mean that it should do any kind of actual work (i.e. I want to place everything myself to learn) but I'm tired of using drawing programs! I found a huge list (and another similar one) of chemistry software for OS X, but I want something pretty small and simple (and free) since I'm just starting out. Also, I figured to ask here rather than at the computing reference desk, since the science geeks are over here! Thanks in advance. -- Aeluwas (talk) 16:23, 11 February 2008 (UTC)[reply]

Category:Chemistry software has a list of ones that have Wikipedia entries. Molecule editor has external-links to the homepage of many standard ones (including ones with no WP page). BKchem, ISIS/Draw (Windows-only), and XDrawChem are all free and make it easy to do nice organic structures and save or export them in standard graphics formats, without a steep learning curve. DMacks (talk) 19:10, 11 February 2008 (UTC)[reply]
I remember using 'chemdraw' on a 'very old' (68000 processor mac) - no idea what it is like now but at the time it was very simple to use.87.102.79.203 (talk) 19:09, 11 February 2008 (UTC) My god that's expensive for what you get - maybe 'xdrawchem' is the free alternative. Good luck. The price of 'chemdraw' is simply outrageous.87.102.79.203 (talk) 19:13, 11 February 2008 (UTC)[reply]
There's a very deep academic discount, but if something that's free suffices, that's even cheaper still:) DMacks (talk) 19:20, 11 February 2008 (UTC)[reply]
Hmm, thanks guys. I've looked through a couple of these, but haven't found anything that does what I need (i.e. drawing simple lewis structures). I'm sure they'll come in handy later, though. I'd like something to help me create pictures like these (the ones on the left). -- Aeluwas (talk) 12:42, 12 February 2008 (UTC)[reply]
Oh sorry, it should be trivial to take a program that does line structures and make it do lewis structures. Maybe you could e-mail some of the free software people and ask if their software can do lewis structures as well. I found http://www.acdlabs.com/educators/cincinnati.html quote:

ACD/ChemSketch is ACD/Labs' chemical structure drawing and visualization application for the Microsoft Windows (95, 98, NT, & 2000) operating system. The software enables you to draw virtually any chemical structure within the scope of your imagination and provides the ability to add chemical structures and drawn pictures, such as lab equipment, Lewis structures, bond ....

it does a lot more as well - here's the main page http://www.acdlabs.com/download/ Note I've never used this program myself.(looks good though) 87.102.81.140 (talk) 13:26, 12 February 2008 (UTC)[reply]
Seems to be what I'm looking for, thanks! Way more advanced than I need at the moment, but I suppose the more advanced functions will come in handy soon enough as well. :) -- Aeluwas (talk) 16:31, 12 February 2008 (UTC)[reply]

I randomly found this article. It claims that these receptors "sense" oxygen levels (actually it isn't clear from the article how). But I found no mention of them in the Internal senses section of the Sense article. Is that an oversight or is the J receptors article misleading? Rmhermen (talk) 16:56, 11 February 2008 (UTC)[reply]

There are literally thousands of chemical receptors like that. Calling them all "internal senses" would be unwieldy and pointless. It is unclear whether the concept of "internal senses" itself is meaningful. --169.230.94.28 (talk) 22:13, 11 February 2008 (UTC)[reply]
I can't recall from memory regarding their exact nature, but the stub seems to describe a mechanoreceptor response, not chemical. I could be wrong about that, but this is what I understand from the wording in the article. Wisdom89 (T / C) 22:18, 11 February 2008 (UTC)[reply]

Using the Hall Effect to Find the Speed of a Flowing Ionic Liquid

Hi, can somebody please give me an indication of how to go about measuring the speed of an ionic fluid using the Hall Effect. Is there any way of doing this theoretically i.e. without simply calibrating the meter by measuring the Hall voltage of a certain liquid at various speeds and plotting a graph?

90.241.222.228 (talk) 17:05, 11 February 2008 (UTC)Shauno[reply]

You could just treat the fluid in the same way as an electric current (knowing the charge carriers) as long as you know the magnetic field strength..
There are numerous problems with this - including ion-pairing (reduces effective charge amount) , shielding by the fluid of the magnetic field (reducing effective field strength) and maybe electrode polarisation as well as the physically resistive effect of the charge polarisation..
So simply - you'd need to calibrate.
You could try to build up a model using all these factors (if relevent) - and use that.. (Beyond me though)87.102.79.203 (talk) 19:05, 11 February 2008 (UTC)[reply]
  • The Hall Effect is when charged particles (in your case, the ions of your ionic fluid) move perpendicular to a magnetic field, they experience a force in the third direction (i.e. perpendicular to both the fluid velocity and the magnetic field). Now if you allow the ions to collect at the edges of the fluid channel, eventually an electric field will build up that will oppose the motion of more ions in that direction. A simple theoretical approach would be to use the equilibrium: force from electric field balances force from Hall effect to find the linear coefficient relating velocity and voltage. JohnAspinall (talk) 19:07, 11 February 2008 (UTC)[reply]

Bouncing Squash Balls

If i knew the volume of a squash ball and the temperature, could i work out how high it would bounce from a drop, using gas laws and kinetic energy equations? —Preceding unsigned comment added by 81.151.74.117 (talk) 19:53, 11 February 2008 (UTC)[reply]

I doubt it, because most of the energy loss is in the rubber of the ball, and you haven't claimed any knowledge about that factor.
Atlant (talk) 20:13, 11 February 2008 (UTC)[reply]
There's an alternative method - ignore the rubber and assume energy loss comes from loss of heat from the gas inside. (this is probably simpler than working out how the rubber deforms and loses energy)
maybe - ignoring the effect of the skin - you could assume that the bit in contact is squashed flat.. - squashing it would cause an increase in pressure and hence an upward force on the squashed bit. Without loss of energy it would bounce just as high.
To take account of the loss of energy you could say that the gas is being heated as it is squashed. If this increase in temperature causes loss of heat to the surrounding then the ball will not rebound as much - however since the heating is related to the rate of change of volume and the pressure you'd certainly end up with a differential equation.

These are the factors..

ʃpressuret x volumet dt = energy supplied
Rate of energy loss = k x (Tball-Texternal)
Volume of ball =4/3 pi r3 (1+cos at) .. a is the angle of the flattened bit (think this is right see Spherical cap for correct equation also try http://mathworld.wolfram.com/SphericalCap.html (eqaution 9) note the alpha is 90-a the angle I used below . sorry in a rush)
r(1-cos a) = distance of impact (you might need/want to reverse this to get a)
pi x (r sin at)2 = At area of sqaushed bit
At x Pt - Pexternal atmosphere= Ft force upwards - note that here I've assumed that the original internal pressure was the same as the atmospheric pressure eg 1atm since I'm ignoring the skin
Ft-mg = overall force upwards (m is ball mass)
Pressure x Volume =2/3 Energy of gas

etc..

There's a bit more work to do here - including the above equations as well as newtons laws of motion I think there is all you need to at least construct a differential equation..(If you don't like differential equations you could use a computer to model this bouncing behavior in small steps of time.)
If you want/need more help please ask.87.102.79.203 (talk) 20:48, 11 February 2008 (UTC)[reply]
If you get that far and get stuck the maths reference desk might be your next step. Good luck87.102.79.203 (talk) 20:28, 11 February 2008 (UTC)[reply]
(( What follows is irrelevent to the original question but is added anyway 'just in case' .... Alternatively you could ignore any energy losses from the gas and calculated the energy required to flatten a rubber skin - here you might need to decide if there is a gas is in the ball or if the springyness comes entirely from the rubber itself - you could assume that any energy required to bend/compress the rubber is entirely lost - ignoring the bending force the energy is simply the change of length of a part of the skin times the distance it is moved. You might want to assume that the skin is flattenend vertically and does not move horizontally since that simplifies the eqautions..
In practice solid rubber balls bounce because of the springyness of the rubber, but hollow rubber balls bounce because of the springyness of the gas..
As a final option (and easiest) option you could just say that a certain percentage of the kinetic energy is lost on contact - but that wasn't your question.87.102.79.203 (talk) 20:44, 11 February 2008 (UTC) ))[reply]
We can do this far more simply. . Note that temperature is constant assuming perfect conduction into the outside reservoir (the outside air), as is the number of moles of gas. So , and note that the energy in an ideal gas deformation is exactly . So we now know what kind of change in volume a given amount of energy will induce. Now we have to fudge things a little to make this work, and I don't think this is valid, but with what you are given, it's the only simple approximation I can think of. Let's now assume that the transformation when the ball hits the ground is isobaric - that is, of constant pressure. Then there must be some temperature change such that , which will result in a heat loss of . Note there is a compression and then re-expansion of the ball, so we must double our heat loss value to find the total heat lost in the bounce. Find the kinetic energy from , and you're home. I'm intrigued enough to try plugging in numbers on my own and seeing if this fudge works, so I'll report the results in a later post. SamuelRiv (talk) 21:28, 11 February 2008 (UTC)[reply]
The final equation I get is . Plugging in the relevant numbers gets me a higher height than what I started with, suggesting this equation is wrong. I think my fudge was too fudgey. Back to the drawing board! SamuelRiv (talk) 22:41, 11 February 2008 (UTC)[reply]

How dangerous are Moka express machines...

...and other similar cookware that work with pressurized boiling water?Mr.K. (talk) 20:18, 11 February 2008 (UTC)[reply]

Moka Express and common pressure cookers have relief valves to prevent dangerous build-up of pressure. The temperatures are only a few dozen degrees above normal boiling point of water, so that on its own isn't a substantially worse scalding hazard than a normal tea-pot. I'd be most concerned about accidentally opening the vessel when it's pressurized...water would flash-boil coffee-grounds go flying, etc. DMacks (talk) 20:50, 11 February 2008 (UTC)[reply]
I think that would be difficult to do "accidentally". The pressure will push against the threads, so it should take an awful lot of torque to overcome the friction and unscrew the bottom from the top -- plus the bottom will be very hot, so it's not like you can grab it with your bare hand. Not sure what would happen if you didn't really get them threaded together right in the first place, but my guess is that the water would just bubble out from the joint and no high pressure would ever build up.
It has occurred to me to wonder what happens if the relief valve gets clogged with lime or something. Has anyone ever heard of an actual dangerous explosion from a moka? --Trovatore (talk) 21:01, 11 February 2008 (UTC)[reply]
I can't say for certain about Mokas, but the safety valve on my pressure cooker doesn't come into contact with the cooking liquid and isn't even used for steam venting except in failure, so it's not like to get scaled up. AlmostReadytoFly (talk) 09:25, 12 February 2008 (UTC)[reply]
I don't know about modern pressure cookers, but a traditional pressure cooker has two separate valves: the normal weighted pin valve that keeps the pressure at the nominal "correct" pressure by venting steam , and a safety relief valve. In normal operation, the regulator valve vents about the same amount of steam as a boiling teapot. If that valve ever gets clogged, the pressure will build until the safety valve blows out leaving a half-inch diameter hole in the top: This is a dangerous mess and should never happen. If it does, most of the contents of the pot are expelled violently all at once: you usually end up needing to clean food off of the ceiling of the kitchen, and a cook can be seriously scalded if in the wrong place at the wrong time. This is however safer than what would happen if the safety valve did not operate: the cooker could explode and send heavy pieces of metal (such as the lid) flying. In 1955, my mother was pressure-cooking a whole chicken when the safety valve let loose. She vividly recalls that nearly the entire chicken, bones and all, were pureed through the valve and spewed onto the ceiling. -Arch dude (talk) 21:42, 12 February 2008 (UTC)[reply]

Does reading ruin your eyes?

Does reading books, ruin a persons' eyes? If so can vision be preserved by not reading? How about reading on the computer?

Which is worse for vision, reading on the computer or books? --Frsoroad (talk) 23:05, 11 February 2008 (UTC)[reply]

Straining in the dark by reading books can be damaging, yes. IMHO, working late at night in front of the computer is more deleterious than reading books in normal warm light. Seriously, using wikipedia in the dark is like staring at white light. Wisdom89 (T / C) 23:10, 11 February 2008 (UTC)[reply]
A recent article in the British Medical Journal said that idea that "Reading in dim light ruins your eyesight" is a myth. [6][7] Although low light levels can lead to temporary eye strain, there is no evidence that inadequate light while reading will result in permanent vision damage, or so says the article. -- 128.104.112.12 (talk) 23:34, 11 February 2008 (UTC)[reply]
Several studies have found myopia to be rare among illiterate cultures, and to increase dramatically in frequency in the first generation of children in those cultures to receive schooling. (See for instance Figure 1 of [8], originally from Morgan RW & Munro M (1973): Refractive problems in northern natives. Can J Ophthalmol 8: 226–228). These observations are widely accepted as evidence that spending a large fraction of ones time during childhood focusing at close distance -- e.g., reading -- strongly increases the likelihood of developing myopia. This conclusion is supported by experiments that have shown that chickens consistently develop severe myopia if they are raised in an environment without anything distant to focus on, or given eyeglasses for the same effect. Chicks, however, apparently unlike many people, only need a brief daily exposure to distant scenes to fully prevent this development of myopia. --mglg(talk) 01:17, 12 February 2008 (UTC)[reply]
I always thought that my chicken tasted rather nearsighted. — Ƶ§œš¹ [aɪm ˈfɻɛ̃ⁿdˡi] 02:10, 12 February 2008 (UTC)[reply]
Your argument makes perfect sense, mglg. I always did read much more than my peers back in grade school, but even though my lenses are literally an inch thick, I think the rewards outweigh the disadvantages. 206.252.74.48 (talk) 13:36, 12 February 2008 (UTC)[reply]
Absolutely. Just look up from the book now and then to gaze dreamily into the distance... --mglg(talk) 21:26, 12 February 2008 (UTC)[reply]
A lot of straining of the eyes will develop bags under them as well. 200.127.59.151 (talk) 13:38, 12 February 2008 (UTC)[reply]

HCl Crystal Structure

I cannot find confirmation of this but when frozen does HCl have the same crystal structure as NaCl? i.e. cubic close packed? I assumed that it has this structure because the bond is very ionic, but the proton is very light so this may cause something to happen to the crystal.

Oh and I'm taking about at liquid nitrogen temps ~79 K

Shniken1 (talk) 23:39, 11 February 2008 (UTC)[reply]

See doi:10.1038/213171a0. Sounds like some interesting content to add to the HCl article. DMacks (talk) 01:43, 12 February 2008 (UTC)[reply]
Thanks for that, it is wierd, it doesn't have HCl written anywhere, hence I couldn't find it. :P (Lesson 1.a try all possible chemical names!)Shniken1 (talk) 01:51, 12 February 2008 (UTC)[reply]

Circumcision Controversy

I was just noting today how often this crops up. Why is a minor medical/cultural procedure tinged with so much partisan fervour? Web sites with such fanatical language and claims are myriad. It's not the same for other such things. If one gets an eyebrow piercing there are not fanatical pro and anti camps who will bend logic to their respective ends. Or a pro and anti appendectomy "lobby". I'm guessing it's because it's our "intimate" parts and we hope we have the "right" configuration. I'm not sure if there is any research as to the psychology of circumcision opposition/advocacy, but arrogant opinion is welcome ;-) Fribbler (talk) 23:51, 11 February 2008 (UTC)[reply]

Well an appendectomy procedure is designed to prevent the death of the patient. Circumcision is usually just slicing off a bit of someone’s genitals without pressing medical need. Eyebrow piercing would be just as controversial if it was performed on babies who lack the ability to give consent. --S.dedalus (talk) 00:00, 12 February 2008 (UTC)[reply]
Edit conflict, ARGH! But as Sdedalus says, the main point is that babies cannot give consent for a precedure that is not needed —Preceding unsigned comment added by Shniken1 (talkcontribs) 00:04, 12 February 2008 (UTC)[reply]
Just to clarify: babies don't give consent for ear-piercing, and it is quite common in some cultures that have brought the practice to North America. I don't agree with it, personally, but I don't know of any lobby against it. ៛ Bielle (talk) 00:08, 12 February 2008 (UTC)[reply]
I didn't know they did that on babies... But anyway if you don't wear ear rings for a while don't the piercings close up? i.e it is essentially reversable?Shniken1 (talk) 00:10, 12 February 2008 (UTC)[reply]
I'm against circumcision myself, certainly in infancy. But then why such vehement pro-circumcision argument? It seems almost like a religious/personal/political type position in that people attach much emotion to their slant without science. Which I believe would generally not favour it. Fribbler (talk) 00:26, 12 February 2008 (UTC)[reply]
It has nothing to do with science. It has to do with parents who don't think anyone has the right to tell them they can't have what they see no more harmless than cosmetic surgery done to their kids. Someguy1221 (talk) 00:31, 12 February 2008 (UTC)[reply]
To respond to Shniken1, yes, the holes in the ears will close over if left alone, as would the eye-brow piercings referred to by S.dedalus. Both piercings and circumcisions are painful procedures done on infants, when there is no medical necessity to hurt them and no ability for them to give "informed consent". My own vote would be for child abuse, but I doubt I will find many kindred souls in this. (You did ask for arrogant opinions, did you not?) ៛ Bielle (talk) 00:39, 12 February 2008 (UTC)[reply]
I did! And agree with you aswell. But I still wonder just why it elicits such emotion. The websites say things like, and I paraphrase for emphasis, if pro: "If you don't get it done your child will be austracised as a FREAK!!!" (in the US I imagine, sure we dont really do it here) and if anti "Your child will be scarred for LIFE!!!" I just wonder if there are reasons for the emotional nature of the arguments. Fribbler (talk) 00:49, 12 February 2008 (UTC)[reply]
I think it's not so much a matter of circumcision there as it is that a fair proportion of humans are insane. And wherever a few crazies find an issue to go nuts over, more crazies will go nuts in response. Oh yeah, basic human nature ;-) Someguy1221 (talk) 00:51, 12 February 2008 (UTC)[reply]
It's really too bad, for many reasons, but for this one in particular, that User:SteveBaker is no longer active on the Ref Desks. He had, as I recall, very strong, scientific views against circumcision. ៛ Bielle (talk) 01:52, 12 February 2008 (UTC)[reply]
Since you asked nicely (and I care a lot) - I've copied my response from my Talk page below. SteveBaker (talk) 01:46, 19 February 2008 (UTC)[reply]
My belief is that people are tired of complaining about things that matter, that they decide to complain about something trivial just so they have something to complain about. You pick just about anything (e.g. how to hang toilet paper (there was a RD vote on this!), how much sugar to give to children, fast food, placement of water towers, Star Wars vs. Star Trek) and people will debate it to the end of time. It is mankind's nature to argue and whine and such things. Of course, you can debate whether or not I'm right about that. 206.252.74.48 (talk) 13:48, 12 February 2008 (UTC)[reply]
To Someguy: although there is no pressing medical need for circumcision, (indeed I find it frankly barbaric myself), it is not something which "has nothing to do with science". Circumcision eliminates the possibility of phimosis, a risk factor for penile squamous carcinoma and infections, and also of paraphimosis, which can be a medical emergency. I have also heard that circumcision may reduce HIV transmission rate (in some men, in some situations, etc.). In times past when personal hygiene practices were less stringent, the difference in anatomy may have led to a difference in cleanliness and been more important to penile and general health. Thus, although I wouldn't argue that, today, this choice is probably based more on the fact that daddy wants his kid to look like himself, I think it is presumptuous to say that circumcision has nothing to do with science. Tuckerekcut (talk) 00:22, 14 February 2008 (UTC)[reply]
One could use that argument to say that the amputation of the arms at the elbow eliminates the possibility of carpal tunnel syndrome. Sure, in those cases where a medical condition is present and needs to be addressed, such as phimosis, circumcision might be the best solution; and that's scientifically valid. But circumcising a perfectly health boy in this day and age because one day, maybe, perhaps, he might fail to clean himself properly and might get a nasty infection, so let's nip it in the bud before it ever gets to that stage - that's in no way scientific. Some religions and cultures require circumcision - and if that's the reason it's done to a particular boy, let's be upfront about it and say so. Not that I agree with such religious and cultural requirements, but they are a factor and it's pointless to prevaricate about it and try to justify them on scientific grounds, as I've heard various people do. If their religion or culture didn't require circumcision, it would never occur to the parents to consider it for a second. -- JackofOz (talk) 07:16, 15 February 2008 (UTC)[reply]
It's an issue of religion and of "the correct way to bring up a baby". That's two sure-fire ways to get serious arguments. AlmostReadytoFly (talk) 09:18, 12 February 2008 (UTC)[reply]
Toilet paper should obviously be fed OVER the roll. :P Emery (talk) 23:41, 12 February 2008 (UTC)[reply]
FFS I am so sick of you over the roll people. Without a doubt the best way is under the roll. GET IT THROUGH YOUR THICK HEADS YOU IGNORANT SO AND SOS!!!!Shniken1 (talk) 22:47, 13 February 2008 (UTC)[reply]
Look, Fribbler, it's not going to grow back, and the sooner you realise this and accept having 30% less gloriously erotic tissue compared to me, the sooner you will be able to appreciate what little remains to you and move on. Myles325a (talk) 01:07, 15 February 2008 (UTC)[reply]


Evolution is not wasteful. When creatures are trapped in a dark cave, within an amazingly few generations their offspring evolve to lose their eyes. If human males didn't need the foreskin then we'd undoubtedly have evolved without one...if (as often claimed) there are positive benefits to removing it then we'd most certainly have evolved without it. The fact that we still have it is absolute scientific PROOF that it has a function - even if we don't know what that function is.
The truth of it is this - people who were circumcised at a very young age have no choice in this matter and because that particular part of the male anatomy is something we're all terrified of being found wanting in - they are CERTAIN to claim they are better off - no matter what. The same thing applies to those of us who remain 'intact'. There is absolutely zero serious, peer-reviewed scientific evidence of health benefits either way - so all we're left with are claims of greater or lesser sensitivity and/or enjoyment during sex. But the trouble with that claim is that only a VERY few healthy men get circumcised after they become adults (the very thought makes my blood run cold!) - so only a very few people can testify after having sex under comparable conditions both with and without the foreskin...and those that are in a position to do so have ample reason to lie about the results because they don't want to seem like idiots for making the decision! So we don't have any proof of health benefits and we don't have proof of advantages or disadvantages during sex.
All we're left with is the sure and certain knowledge that we wouldn't have evolved to be the way we are if the foreskin had no purpose whatever. I'm personally pretty sure that the purpose is to cover the super-sensitive area of skin beneath in order to allow the nerve endings there to be protected until the act of intercourse when the foreskin tends to be rolled back to expose the most sensitive patch of skin on the entire body. Nerves that are exposed to daily wear and tear after circumcision are going to lose much of that extreme sensitivity.
So I strongly suspect that circumcised men are losing out on the experience - but I can't prove that.
From an evolutionary standpoint, we might imagine that men who have more fun during sex are more motivated to do it more often - that would imply that they would produce more offspring and therefore those men would be more prevalent in the gene pool. Hence the foreskin may well be a carefully evolved device that most certainly isn't meant to be chopped off at birth!
Sooner or later, science will figure out the truth - and then the debate will really begin. Right now we don't KNOW. But there are other parts of the body that we don't know the purpose of...do we go around routinely chopping off people's earlobes soon after birth? Are appendices removed "just in case" because we don't understand their function?
For parents (PARENTS!!) to go around mutilating the genitals of tiny helpless babies is one of the most disgusting and brutal practices imaginable to me. If parents went around removing the earlobes of their newborns in bizarre religious rituals they'd wind up in jail for sure! Thank goodness the world has realised this in the case of female genital mutilation and is working hard to outlaw the practice - let's just hope that as a society we can see the light in the case of the hundreds of millions of little boys whom their parents have so brutally mutilated in the name of some ancient barbaric tradition.
SteveBaker (talk) 01:46, 19 February 2008 (UTC)[reply]


February 12

sodium chlorate

what are the specifics of making sodium chlorate? or sodium perchlorate? —Preceding unsigned comment added by Steweydewey (talkcontribs) 01:20, 12 February 2008 (UTC)[reply]

Our articles on sodium chlorate and sodium perchlorate describe the synthesis of each. Someguy1221 (talk) 01:22, 12 February 2008 (UTC)[reply]

chemistry

magnesium carbonate is added to an added to an iron binding capacity determination in order to a. allow color to develope b. precipiate protein c.bind with hemoglobiniron d. remove excess unbound iron —Preceding unsigned comment added by 72.155.150.73 (talk) 01:38, 12 February 2008 (UTC)[reply]

Yes, quite probably one of a, b, c, d is the reason (or maybe a combination of those?). DMacks (talk) 01:40, 12 February 2008 (UTC)[reply]
A search eg http://www.google.co.uk/search?q=iron+binding+capacity+determination+magnesium&hl=en&start=10&sa=N suggests it may be 'd'. Who that helps if anyone?87.102.81.140 (talk) 12:55, 12 February 2008 (UTC)[reply]

Decrease of Earth’s orbital angular momentum (around the Sun)?

The motion impulse is sustained in case of zero gravity only as a straight line (?) If a turn or curve takes place this will lead to a reduction of the motion impulse. This applied to the motion impulse of the Earth would imply that the velocity of the Earth around the Sun is decreased. This because the gravitational impact of the Sun is such that the straight trajectory of Earth is curved. That is my question respectively my statement. This implies for Earth’s orbit around the Sun, that the radius is decreased. On the other hand leads the Sun’s burning to a decrease in mass and thus also to a decrease in gravitation. And that leads to an increase of Earth’s orbit. The total effect of these two partial effects remains open. Is this sound reasoning? Or can somebody explain me why the orbital angular momentum of the Earth should remain the same during all these millions of years (despite the gravitational impact of the Sun)? 77.57.61.59 (talk) 01:49, 12 February 2008 (UTC)[reply]

The Earth is in freefall. It doesn't require any energy input and can go on forever. --Sean 02:42, 12 February 2008 (UTC)[reply]


Additionally, the Sun's loss of mass, while existent, is quite negligible. For instance, if the Sun were to completely exhausts its hydrogen supply, it would have lost about 0.7% of its mass. In practice, the Sun won't come anywhere near fully consuming its mass of hydrogen (most of it remains well outside the core where fusion takes place), so something like a tenth of a percent is a better estimate for the mass lost in such fashion. I would expect that solar wind loss is similarly small. — Lomn 14:11, 12 February 2008 (UTC)[reply]

Using a 200V machine in a 220-240V country.

A machine has these power specs:
3 phase 200V
50/60Hz
1.6kw

Australia is rated at 220-240V.

If I acquire this machine, is it okay to use?
If not, is there a feasible solution?

Phrased properly: What difference in voltage ratings between a machine and a country is acceptable?
Rfwoolf (talk) 03:02, 12 February 2008 (UTC)[reply]

Volunteers on an online encyclopedia are not a suitable source of information for such a question. Consult a licensed professional electrical engineer in your country for advice. That said, the supplied voltage will be connsiderably greater than the rated voltage. This could over stress the insulation and contacts, causing catastrophic failure of the equipment. You might contact the manufacturer to see their recommendations. One remedy would be using a transformer to adapt the machine to the excessive voltage. Another would be to have qualified personnel rewind or otherwise modify it to operate on your voltage. It is surprising that the machine has one rated voltage rather than a range of allowed voltages, since power equipment may see voltage somewhat above or below the "official" supply voltage during actual operation. Edison (talk) 03:20, 12 February 2008 (UTC)[reply]
She'll be fine mate. Give it a whirl.Shniken1 (talk) 03:54, 12 February 2008 (UTC)[reply]
Does it have a motor in it?Tuckerekcut (talk) 04:50, 12 February 2008 (UTC)[reply]
Yes. Rfwoolf (talk) 05:00, 12 February 2008 (UTC)[reply]
Standard residential power is single-phase, not 3 phase. I suspect that the motor you are looking at is what we call here (Canada) 208 volt 3 phase. Such motors will not work using 230V single-phase power (which is what will be available in residential buildings in Australia). This motor will be equipped with three conductors (plus ground) instead of two and there will be no way to wire it. As stated above, we are not a reliable source of information, but I can warn you that connecting 3-phase equipment to a single-phase power supply is by no means safe. —BradV 07:02, 12 February 2008 (UTC)[reply]

Thanks for all the help. I've only just found out about the 3 phase thing, which I did look up, and now I realise that only certain factories are connected to 3-phase, and we wouldn't have that available, so that puts this issue to bed.
However, for my own education, the question still remains, what difference in voltage ratings between a machine and a country is acceptable? What would happen if the above machine was a 1-phase 200V. Could I use it in Australia which has 220-240V?
Rfwoolf (talk) 07:40, 12 February 2008 (UTC)[reply]

This is however an extremely theoretical question since no country has a 200V supply (see Mains power systems) so no one bothers to manufacture equipment for a 200V supply Nil Einne (talk) 09:01, 12 February 2008 (UTC)[reply]
I think you'll find that Japan has areas of the country where industrial power is 200V (and residential power is 100V). This diversity (between the low of 200 V and the high of 240 V) is a recurring challenge for the equipment that my company designs.

All houses in Japan receive 200V. Most machines are 100v so only two wires are used. But there are many machines that are 200V used in houses in Japan. For example, larger air conditioners, cookers, dishwashers, and so on. The government had a plan to convert everything to 200V because of the more efficient energy costs, but this has not happened. All the same, 200V machines are not unusual and easy to install.

Atlant (talk) 13:11, 12 February 2008 (UTC)[reply]
If that device can tolerate such overvoltage, it will work (although it will heat up more)(it also might reduce it's lifespan). If that device cannot tolerate such overvoltage, it will either blow fuse or it will get damaged. -Yyy (talk) 09:57, 12 February 2008 (UTC)[reply]
I didn't answer before because I couldn't get my head around what was meant by a voltage in a three-phase system. I was hoping an Australian EE would answer. Our article "Mains electricity#Voltage levels" says that very often loads for 230V supplies will be marked "200V". --Milkbreath (talk) 11:56, 12 February 2008 (UTC)[reply]
Have you seen Three-phase electric power, Polyphase system and Three-phase? Nil Einne (talk) 15:39, 12 February 2008 (UTC)[reply]

Many large industrial scale electrical devices are built with tapped windings which allow the same machine to be connected to several different voltage levels. The full winding is used when it is connected to the highest rated voltage, and there are taps along the winding to allow it to be connected to somewhat lower voltages. The same motor might be capable of connection to a delta or wye three-phase supply as well. A supplier who has a motor with various taps and connection options needs stock only one motor to handle demands for, say 208 volt or 240 volt supply. Likewise transformers or autotransformers can be used to supply the desired voltage, but this is not a cheap option. If a factory had to install their own transformer because their local utility was supplying them with a higher voltage such as 4,000, 12,000 or 34,000 volts, then it might be quite economical to step the supply voltage down to the quirky voltage required by the electrical device. If a motor is operated on a higher than normal voltage, it may actually draw less current (depending on the type of motor), while it would draw more current if operated at lower than normal voltage. For these motors, low voltage would cause overheating due to high current, but high voltage would stress the insulation rather than overheating the conductors. When only single phase power is available, a phase convertor can sometimes be used to allow the use of a three phase motor. Or a second phase can be brought to the location from the utility's nearest three phase supply point, and the motor could be operated open delta. The most expensive option, and the one with the best results, is to bring in the other two phases. Again, an EE qualified to analyze and design power applications is the best person to advise on this. Edison (talk) 15:24, 12 February 2008 (UTC)[reply]

But you'd still have major problems getting the device to work in Australia though wouldn't you? Since three phase power in Australia is of course 415 V not 208 V... Nil Einne (talk) 15:33, 12 February 2008 (UTC)[reply]
The voltage tolerance will depend on the specific machine. I worked with one that had transformer taps for every common voltage from 100 to 415, plus additional taps around those voltages for better matching. Proper installation required matching voltages to +/- 2.5 volts. (Incidentally, it's also the only machine I've ever used to have water-cooled wires.) --Carnildo (talk) 21:45, 13 February 2008 (UTC)[reply]

Strange light

Hello. A few nights ago, I saw a golden sphere in the sky. At first I thought it was a star, but I realized it was moving. Then the light started to dim, and then it disappeared. I don't know what this was. Does anyone have any ideas? Thanks. JetLover (talk) (Report a mistake) 03:32, 12 February 2008 (UTC)[reply]

How did you know it was a sphere?Shniken1 (talk) 03:52, 12 February 2008 (UTC)[reply]
From my viewpoint, it looked like a sphere. JetLover (talk) (Report a mistake) 03:54, 12 February 2008 (UTC)[reply]
See Ball lightning --S.dedalus (talk) 04:50, 12 February 2008 (UTC)[reply]
Since this appears to be an UFO I would suggest you look at the page and associated external links. Avoid anything which talks about conspiracy, aliens etc seriously. Look for stuff which discuss which try to explain or suggest logicial possibilities for UFO sightings Nil Einne (talk) 16:02, 12 February 2008 (UTC)[reply]
You don't say where you are but a few nights ago, Saturday to be precise, there was an extremely bright sky transit of the International Space Station from west to east at 18.14 hours over southern England. It took about 4 minutes to go from the western horizon to the eastern and it certainly rises as a golden/red colour and sets the same colour but is a bright white overhead. Richard Avery (talk) 09:16, 12 February 2008 (UTC)[reply]
Similar to a question lower down, could it be a satellite flare? They're bright, they move, they dim. -- Consumed Crustacean (talk) 21:47, 12 February 2008 (UTC)[reply]
Hmm, sounds like an iridium flare to me. You can also use Heavens-Above to search for Iridium flares and other satellites, in the future, as well as in the past. Be sure to use your exact city, otherwise the predictions for both the past and future will be off. Hope this helps. Thanks. ~AH1(TCU) 22:31, 12 February 2008 (UTC)[reply]
Oh, by the way, sometimes the ISS will also hit a point in the sky, start to dim, then dissapear, as it climbs over the shadow of the Earth. You can use Heavens-above for ISS too. I even wrote a Wikihow article on finding an iridium flare/ISS. Hope this helps. Thanks. ~AH1(TCU) 22:34, 12 February 2008 (UTC)[reply]
It probably was a satellite flare. Thanks for the help. JetLover (talk) (Report a mistake) 00:58, 13 February 2008 (UTC)[reply]
I doubt it was an iridium flare as these tend to be fleeting - a second or three, whereas the ISS flare, if it can be called a flare can last for 3 - 4 minutes as it tracks across the sky. Iridium flares are, in my experience, quite difficult to observe due to their unpredictability and dimness. Richard Avery (talk) 07:55, 13 February 2008 (UTC)[reply]

time of twilight

is there a mathematical equation to calculate the amount of monutes from sunset to dusk in different parts of the world. for example in NYC how many minutes after sunset will it be completly dark.Yribowsky (talk) 03:49, 12 February 2008 (UTC)yribowsky[reply]

I doubt it. I think it depends a lot on landscape. AlmostReadytoFly (talk) 09:51, 12 February 2008 (UTC)[reply]

The length of time from sunset until it gets dark - twilight - depends on the latitude and on the declination of the sun. The further north or south one goes from the equator, the longer will be the period of twilight. The declination of the sun depends on the season of the year. Therefore in a particular place, the length of twilight will vary throughout the year. To do a mathematical calculation on the length of twilight requires a knowledge of spherical trigonometry.Simonschaim (talk) 13:26, 12 February 2008 (UTC)[reply]

The Twilight page has several external links to twilight time calculator websites, including Length of Day and Twilight (Formulas), which begins by saying: This article describes, how the length of day can be calculated for any given Northern latitude and any day of year. It also includes calculation of the twilight duration. Pfly (talk) 06:07, 13 February 2008 (UTC)[reply]

Ok, so much for the time bizzo. Now, how do we calculate the number of “monutes” Myles325a (talk) 01:26, 15 February 2008 (UTC)[reply]

Nearest star of spectral class O

What Class O star is nearest to Earth? --Cam (talk) 04:30, 12 February 2008 (UTC)[reply]

Zeta Ophiuchi maybe? 458 light years away. Clarityfiend (talk) 06:11, 12 February 2008 (UTC)[reply]
Thanks! Looking around on the web, it does seem like it's the closest, but I can't find a definitive statement anywhere. --Cam (talk) 02:22, 13 February 2008 (UTC)[reply]

Space?

Watching my usual satellite passing over at 0645 this morning, (in Scotland), I was surprised not to be able to any stars. Since I believe that satellites circle 2-300 miles above the earth, what could have been between the satellite and the stars to obscure them. Too high for clouds, or am I wrong? Any ideas please.--Johnluckie (talk) 08:29, 12 February 2008 (UTC)[reply]

Are you sure it wasn't simply too bright at the time to see anything else (too much light pollution or perhaps it was dawn?)? You didn't mention what satellite this was but I assume you are referring to a satellite flare rather then simply observing the satellite. Satellite flares can sometimes be fairly bright, for example Iridium flares can sometimes be -8 magnitude which is a lot brighter even then Venus (~-4 i.e. about 40 times dimmer), and Sirius (-1.5) doesn't even come remotely close Nil Einne (talk) 08:52, 12 February 2008 (UTC)[reply]
Thanks for this, how do I differentiate between the satellite and it's flare? —Preceding unsigned comment added by Johnluckie (talkcontribs) 09:02, 12 February 2008 (UTC)[reply]
Well, the satellite itself would just look like a star moving across the sky in a straight line. A flare is just a flash. —Preceding unsigned comment added by Mattbuck (talkcontribs) 12:12, 12 February 2008 (UTC)[reply]
To clarify, a flare lasts a few seconds and you can see it moving during that time. You can look up Iridium flares at Heavens-Above. Icek (talk) 15:34, 12 February 2008 (UTC)[reply]
Thanks for these answers. I did see the satellite traversing for at least five minutes as I sat in my hot tub in the garden. It was dawn, I did see one faint star, but I still ask where were the other stars!--Johnluckie (talk) 16:44, 12 February 2008 (UTC)[reply]
According to apparent magnitude a non-flaring ISS can still be ten times brighter than any star, under the right conditions. Algebraist 17:30, 12 February 2008 (UTC)[reply]
You can look up ordinary satellite passes at Heavens-Above as well (but just now the website seems to be down). Icek (talk) 18:14, 12 February 2008 (UTC)[reply]
Huh? Heavens-above seems to be working fine for me. Anyway, if you were able to see one "star" at dawn, then most likely it was Venus or Jupiter. Maybe the dawn twilight washed out all the stars, or maybe there were thin clouds that were thin enough to let the ISS and the brightest stars be visible, or maybe there was too much light pollution in the area. Anyways, you can use http://fourmilab.ch/yoursky for creating star maps of your area (try finding out yoor coordinates and entering them into the yoursky. Hope this helps. Thanks. ~AH1(TCU) 22:27, 12 February 2008 (UTC)[reply]

Thanks for all these splendid answers and links; much appreciated.--Johnluckie (talk) 07:44, 13 February 2008 (UTC)[reply]

Golf

At home, I have a golf club that is much lighter than normal. But why does it hit things with the same force as a normal golf club? Interactive Fiction Expert/Talk to me 11:31, 12 February 2008 (UTC)[reply]

The lighter club may allow you to swing faster, which can compensate for the loss of mass. Additionally, many modern woods and drivers are engineered with a degree of "springiness" to better hit long drives. Our golf club and wood articles discuss this to some extent. The howstuffworks article on clubs may also be useful. — Lomn 14:05, 12 February 2008 (UTC)[reply]

Calculating power use

I am trying to figure out how much power my computer uses in standby mode. I found rates of 1-6 watts on several reference sites. What I can't figure out is how to convert that to kiloWatt hours, the units used on my electricity bill. Watt explains that the time is included in the unit, specifically one joule per second, so does 1 watt mean I have to multiply by 3600 to get the amount of power used in an hour? That gives me an unrealistically high number. It's been so long since I've done this kind of simple physics (ten years!) that I can't remember how to do it, and I am consternated. --Ginkgo100talk 16:01, 12 February 2008 (UTC)[reply]

1 watt is one joule per second. However, no-one uses joules for electricity bills, they use kilowatt-hours. A kilowatt hour is by definition the energy expended in maintaining a power of 1 kilowatt (=1000 watts) for one hour (so it =3,600,000J). Thus if your computer is running at 1 watt (say) it will take a thousand hours (approx. six weeks) to use a kilowatt-hour of electricity. At 6W, it'll take about a week. Algebraist 16:07, 12 February 2008 (UTC)[reply]
or to put it another way 1W = 1/1000 (0.001) kWHr if used for an Hour. So 1kWHr of electricity will run a 1W computer for a thousand hours. and a x watt computer will run for 1000/x hours (divide) for the price of 1 kilowatthour.87.102.81.140 (talk) 16:40, 12 February 2008 (UTC)[reply]
So in other words I divide by the time rather than multiply. Thanks, I knew I was doing something horribly wrong! --Ginkgo100talk 16:45, 12 February 2008 (UTC)[reply]
Wait... that's not it either... but I will use your formula. --Ginkgo100talk 16:46, 12 February 2008 (UTC)[reply]
Multiply by the number of hours you use it, to get Watt-hours, then divide by 1000 to get kiloWatt-hours. 81.174.226.229 (talk) 16:55, 12 February 2008 (UTC)[reply]
Actually it's (watts of computer )x 3.6 x (hours used) to get the number of kilowatthours required —Preceding unsigned comment added by 87.102.81.140 (talk) 17:03, 12 February 2008 (UTC)[reply]
Huh? to get from watts to kilowatts you multiply by 3.6? that would cost me over $10000 a month to run one light bulb. 81.174 has it right —Preceding unsigned comment added by Mad031683 (talkcontribs) 17:19, 12 February 2008 (UTC)[reply]
Damn SineBot wastes no time. Mad031683 (talk) 17:20, 12 February 2008 (UTC)[reply]
No, it's not. You're thinking of Joules. Watts to Watt-hours is a multiplication, which is why they're called Watt-hours.
  • If you take Watts and multiply by the number of seconds (i.e. 3600 times the number of hours) you get W.s=Joules.
  • If you take Watts and multiply by the number of hours you get W.h=Watt-hours
  • If you take Watt-hours and divide by 1000 you get kWh
(I'm IP 81.etc.) AlmostReadytoFly (talk) 17:56, 12 February 2008 (UTC)[reply]
There's an article Kilowatt hour "One watt hour is the amount of energy expended by a one-watt load (e.g., light bulb) drawing power for one hour."
which is of course (3600 x 1 watts){answer is in joules} since there are 3600 seconds in an hour, and time (seconds) x power (joules per second = watts) = energy (in joules).87.102.9.73 (talk) 19:20, 12 February 2008 (UTC)[reply]
I understand now thanks to the various comments here... I know how 87.102 got 3.6 even though it's wrong, and I know how to figure that it's not a big deal to leave my computer on stand-by at night. Thanks to everyone who responded! --Ginkgo100talk 03:43, 15 February 2008 (UTC)[reply]

Killing a cold virus by Exercising?

Good morning, At work, we’ve had some sort of cold virus that has infected many of my team members. One colleague claims that he was “not feeling well” and running a slight fever on Saturday. By Sunday, his temperature was just over 100 degrees Fahrenheit (38 Celsius). Despite the high body temperature, he went to the gym Sunday afternoon to exercise and workout. By today, he claims that he is much better and now has a normal body temperature. He said he “cooked” the virus (killed it) by exercising. He said when he exercises, his body temperature rises, and that temperature is too high for the virus to survive. Thus he is cured by exercising at the onset of his cold. Is this plausible and make any sense? Thanks! Rangermike (talk) 16:50, 12 February 2008 (UTC)[reply]

Hmm, I'm pretty sure that (at least heavy) exercise while sick can cause myocarditis. Not good! -- Aeluwas (talk) 17:02, 12 February 2008 (UTC)[reply]
See fever. An elevated body temperature can inhibit the reproduction of some pathogens while increasing the production of white blood cells. So, it is valid to claim that a fever helps fight an illness. As far as exercising, increased blood flow and raised blood pressure will move blood-borne pathogens throughout the body at a faster rate. It is valid to claim that this could lead to further infection. All in all, unless you do a controlled experiment all you will have is opinions. -- kainaw 17:11, 12 February 2008 (UTC)[reply]
(Question undeleted. If you want it deleted, you must discuss it on the talk page. -- kainaw 18:22, 12 February 2008 (UTC))[reply]
Wouldn’t it be similar and less dangerous just to raise the thermostat, take a hot bath, bundle up, and drink hot liquid? --S.dedalus (talk) 02:36, 13 February 2008 (UTC)[reply]
I have heard of "sweating out" a cold in a sauna or steam room. It did feel like it was working, but you know... AlmostReadytoFly (talk) 07:09, 13 February 2008 (UTC)[reply]
When I was a little kid and had a high fever my mother would put me naked in the bathtub and pour rubbing alcohol on me with the fan blowing on me (to evaporate the cooling alcohol) because it would make me really cold. She thought this was lowering my temperature but it was sheer torture which I remember as akin to child abuse. As an adult, perhaps in traumatic response to this, whenever I get a fever and feel sick and chilled, I immediately take the longest and hottest bath I can manage and then get back in bed with lots of covers to sweat some more. I can say that I have taken antibiotics just once since 1984 and have not had any fever/sickness that lasted over a day. That said, this is a personal anecdote from my life and it not intended as information to be used to diagnose, treat or cure any disease experienced by another human being. Saudade7 08:52, 13 February 2008 (UTC)[reply]

Hmm, I'm pretty sure that (at least heavy) exercise while sick isn't a good idea since you are wasting energy on working out, when you want to conserve that energy for fighting off the virus. 64.236.121.129 (talk) 15:00, 14 February 2008 (UTC)[reply]

Questions about EMP

Will EMP affect electronics that are turned off? There are electrical signals in the human brain. Could an extremely powerful EMP affect humans at all? 64.236.121.129 (talk) 17:15, 12 February 2008 (UTC)[reply]

It is possible for turned off electronics to get a shock from nearby turned on electronics. An overly-simplified way to think of it is to consider dunking the electronics in water. If you put two radios in water, one turned on and one turned off, it is possible that the one turned off could still be damaged. As for humans, I do not know of any EMP tests on humans. I have seen studies on rats - including a good one that had two control groups and tested the rats for three days on maze-learning tests. In general, they find that there is a reduction in brain function following extremely dangerous EMP shocks, but those are repaired over time. It is theoretically possible to do enough brain damage to kill a person (or animal) with EMP. -- kainaw 17:39, 12 February 2008 (UTC)[reply]
There is a technique called Transcranial magnetic stimulation that does exactly that: sends an EMP through the skull into the brain, affecting the brain areas closest to the EMP source. The EMP source is a particular configuration of coils placed over the scalp. --Dr Dima (talk) 19:45, 12 February 2008 (UTC)[reply]
I'm pretty sure that magnetic resonance imaging uses intense magnetic fields, but doesn't scramble brains. I might have my forces confused, though. --Mdwyer (talk) 23:50, 12 February 2008 (UTC)[reply]
Let's make a distinction between these three electromagnetic phenomena.
  • MRI uses a strong static magnetic field, plus a very small radio-frequency electromagnetic wave. A static magnetic field doesn't induce currents.
  • Transcranial magnetic stimulation essentially uses the brain as the secondary winding of a transformer. A changing magnetic field induces an electric field that drives currents in one region. There is no electromagnetic wave propagating in free space. Getting a desirable outcome, instead of damage, is due to the practitioner and the manufacturer controlling the induced electric field.
  • EMP is an electromagnetic wave generated in a medium of charged particles. It propagates through free space.
When you want to know the "damage" caused by any of these things, you primarily need to ask "how much current is made to flow?" Bigger electric fields push more current; more current causes more heating, burning, and other damage. JohnAspinall (talk) 16:08, 13 February 2008 (UTC)[reply]
I suggest an experiment: Construct an aerial system connected via a (working) semiconductor diode to ground. Then let off a family size nuclear device fairly nearby (but shielded from the heat and blast etc). When the explosion has died down, examine the diode for functionality. I think you'll find its dead (like a lot of people in the vicinity) —Preceding unsigned comment added by 79.76.189.102 (talk) 00:50, 13 February 2008 (UTC)[reply]
Your use of the term "family size" made me imagine them being sold at Sam's Club. 206.252.74.48 (talk) 13:46, 13 February 2008 (UTC)[reply]
They are used as a sterilizing agent; you'll find them in the cleaning supplies section. -SandyJax (talk) 19:08, 13 February 2008 (UTC)[reply]
Reminds me of Back to the Future where 1955 Doc believes plutonium to be readily available in 1985 drugstores. 206.252.74.48 (talk) 20:34, 15 February 2008 (UTC)[reply]

Are all indoor heaters 100% efficient?

If an electric space heater is heating up a room, and is not completely efficient in its method of operation, presumably all inefficiency is lost as waste heat. But if you consider that waste heat to be useful in itself (it also heats up the room), can the heater be said to be 100% efficient (thereby breaking the law)? If not, where did the lost energy go? Thanks. --Sean 17:18, 12 February 2008 (UTC)[reply]

Yes indeed. It is 100% efficient. But it doesn't break the 2nd law of thermodynamics. If the goal is to get heat, then the so called "waste heat" is not waste. 64.236.121.129 (talk) 17:22, 12 February 2008 (UTC)[reply]
Not quite, if it produces light and sound (like a vibration say), then not all of the energy is heat. Wisdom89 (T / C) 17:25, 12 February 2008 (UTC)[reply]
What if it wastes energy producing ozone or even X-rays? There is a lot of room for energy "loss". -- kainaw 17:30, 12 February 2008 (UTC)[reply]
If it produced x-rays or ozone, the person using the space heater would be dead. 64.236.121.129 (talk) 17:34, 12 February 2008 (UTC)[reply]
Wouldn't you also have to take the generator into account? Mad031683 (talk) 17:37, 12 February 2008 (UTC)[reply]
Why would the user be dead? While X-rays are not extremely common in household appliances, ozone is common in those ionizing air cleaners. I haven't heard of anyone getting killed by using an ionizing air cleaner. -- kainaw 17:40, 12 February 2008 (UTC)[reply]
X-rays are actually common in CRT monitors, but they are stopped by the glass. Space heaters are not 100% effient because they do not release all their energy as heat, some of it is lost to other spectrums, including visible light if you can see it glowing. 206.252.74.48 (talk) 18:44, 12 February 2008 (UTC)[reply]
Well, assuming there are no windows, any light produced is still 100% efficiently heating the room, right? The electricity gets turned into heat, which makes the element do some black-body radiating, and those photons then plink me in the eyeball and warm me up a little, so there's no loss of efficiency. Same goes for sound, but perhaps not for the ozone production mentioned above, since that stores energy in chemical bonds. --Sean 19:13, 12 February 2008 (UTC)[reply]
You were talking about electric heaters I assume - in which case the answer is (excluding doubters and niggles) YES!87.102.9.73 (talk) 19:30, 12 February 2008 (UTC)[reply]
Guys. Learn some thermodynamics. please. Not only is it possible to make a heater exactly %100 efficient, it's possible to make a heat pump provide more heat output than the total free energy input. See Coefficient of performance. It doesn't break the first law, because the energy is coming from an external heat source, and it doesn't break the second law, because the total entropy increases, making it an irreversible process. So, if you want to heat up a room, a %100 efficient heater is actually pretty wasteful compared to a heat pump. —Keenan Pepper 19:10, 12 February 2008 (UTC)[reply]
The question wasn't whether or not it was possible to construct a 100% efficient heater, it was whether or not all indoor heaters are 100% efficient. The answer to this question is indubitably "no". Wisdom89 (T / C) 19:43, 12 February 2008 (UTC)[reply]
except they (fan, convection, oil storage) practically are 100% effecient eg (99%) making the answer an indubutable 'yes' ?87.102.9.73 (talk) 21:20, 12 February 2008 (UTC)[reply]
Um except 99% is not 100%... This is important, not nitpicking, if something is 100% efficient then it is 100% efficient, not nearly 100% efficient. Nothing can be 100% efficient which I presume was the point Nil Einne (talk) 09:07, 13 February 2008 (UTC)[reply]
Don't I need something (a resovoir) that is already hot (to put it explicitly) to get a heat pump to pump more than 100% in heat of the energy it expends - ?? Just checking, not really sure......87.102.9.73 (talk) 19:28, 12 February 2008 (UTC)[reply]


If sound is produced most of it will be absorbed within the room and heat it up, light (even a lot of xrays) will absorbed within the room and heat it up. Ozone will decay to oxygen, releasing heat. So yes a heater is 100% efficient if you define what you want as heat (in any form). If you define what you want as 'heating the air' or 'increasing the temperature of the air in the room' then no it will not be 100%.Shniken1 (talk) 23:03, 12 February 2008 (UTC)[reply]
Most but not all Nil Einne (talk) 09:08, 13 February 2008 (UTC)[reply]

By the definitions here (no windows, sound stays in the room, etc) many appliances (electric fans, vibrators, etc) would be 100% efficient space heaters. Edison (talk) 19:09, 13 February 2008 (UTC)[reply]

Well, if it emits even a single neutrino, it's no longer 100% efficient :)) --V. Szabolcs (talk) 19:53, 18 February 2008 (UTC)[reply]

Eyeglass prescription relation to 20/20 scale

I've been trying to figure out what my numbers on the 20/20 scale are, and the Snellen Chart doesn't help because I can't see any letters on it at all. My eyeglass prescription is roughly -8 dioptres for each eye. Is there some formula or trick to find out what my eyesight is using this number? 206.252.74.48 (talk) 17:31, 12 February 2008 (UTC)[reply]

You can't see anything on the Snellen chart? Are you considered legally blind? -- MacAddct  1984 (talk • contribs) 18:10, 12 February 2008 (UTC)[reply]
At -8, quite possibly if they want to be. Without my glasses, I'm not convinced I could see anything on it from the proper distance. Skittle (talk) 18:27, 12 February 2008 (UTC)[reply]
Ah, from the article "The biggest letter on an eye chart often represents an acuity of 20/200, the value that is considered "legally blind." Many people with refractive errors have the misconception that they have "bad vision" because they "can't even read the E at the top of the chart without my glasses." But in most situations where acuity ratios are mentioned, they refer to best corrected acuity. Many people with moderate myopia "cannot read the E" without glasses, but have no problem reading the 20/20 line or 20/15 line with glasses. A legally blind person is one who cannot read the E even with the best possible glasses." Skittle (talk) 18:33, 12 February 2008 (UTC)[reply]
So, 20/20 just refers to how far you can see compared to normal people. You know that your vision is worse than 20/200 without glasses, but with glasses it's probably rather better. I've had a look for any easy correlations, but can't find any. Skittle (talk) 18:37, 12 February 2008 (UTC)[reply]
To clarify, when I say "see" I mean "see clearly". I can see the letters on the chart without my glasses, but they are just black and gray blobs. My eyesight with glasses is a little better than 20/20, but I want to know what it is without them. 206.252.74.48 (talk) 18:41, 12 February 2008 (UTC)[reply]
Yeah, I've been there... The Snellen chart simply isn't really that useful to extreme myopes. The 20/200 number just says that you can see at 20ft what someone else can see at 200ft. So to find the right number, you need to find a gigantic 'E', or do some extrapolation. I think the math is in the linked article. --Mdwyer (talk) 19:54, 12 February 2008 (UTC)[reply]
I did some math a while ago, it came out to 20/14400 - which for some reason I don't think is quite right. That means that I have to be within 20 feet to read an enormous 'E' that someone with good vision can read from nearly 3 miles away. The article never states what prescription would be given to someone with 20/200 vision. 206.252.74.48 (talk) 20:06, 12 February 2008 (UTC)[reply]
The article does point out that the 20/40 line is twice the size of the 20/20 line. So, if there was a 20/14400 line, it would be 720 times as big as the 20/20 line. It doesn't give the exact size of the 20/20 line. Assume it is 1 inch. You are talking about a 20/14400 line that is 720 inches (60 feet) tall. It is perfectly reasonable to assume that a person with 20/20 vision could see a 60 foot letter E that is 2.7 miles away. Just looking out my window right now, I can see a few 5-7-story buildings, which is roughly 60 feet, that are at least 3 miles away. I can clearly make out the windows. So, if they pasted a big letter on the side of the building, I'm sure I could tell you what letter it was. Now, could your vision be so bad that you couldn't make it out while standing on the opposite side of the street? It is possible. I don't know how bad your vision is. -- kainaw 20:17, 12 February 2008 (UTC)[reply]
Ah, thanks for clarifying that. If I stand across the street and look at a 60 ft tall letter, I can only tell what it is because of the general shape (since at these scales seeing a letter clearly is not needed), but I won't be able to see it clearly at all, it would just be a blur in the general shape of an 'E', which probably means that my calculations are correct. I am suddenly reminded of an ad at my local mall for eyeglasses that is painted in huge letters on the wall. I can't read any of the 10 ft tall letters from 20 ft away without my specs. Like Mdwyer said, the Snellen chart becomes quite useless after 20/200, but I'm sticking with 20/14400 until greater logic or mathematics prevails. 206.252.74.48 (talk) 20:31, 12 February 2008 (UTC)[reply]
Keep in mind that the Snellen chart isn't intended to measure diopters. It is only a subjective measure of acuity. So, I don't think there really is a snellen::diopter mapping. For that matter, diopters change depending on the position of the lens. That's why my glasses were -8, my contacts with -9.5, and my implants are something like -11. I really like the 20/14400 number, though. I always just told people I was off the chart. --Mdwyer (talk) 23:48, 12 February 2008 (UTC)[reply]

Photosynthesis vs photovoltaics

Which one is more energy efficient? Which method derives more energy from a given amount of sunlight? 64.236.121.129 (talk) 17:33, 12 February 2008 (UTC)[reply]

The photosynthesis and high efficiency solar cells both give values for efficiency. Have to be careful with your terms: does "given amount" mean overall, or of the usable part of the spectrum for each method, and do you mean useable energy (after processing into some externally-accessible form) or just the initial light→energy conversion? DMacks (talk) 20:46, 12 February 2008 (UTC)[reply]


Tandem Repeats

Are tandem repeats the same as regulatory genes that sit above a certain gene and the lengths of the repeats are when to turn on or off? —Preceding unsigned comment added by 67.121.105.145 (talk) 20:33, 12 February 2008 (UTC)[reply]

No, tandem repeats are not the same thing as regulatory sequences. --mglg(talk) 21:31, 12 February 2008 (UTC)[reply]

But don't the tandem repeats control the turning on or off of a gene? —Preceding unsigned comment added by 67.121.105.145 (talk) 02:32, 13 February 2008 (UTC)[reply]

In a (genetically) healthy individual, no; tandem repeats are not part of a normal cell's mechanism of controlling gene expression. However, there are some diseases, most notably Fragile X syndrome, which are due to trinucleotide repeats. You may be interested in reading about trinucleotide repeat disorders. (EhJJ)TALK 03:37, 13 February 2008 (UTC)[reply]
Minor nit: There's no strong reason to consider fragile X more notable than other trinucleotide repeate disorders. --David Iberri (talk) 03:43, 13 February 2008 (UTC)[reply]
Good point. It was just the example that kept coming up at my med school, and was the first that came to my mind. (EhJJ)TALK 16:39, 13 February 2008 (UTC)[reply]
Well it depends, the number of repeats in this case do cause the transcription to abort i.e. a little like a regulatory defect, whereas in Huntington's disease the repeats lead to many extra amino acids in the protein. In this latter case the protein does not function but there is no change in the production of transcript. Given the original question was from a regulatory perspective Fragile X is a more appropriate choice. David D. (Talk) 03:44, 14 February 2008 (UTC)[reply]

empty space?

I read some where that on a moecular level the human body is 99% free space. When atoms bond there is space between them, much like filling a box full of balls. Is this true and if it is what is in the empty space? cris —Preceding unsigned comment added by 24.183.237.19 (talk) 22:21, 12 February 2008 (UTC)[reply]

There's some but not much space between atoms, and only sometimes any at all: a covalent bond is literally when the electron clouds of adjacent atoms overlap. However, atoms themselves are mostly empty space: a very small nucleus with a very large electron cloud extending far beyond it. DMacks (talk) 22:30, 12 February 2008 (UTC)[reply]
In the Standard Model of particle physics, elementary particles are assumed to be point-like objects with no size - so every object, including the human body, is actually 100% empty space. In various alternative (and currently speculative) theories such as string theory, elementary particles have extension and are not dimensionless points - but they are still unimaginably small compared to the distances between them. However, "empty space" is, in fact, a sea of transient virtual particles, so it is not exactly "empty" anyway. If all this seems weird, that is just because our usual working models of reality which we have built by interacting with everyday objects just don't apply at sub-atomic scales. Gandalf61 (talk) 22:37, 12 February 2008 (UTC)[reply]
I'm not sure I'm happy with either of the answers above. The Standard Model is a field theory; space is occupied to the extent that fields can be said to occupy space. Field strength is never exactly zero anywhere, so you can't draw a distinction between empty and nonempty space without inventing an arbitrary cutoff where you consider the field to be weak enough that it doesn't matter. You can choose the cutoff such that the whole universe is occupied, or such that interstellar space is mostly empty but human beings are solid, or such that human beings are mostly empty except for widely-spaced nuclei, or such that the whole universe is empty. As for the zero-point energy, I'm not sure I believe in it at all. It doesn't have any physical consequences in the Standard Model (not even the Casimir effect, as discussed in this paper), and the one physical consequence you might expect it to have in quantum gravity is famously absent (the cosmological constant problem). Describing the zero-point energy as a sea of virtual particles is even more dubious, since the virtual particles only appear in individual Feynman diagrams, and only a sum over many diagrams (with different virtual particle lines) has any physical significance. -- BenRG (talk) 17:26, 13 February 2008 (UTC)[reply]
BenRG - yes, the Standard Model is a field theory, but nevertheless it contains point-like particles and uses quantum fields to describe these point-like particles - our quantum field theory article says "Quantum field theory thus provides a unified framework for describing "field-like" objects (such as the electromagnetic field, whose excitations are photons) and "particle-like" objects (such as electrons, which are treated as excitations of an underlying electron field)".
And when you say "the virtual particles only appear in individual Feynman diagrams" are you saying that virtual particles have no actual existence, and are only introduced into Feynman diagrams as a calculational fiction ? The first sentence of our virtual particle article says "In physics, a virtual particle is a particle that exists for a limited time and space ...", which certainly implies that they are not solely aids to calculation. Gandalf61 (talk) 09:33, 14 February 2008 (UTC)[reply]
It may help to imagine looking closely at something - like a TV, magazine picture, or pencil line. When you look closely, what looked like a solid clear-cut thing is actually made up of lots of dots, grey fuzz and is actually pretty fuzzy or patchy. Solid matter, like our bodies, is a bit like that. While at a distance it looks solid, if you could make a powqerful enough microscope, you'd see that it's made up of a cells, which are made up of molecules, made up of atoms... and at this level "matter" is tiny dots in a lot of space, held apart by forces such as the electromagnetic force and strong force etc (there are others), which prevent these points from coming too close and keep them separated, and stop them collapsing into almost nothing. It is in that sense that people are "mostly made up of empty space".
If you looked even closer you'd see the dots and the space itself are a sort of fuzzy haze of potentials and quantum theory entities that may-or-may-not exist and can-and-cannot be seen, which is a lot harder to describe... but by then you're into quantum theory.... FT2 (Talk | email) 17:42, 13 February 2008 (UTC)[reply]


February 13

how many delegates per state

how many delegates per state —Preceding unsigned comment added by Len70 (talkcontribs) 03:04, 13 February 2008 (UTC)[reply]

I think it depends on the population. That's why some get more than others. However, this question is probably better suited for the humanities reference desk Zrs 12 (talk) 03:20, 13 February 2008 (UTC)[reply]
It does seem a little off-topic here, but as long as it's come up, let me point out that it's entirely up to the political party. The parties are free to run their nomination process pretty much however they want, and you're free to vote for their nominee or not. Oh, I suppose there are some limits--say, the parties can't stipulate a test on who's allowed to vote in their primaries outside of the ones provided for by the state in question--but they are free to weight the votes from the various states however they want, up to and including completely ignoring the vote in some (or all) states.
I think the Democratic Party has a notoriously complicated formula that's based, not just on population, but also on the Democratic vote from the last general election. --Trovatore (talk) 03:35, 13 February 2008 (UTC)[reply]
For current delegate counts, see the Democratic Party (United States) presidential primaries, 2008 and Republican Party (United States) presidential primaries, 2008 articles. —Lowellian (reply) 00:07, 14 February 2008 (UTC)[reply]
Interesting question; is political science a science or an art? --hydnjo talk 03:40, 14 February 2008 (UTC)[reply]

Who discovered the peanut allergy?

Who discovered the peanut allergy? —Preceding unsigned comment added by Byrnesix (talkcontribs) 05:10, 13 February 2008 (UTC)[reply]

I do not have online access to the expensive databases of articles listed, but Google Scholar shows few mentions of "peanut allergy" before the 1980's when it became a widely published about term. Google News search shows no "peanut allergy" usage in news papers and such before 1962, when there was one story in a medical column by Dr. Burton H. Fern, M.D. (the Newark Advocate, March 19, 1962, p.4 and other papers) The medical columnist referred to peanut allergy with pretty much the modern description, and spoke as if it were already a well known phenomenon. The next press coverage I could find was in the New York Times, March 22, 1986. Edison (talk) 19:03, 13 February 2008 (UTC)[reply]
I searched medline and found references dating back to the '50's. That was as far back as that database goes. Delmlsfan (talk) 00:36, 14 February 2008 (UTC)[reply]
A quick JSTOR search turns up nothing useful on the topic. --98.217.18.109 (talk) 22:09, 17 February 2008 (UTC)[reply]

Ammonia and copper sulphate

wat happen wen ammonia is added to copper sulphate? —Preceding unsigned comment added by 202.141.29.35 (talk) 05:51, 13 February 2008 (UTC)[reply]

There are two things happening. But this is an easy Homework question! But the Cuprammonium method

for the production of rayon starts with this! --Stone (talk) 11:16, 13 February 2008 (UTC)[reply]

lowenstein-jensen media...?

why we keep lowenstein-jensen media in a glass bottle and not in plastic container ? please mail me the answer at [ email removed ] —Preceding unsigned comment added by 202.141.98.100 (talk) 09:00, 13 February 2008 (UTC)[reply]

Glass (q.v.) does not react with most chemicals, unlike most plastics.--Shantavira|feed me 11:00, 13 February 2008 (UTC)[reply]
Anyone fancy putting together an article on Löwenstein-Jensen media? --Tagishsimon (talk) 12:25, 13 February 2008 (UTC)[reply]
It's over at Lowenstein-Jensen medium. I'll make some redirects.--Shantavira|feed me 15:58, 13 February 2008 (UTC)[reply]
Maybe the malachite green will stain the plastics as it is a lipophilic dye (especially the leuco form). Сасусlе 04:20, 14 February 2008 (UTC)[reply]

Human Spleen

How much blood can a spleen hold? Weasly (talk) 11:47, 13 February 2008 (UTC)[reply]

In humans, not much. Our article gives a volume for the human spleen as 125 × 75 × 50 mm which would be 468 cubic centemetres. Assume that half of this is given over to spleen material, and you get something like 234cc, or 0.25 litres. Even that may be an over-estimate, as in humans the spleen is not a store for blood, but only for platelets. --Tagishsimon (talk) 12:22, 13 February 2008 (UTC)[reply]
That's interesting. In blood donation, donors typically donate around 450 ml of blood with no ill effects - in particular, no significant drop in blood pressure. Our article on bleeding says that blood loss of up to 15% of total volume typically causes no ill effects - for a total blood volume of 5 litres, this is 750 ml. If the spleen can only replace up to 250 ml then what mechanism does the body use to maintain normal blood pressure for these higher volumes of blood loss ? Gandalf61 (talk) 15:03, 13 February 2008 (UTC)[reply]
The following would be of interest to you baroreceptors, Renin, Angiotensin II, Aldosterone, epinephrine, and norepinephrine. Wisdom89 (T / C) 19:09, 13 February 2008 (UTC)[reply]
The body mostly makes up the missing volume with water squeezed out of other tissues. (One of the reasons why the nurses give you fruit juice before and/or after whole blood donations.) The circulating blood will be slightly more 'dilute', but still quite serviceable. Pressure is also maintained through vasoconstriction—the muscles in arterial walls contract, reducing the volume that the available blood must fill. At least, that's the short answer—our article on hypovolemia goes into quite a bit more detail. TenOfAllTrades(talk) 15:23, 13 February 2008 (UTC)[reply]
It sounds like the questioner may be referring to a function of some nonhuman spleens. The functions of the human spleen may not be entirely understood, but it is relatively well established that unlike some of its nonhuman counterparts, the human spleen does not serve as a storehouse of blood to be tapped in times of need. Some spleens (eg, those of horses) are supplied with smooth muscle that can squeeze the spleen to increase effective blood volume under times of physiologic stress. Human spleens have scant smooth muscle and are therefore unable to serve this function. --David Iberri (talk) 22:19, 16 February 2008 (UTC)[reply]

Passenger Zeppelins LZ120 and LZ121

I'm searching for design and operational statistics on the two small post-war Zeppelins seized as War Reparations. Design Statistics (OAL, BEAM, Displacement, Installed power, cruising speed, range/endurance, payload, normal crew size, passenger accommodations), any photos would be helpful. Any citations or links regarding in-service operations would also be of interest.

68.58.89.210 (talk) 11:50, 13 February 2008 (UTC)[reply]

Penicillin

Hi there, I am uncertain as to why penicillin inhibits the growth of prokaryotic gram positive bacteria which often cause infections in the body but has no effect against the human body's own eukaryotic cells. Is it something to do with organelles? If anyone could enlighten me on this subject I would much appreiciate it. —Preceding unsigned comment added by 172.214.45.199 (talk) 11:58, 13 February 2008 (UTC)[reply]

See penicillin#mechanism of action. Penicillin acts by inhibiting an enzyme that builds the bacterial cell wall. Humans don't have this enzyme, or indeed cell walls, so are unaffected. Algebraist 12:16, 13 February 2008 (UTC)[reply]
"or indeed cell walls"? Surely some mistake. In the absence of cell walls, what demarcates the boundary of a human cell? --Tagishsimon (talk) —Preceding comment was added at 12:24, 13 February 2008 (UTC)[reply]
The plasmalemma, or cell membrane. --Ouro (blah blah) 12:32, 13 February 2008 (UTC)[reply]
More generally, antibiotics are useful because they affect an enzyme in the pathogen more than they affect the corresponding enzyme (if any) in the host. (Ideally, they'd effect a metabolic pathway absent in the host, or one which can be bypassed in the host). As Ouro points out, one distinction between human cells and many bacteria is that human cells have no cell wall; penicillin and the cephalosporins work on this principle. Many antibiotics work because they are more effective at inhibiting bacterial mitochondria ribosomes (with 50S and 30S subunits) than they are on human mitochondria ribosomes (with 60S and 40S units). This may be what you are referring to about involvement of organelles, but that's not the mechanism for penicillin, details of which can be found here. - Nunh-huh 17:58, 13 February 2008 (UTC)[reply]
I believe you meant ribosomes, not mitochondria, as a slew of antibiotics sabotage bacterial protein synthesis. Wisdom89 (T / C) 19:05, 13 February 2008 (UTC)[reply]
Yes, indeed. - Nunh-huh 19:25, 13 February 2008 (UTC)[reply]

advantage of science

what is the advantage of science —Preceding unsigned comment added by 203.106.168.126 (talk) 12:33, 13 February 2008 (UTC)[reply]

Wikipedia, mp3 players and insanely wonderful DSLRs. Have a go at Philosophy of science and related articles. --Ouro (blah blah) 12:39, 13 February 2008 (UTC)[reply]
Knowledge. Have a look at Reliable Knowledge in that article too. AlmostReadytoFly (talk) 12:44, 13 February 2008 (UTC)[reply]
Umm, how about everything that we have? We would still be living in forests, living as foragers if it weren't for science. 64.236.121.129 (talk) 15:25, 13 February 2008 (UTC)[reply]
Not exactly. Civilisation is older than science as we know it. See History of Science. AlmostReadytoFly (talk) 15:50, 13 February 2008 (UTC)[reply]
If people live in buildings, then you need science, whether it is formal or not. If people use tools, then that's science. 64.236.121.129 (talk) 20:54, 14 February 2008 (UTC)[reply]
No, tool use is not science. (Even chimps can use tools.) And no, just living in buildings doesn't mean you need science. Please read science—it's not just the sheer presence of technology of some form, it's a systematized way of increasing knowledge, and not just any system either, it must have a combination of certain principles to be considered "science". The wheelbarrow required no science, no deep understanding of mechanics. By definition I would consider "informal science" to be oxymoronic—science is a system, it is not informal. --98.217.18.109 (talk) 22:13, 17 February 2008 (UTC)[reply]

And even if we did live as foragers, we'd still need science, like the branch of Botany, because we'd need to know which plants to eat, right? Science explains our very lives, and the world we live in. Before people began to use science, they used their religions to explain things that they didn't understand. (for instance, the ancient Egyptians believed that Ra was the sun god, and he made the sun rise and set). But, the ancient Greeks were some of the first to use science to explain why things happen the way they do. So, science is very important.--Princess Janay (talk) 15:56, 13 February 2008 (UTC)[reply]

There was actually a lot of discussion in the 1970s and 1980ss about whether or not "folk knowledge" and systems of folk knowledge constituted something approximating "science" (e.g. the work of anthropologist Robin Horton). Personally I lean against it, though it is unfashionable to do so—science is a particular form of knowledge production, and is not just a generic term for knowledge production. It is not the same thing as simple cataloging, or of simple unmethodological observation with willy-nilly confusion of correlation and causation. It is a methodology for producing, proving, and disproving knowledge. It works very well for questions concerning the natural world. It is above all disciplined. For the record, religion was not the only explanation available before science, and science did not get rid of religion. (And of course the early history of science is intimately tied with religious concerns.) --98.217.18.109 (talk) 22:13, 17 February 2008 (UTC)[reply]

What is the advantage of science? This is a question that used to be asked a lot in the 1930s but is almost never asked today. Why not? In part because science was credited as having won World War II—that is, because science became seen as intractably linked to technology, and technology became linked with economics and power. So that is perhaps one advantage to science, though many scientists would find it a disgusting, utilitarian one. The more idealistic would say knowledge is an advantage for its own sake, though the pessimistic would point out that knowledge is a two-way street: it confers advantages and disadvantages. Not everybody appreciated such knowledge either: there are many who wish that Darwin had become a priest as he had planned to be, and left speculation as to the purpose of human life to the theologians. (Note that one reason this question was asked so much in the 1930s is because faith in science as an improver of life was at quite a low between the wars, as it was blamed for much of the horror of World War I.) But perhaps we can find another way to look at this: science can produce knowledge that is highly reliable, and with highly reliable knowledge you can do a lot of things, whether they are technological in nature or philosophical in nature. It is almost always an advantage to have reliable knowledge over unreliable knowledge, even if it sometimes makes one unhappy. Our civilization owes much to the increased amount and increased ability to have reliable knowledge—you can, as we did for thousands of years and many still do today, live in a world where you don't know why people get sick, where you don't know when it is going to rain, when you make things based on intuition and the feel in your hands rather than any knowledge of deeper principles. You can get along just fine not knowing what an atom is and the principles of the chemical bond. Many people, even in advanced societies, know nothing of any real scientific knowledge. However they indirectly benefit from the fact that others do have such knowledge, and have developed good systems for testing knowledge. They benefit without knowing it: it is behind their great wealth, it is behind their great varieties of foods, it is behind the fact that they are very likely to make it out of childhood unscarred by disfiguring disease. Science, taken as a whole, is responsible for much of what we call improvement in the style of living. It is also responsible for many negative things as well—pollution on an unprecedented scale, weapons which can kill to an unimaginable degree, and, many have argued, a disrespect for the dignity of the human, and perhaps worst of all, television—but on the whole I think our lives are longer and more fulfilling than they would be without scientific advancement. Civilization would be significantly hindered without scientific advances; we would have less time for education, leisure, and growth. We would have less effective tools and medicine would be almost crippled. We would not, in fact, be here asking such questions today, on our space-age computers, scrawling messages at the speed of light across a great distance, across the world. Science has brought us all these things. It has transformed us from a petty people who see no further than the boundaries of our tribe into a people who can, if we choose to (though it is not always the best choice), set foot on the heavens themselves. --98.217.18.109 (talk) 22:25, 17 February 2008 (UTC)[reply]

Neural system viruses

I happened to catch a documentary on rabies, which stated it bypassed the immune system by travelling via the nervous system and CNS. As I understand it, herpes likewise avoids the immune system (in its dormant state) by hiding in the nervous system.

Is this common? Is there general medical research into the treatment of virii that share this trait? Is there a generic medical issue around "how to target virii in the nervous system" and how might such things be targetted? What are the relevant WP articles?

Not so much interested in specific illnesses. More the medical issues (knowledge, treatments, research, approaches, etc) related to infections that make use of the nervous system to bypass the usual immune functions.

Thanks. FT2 (Talk | email) 16:45, 13 February 2008 (UTC)[reply]

You've actually named the two major viruses that are disseminated by neural spread; others, like polio, can spread neurally, but primarily spread by hematogenous dissemination. There is indeed medical inquiry into the pathogenesis of such viruses. Some terms that may help you find pertinent articles: a neurotropic virus is one which can infect neural cells. A neuroinvasive virus is one that can spread to infect the central nervous system from a peripheral site (these are the viruses you're asking about). A neurovirulent virus is one which not only infects neural cells, but causes disease in those cells. Herpes simplex virus is a virus with low neuroinvasiveness and high neurovirulence. Mumps is caused by a virus with high neuroinvasiveness but low neurovirulence; rabies has both high neuroinvasiveness and high neurovirulence. Viral tropism tends to be multifactorial, so looking for general principles is less rewarding than looking at more specific issues. We seem to have articles on tropism and virulence, but I don't think they're of much use. - Nunh-huh 17:44, 13 February 2008 (UTC)[reply]
Now added: Neurotropic virus.
Can you check and see if it can be improved? Thanks :) FT2 (Talk | email) 18:25, 13 February 2008 (UTC)[reply]

ROCKET ENGINE

Hello


Why a same rocket engine have a greater Isp in vacuum than at sea level


Thank you... —Preceding unsigned comment added by 74.13.159.88 (talk) 16:46, 13 February 2008 (UTC)[reply]

At sea level, there's 14.7 psi of atmospheric pressure resisting the rocket's exhaust. In vacuum, no such resistance exists. — Lomn 18:28, 13 February 2008 (UTC)[reply]

Cryo-insulation gel?

I'm writing a book, and i need help. Say you wanted to keep an entire room frozen on the level of Cryonics. Is there a substance that would parallel a fictional cryo-insulation gel? Here7ic (talk) 18:25, 13 February 2008 (UTC)[reply]

You'd probably use vacuum and possibly some fairly esoteric kinds of heat pumps. Preserving a room at very cold temperatures would be difficult on earth without some active means of cooling, because of the tendency of heat to pass between bodies. You may need to specify what kind of temperature you're considering and what kind of room or environemnt, or other plot requirements -- liquid nitrogen is readily transported in bulk already, whereas temperatures close to absolute zero are more demanding. FT2 (Talk | email) 18:31, 13 February 2008 (UTC)[reply]
Let's use the Cryonics article's temperature of -196° C. It's in an underground facility beneath the Von Tobel community center in Las Vegas. The President and pertinent staff are being kept there after a nuclear holocaust, and a recovery team is on it's way down. However, as they make their way to the sub-level, they notice a leak of the "cryo-insulation gel", and eventually find that the preservation method failed and the president is dead, blah blah blah sci-fi horror masterpiece of mine. Here7ic (talk) 18:39, 13 February 2008 (UTC)[reply]
If you're looking for an insulating gel aerogel might be helpful. (Although it wouldn't leak, but it's sci-fi so you can take some liberties. Mad031683 (talk) 18:52, 13 February 2008 (UTC)[reply]
Maybe giant atomic rats found aerogel to be good for nest-building? --Sean 19:06, 13 February 2008 (UTC)[reply]
For liquid nitrogen etc good old expanded polystyrene is useful in the non-scifi world.. (think you want foam not gel)87.102.114.215 (talk) 21:57, 14 February 2008 (UTC)[reply]

Atomic bomb

Hi, I'm looking for an account of a university/college student who decided to work out how to build an atomic bomb, using only commonly-available knowledge as a foundation. Some things that comes to mind is that the author got disheveled as his work progressed and got to resemble a hobo, and that he couldn't figure out how the implosion mechanism should work. In the end he phoned DuPont (the manufacturer of the relevant part) and tried asking about their products: "I heard my professor say that your so-and-so product will work like this-and-this in such-and-such circumstances." The person who answered the phone (unknowingly, I guess) replied with hints. The author handed in his report on how to build an atomic bomb and there was talk at campus on whether his report should be classified or not.

I know this isn't exactly a scientific question, but who was that guy and where can I read his account again? Any hints will be much appreciated. :) --Kjoonlee 19:56, 13 February 2008 (UTC)[reply]

Could this be David Hahn, the radioactive Boy Scout? He was trying to build a nuclear reactor, though, not a bomb. David Dobson (nuclear physicist) and Bob Selden were nuclear physicists who decided to see if it were possible to build a bomb (but were content merely to describe the plans, rather than actually build the thing). They obtained crucial information about their implosion mechanism from papers published by President Dwight Eisenhower's "Atoms for Peace" program. - Nunh-huh 20:02, 13 February 2008 (UTC)[reply]
Also, they did this as part of a Pentagon project [9] rather then something they decided to do out of the blue Nil Einne (talk) 20:34, 13 February 2008 (UTC)[reply]
The author had decided to do it because he was taking a lecture on nuclear physics, and did it for his final report. The professor agreed to be an adviser, but treated the author's mistakes the same way he treated his accomplishments, without comment. --Kjoonlee 20:54, 13 February 2008 (UTC)[reply]
I might recall reading the same article as you have. Republished in Reader's Digest, mid-eighties or thereabouts. I'll look around.—eric 21:13, 13 February 2008 (UTC)[reply]
No luck, but i'm fairly certain that this was a hydrogen bomb design, and might have been related to the misclassification of UCRL-4725.—eric 22:13, 13 February 2008 (UTC)[reply]
You might find United States v. The Progressive relevant/interesting. --Sean 00:17, 14 February 2008 (UTC)[reply]
Also note that Howard Morland, the guy who caused that whole case, is now a contributor on Wikipedia: User:HowardMorland. --98.217.18.109 (talk)


I can also recollect reading this article a long long time ago and think it was in the Reader's Digest and that it was the building of an atomic bomb. (It is possible that the title was "The Atomic Bomb Kid" but I am certainly not sure if this is the correct title.) The University student worked at his paper day and night and only just managed to hand it in at the last moment. After having not heard from his Professor, he thought that he had failed on this paper, but his Professor then informed him that it had been classified. Simonschaim (talk) 09:00, 14 February 2008 (UTC)[reply]

The Manhattan Project is an interesting fictionalization of such an attempt. But it presumes access to plutonium.
Atlant (talk) 14:16, 14 February 2008 (UTC)[reply]
Some relevant articles: John Aristotle Phillips (who is probably who you are referring to—"The A-Bomb Kid", though there are actually at least half a dozen of the "some college kid make a bomb design based on public information" stories out there, most are not that interesting, because the all the basic concepts were declassified by the early 1950s, and it's just a matter of knowing a few nuclear constants to come up with a very, very rough idea of how to make a very, very crude weapon). Note that "designing" something like that is about as close to making it as an encyclopedia's drawing of an airplane is to being able to construct a 747. Also, the Nth Country Experiment established pretty well by the mid-1960s that it was not hard to "design" a bomb based on public domain information. --140.247.11.3 (talk) 00:46, 15 February 2008 (UTC)[reply]

I have another Physics Magazine,Question For You and It`s not a Homework Question

If the mass of an empty balloon the same as the mass of that same balloon when It is filled with air. Why is the balloon attached to the string.What effect does that have on the Force. Does the balloon change from one trial to the next.Does this affect your results. What is making the balloon move.Describe what is happening and how this affects the motion of the balloon. Calculate the Force the balloon applies to move across the room.Is the force the same for all three trials. How are the motion and Forces acting on the vertical balloon rocket different from the string guided balloon in this activity. —Preceding unsigned comment added by Yeats30 (talkcontribs) 23:08, 13 February 2008 (UTC)[reply]

What physics magazine is this from? I can answer one of your homework physics magazine questions for you. "What effect does that have on the Force." It causes a great disturbance in it. Mad031683 (talk) 00:08, 14 February 2008 (UTC)[reply]
How long is the piece of string? Boomshanka (talk) 00:24, 14 February 2008 (UTC)[reply]
Before more people ask, this is the same user who claims to be getting questions out of a physics magazine when he should be learning basic grammar and typing skills.
Answering the start of the question - adding air to a balloon adds mass. It is impossible to add air and not add mass. So, you appear to stating that the mass of the balloon is the same, discounting anything contained inside the balloon. That is a very strange way to refer to it since normal people call the balloon and the contents inside of the balloon the "balloon".
Why is it attached to a string? Because whoever owns it likes strings. You could let it roll across the floor without a string if you like. You could tie a ribbon to it. You could shove it down your underpants. Whatever makes you happy while you read your imaginary magazines.
What is the affect on the string force? There's not enough room here to get into string theory.
Balloons change all the time - especially when going through trials. They get stretched out when you blow them up. Sometimes they get marked on. Sometimes they pop and become rather useless balloons. When they pop, the results of using it another test become rather speculative. You can only imagine what the balloon would do if it could still hold air in nice containment.
What makes balloons move? A bad housing market plagued by foreclosures has caused many an experimenter and his balloons to move. Hopefully the government will help out soon. Won't somebody think of the poor balloons?
Vertical balloon rocket? Is that part of the new Mars Mission? I remember reading that it has to be a vertical balloon rocket because those horizontal balloon rockets did a terrible job of making it into orbit. Hopefully that helps. -- kainaw 00:42, 14 February 2008 (UTC)[reply]
Gee, you sure seem to be giving us lots of questions from 'physics magazines' and 'algebra magazines'. The rest of us at the Desk are awfully curious about how you're finding all of these—could you start providing references (journal title, volume, page number, date) for your sources? Seems a shame that the original question creators aren't getting their fair credit. TenOfAllTrades(talk) 00:49, 14 February 2008 (UTC)[reply]

I may be wrong, but this sounds like a homework question to me... What do you all think? Zrs 12 (talk) 01:24, 14 February 2008 (UTC)[reply]

It doesn't just sound like homework, it sounds like the homework is to write up an experiment which was performed in class. -mattbuck (Talk) 01:34, 14 February 2008 (UTC)[reply]

I have all the answers you need below:

  • The balloon is attached to the string because otherwise the quantum flux will expand, causing the balloon to explode
  • The effect on the force is and overall increase in the number of midi-chlorians
  • Whether or not the balloon changes is irrelevant because all the balloons are free balloons.
  • The Rial Krowemoh effect is making the balloons move
  • The Rial Krowemoh effect means the quantum flux on the balloon results in a level of midi-chlorians less then the atmosphere. Therefore the balloon moves in a 97.298 degree angle on the vertical.
  • Using the Resol Enizagam Scisyhp equation we find that the force on the balloon is 59 midi-chlorians for all three trials. (97.298 x 0.495^0.56 + lg 5.93)
  • The force acting on the vertical balloon tends towards the Idej whereas the force on the string tends towards the Htis.

I guarantee you that these answers are 100% correct and you will get an A+ for them. I suggest you copy them out exactly, don't worry too much if you don't understand anything. Your teacher will and if he or she doesn't he or she will surely ask a Physics Professor who will be able to explain them. They are beyond what is normal in secondary school physics level but there's no harm in being advanced for your age Nil Einne (talk) 06:48, 14 February 2008 (UTC)[reply]

Sorry to rain on your parade Nil Einne, but it's the Rail Krowemoh effect, not Rial. -mattbuck (Talk) 12:21, 14 February 2008 (UTC)[reply]
Indeed. Rail Krowemoh was a very notable physicist, and it's important to get his name right. The Rail gun (and not Rial gun) was named after him because he discovered how electricity and magnetic fields interact. —Lowellian (reply) 12:55, 14 February 2008 (UTC)[reply]
Really? In Russian it's always called a Krowemoh Gun (Нонсенс Пу́шка) after Rail Krowemoh (Рельсы Нонсенс). So it's a popular misconception, one not shared by Russian speakers, that it has anything to do with "rails". -- Finlay McWalter | Talk 13:01, 14 February 2008 (UTC)[reply]
You're completely right. Luckily you saved our friend from nearly losing his/her A+! Nil Einne (talk) 19:11, 15 February 2008 (UTC)[reply]

This reminds me of the old Siamese National Anthem. The first line was "Oh Wattana Siam". You need to say it out loud to get the full impact. --Dweller (talk) 13:34, 14 February 2008 (UTC)[reply]

Homework? I always thought he was asking us to solve physics magazine problems so he can win a prize of some sort (a beer can sleeve with a logo, no doubt). But now it seems that he has been playing some of us for fools. He probably doesn't even drink beer! 206.252.74.48 (talk) 14:13, 14 February 2008 (UTC)[reply]

Length/Width measurement

What is the name of something that can be used to measure the width of something, such as a ball. It has two plates which the thing you want to measure is put in between, and they can be pushed together or spread apart until both plates touch the object that is beaing measured, and the width of the object can be read off it based on where a pointer is. What is the name is the measurement device? I think it is a 'something calorimeter', but on wikipedia any calorimeters or colorimiters are to do with colour or light. —Preceding unsigned comment added by 86.129.217.188 (talk) 23:09, 13 February 2008 (UTC)[reply]

vernier caliper.—eric 23:16, 13 February 2008 (UTC)[reply]
Well, since the questioner mentioned a "pointer", probably a "dial caliper" rather than a vernier caliper, but okay ;-). Micrometer is another possibility.
Atlant (talk) 14:10, 14 February 2008 (UTC)[reply]
ok thanks —Preceding unsigned comment added by 86.129.217.32 (talk) 20:49, 14 February 2008 (UTC)[reply]
and calorimeters are to do with heat --ColinFine (talk) 21:06, 17 February 2008 (UTC)[reply]

February 14

Lunar Eclipse

In science class, we were discussing the upcoming lunar eclipse and an interesting question came up. If one were on the moon during a lunar eclipse, what would happen? Would the moon be in a shadow? Or would it be brighter? Admittedly, this was assigned as an extra credit question, but I have tried searching online and have no idea. I had the idea that it would be darker on the moon, but I could find no information online to corroborate that. Oh, and please include sources, if possible! Thanks, FruitMart07 (talk) 01:44, 14 February 2008 (UTC)[reply]

You're thinking along the right lines, and if you continue you'll appreciate that it's going to look something like a solar eclipse seen from the earth. Both are eclipses of the sun. The difference is that the earth has a bigger diameter than the moon, and so one would not see a full corona, but might witness Baily's beads. And yes, it will be dark on the moon, in the same way as it is dark on the parts of the earth across which the solar eclipse passes. --Tagishsimon (talk) 02:04, 14 February 2008 (UTC)[reply]
But there's another big difference. Light passing through an atmosphere gets refracted and scattered. (The Earth's atmosphere scatters blue light more, so when you look through a great amount of it, like at the setting sun, you see red because that's what didn't scatter so much.) Now, in a solar eclipse, the light does not enter the Earth's atmosphere until it has gotten nearly all the way to Earth, so it doesn't have time to scatter much. The result is that in the umbra the light is almost totally blocked and it's almost like nighttime.
In a lunar eclipse, on the other hand, the light hits the Earth's atmosphere first and starts spreading out due to scattering, with the red light spreading least. It then travels roughly a quarter-million miles, still spreading out. The result is that some light scatters enough to fill the umbra, which is why during a lunar eclipse you can still see the Moon and it looks reddish. If the Earth isn't blocking all the light because some gets through the atmosphere, what would it look like if you looked at the Earth when it was in front of the Sun? --Anonymous, 02:53 UTC, February 14, 2008.
If you were on the Moon, watching the Earth eclipse the Sun, during totality, the Earth would probably appear as a black disk with a thin reddish ring around it. Other things to look for are stars and city lights. I don't know about the relative brightnesses of the ring, the lights, and the stars, so one or more might be too dim to be seen. --Carnildo (talk) 21:50, 14 February 2008 (UTC)[reply]
Thank you guys so much!! Everyone says this class is really hard, so I'm trying to rake up a lot of extra credit points... I concluded that there would be less light on the moon because the earth would be blocking much of the sunlight. It's a straight-forward conclusion, really, but I just needed to think it through. Thanks for all of the help! (Oops! Forgot to log in!!) FruitMart07 (talk) 00:07, 15 February 2008 (UTC)[reply]

Mercury Perihelion

OK, I've spent the past 2 hours reading and rereading everything on Mercury's orbit around the sun, and how it relates to relativity. So, I think I get it, but I would appreciate a nice laymen's explanation that details the major equations. Also, any links to failed theories that attempted to explain Mercury away would be appreciated. Thanks! Zidel333 (talk) 04:09, 14 February 2008 (UTC)[reply]

I don't know if this is what you're looking for, but I think the GR perihelion shift can be correctly understood in terms of the spatial curvature of the Schwarzschild metric, specifically the fact that the circumference of a circle centered at the sun is less than 2π times its "effective" radius. When you've completed what would be one Newtonian orbit, you've actually gone a bit farther than a full circle, so the orbit precesses forward. Let me work this out numerically and see what happens. We're ignoring t, and also θ since everything is in a plane, so we're left with . As you probably know, this can be embedded in with cylindrical coordinates by taking . Supposing Mercury's orbit is roughly circular with a radius of R (which is not even close to true, but never mind), the only part of this embedded geometry that matters is the annular region at radius R. To first order this looks like an annular region of a cone with a slope given by . If you slit open a cone along a line segment from the apex to the base, you can flatten it into a disc with a slice missing. The angle of the missing slice, as a fraction of a whole circle, is our predicted perihelion shift per orbit. For a cone slope of n, that fraction is , or in this case about . Plugging in 1.5 km for m and Mercury's semi-major axis for R, that's about one part in 40,000,000, which is pretty close to the right answer of one part in 12,500,000. There might be a missing factor of two in there from the temporal curvature (the way there is with the bending of starlight) which would bring the prediction to one part in 20,000,000. Of course, Einstein didn't do anything like this; he used some form of linearized general relativity and worked out equations of motion, but I've forgotten the details and it's not as interesting.
I don't know anything about the history of other attempts to explain the anomalous precession, but I don't think it's hard to come up with modifications of Newtonian gravity which give the right result. What was exciting about the GR calculation was that it produced the right value without any new adjustable parameters. -- BenRG (talk) 21:47, 14 February 2008 (UTC)[reply]

Anthropology?

This is probably an anthropology question – if an Australian aboriginal man wears a red head band or a white headband in a ceremony, what particular role or status do these colours signify? Someone told me what the white one meant, but I forget (add: maybe it meant "elder") and I can't find it anywhere on google etc. Julia Rossi (talk) 06:37, 14 February 2008 (UTC)[reply]

Come on Jack, we're all waiting for you on this one, surely you've got some bonzer mates that can put you right. Richard Avery (talk) 13:35, 14 February 2008 (UTC)[reply]
To my shame, I have no indigenous friends. What I would say, though, is that the Australian indigenous peoples, while speaking with one voice on many matters, are ethnologically and culturally as diverse as the Incas, the Lapps and the Masai. It's not true, for example, that all or even the majority of tribes used boomerangs and didgeridoos, despite these having become symbols for indigenous people as a whole. Two tribes chosen at random would be much more likely than not to have spoken mutually unintelligible languages and to have widely different cultural practices. For a continent the size of Australia, it's not hard to see why this would be the case. The wearing of headbands and the significance of the colours is not something I can comment meaningfully on. Sorry. -- JackofOz (talk) 00:36, 15 February 2008 (UTC)[reply]

Sounds like he joined the Bondi Lifesaving Club Myles325a (talk) 02:13, 15 February 2008 (UTC)[reply]

Don't feel bad, Jack, Brendan Nelson hasn't got any indigenous friends either. Glad to see yu and Myles chimed in even though none of us are the wiser. Carmonozicarmoncarmon etc. ; )) Julia Rossi (talk) 08:32, 15 February 2008 (UTC)[reply]
BTW if it helps, it's a Northern Territory tribe like the Arrente (or pr: Arunta), so bit by bit maybe someone will know something. Julia Rossi (talk) 08:35, 15 February 2008 (UTC)[reply]

Colour

I can't remember whether an object that absorbs red light is red or an object that absorbs other light, but bounces back red is red. Which is it?KarateKid101 (talk) 09:51, 14 February 2008 (UTC)[reply]

Think about it carefully. If an object absorbs all light equally, it is black, because you don't see anything reflected. If an object reflects most light equally it is white (in white light), because you see all the light reflected from it. If an object absorbs red light you're not going to see red, but the other colours it does not absorb. For an object to be red, it needs to absorb most of the light that is not red hence what you see is the red light that it does not absorb but reflects. Nil Einne (talk) 10:05, 14 February 2008 (UTC)[reply]

Repeting things to help retrograde amnesia?

The article on retrograde amnesia is pretty clear, but I wonder about one thing. It says trying to force someone to remember can be too stressful, but what about repeating a past event without telling the person you are. (I asked in the discussion section, but this venue may be more appropriate.) I know TV often exaggerates things, but will repetition of a specific event help more than just surrounding with familiar things? For instance, in The Jeffersons, George repeated his first date with Louise - a comical disaster - to help her get her memory back. Other sitcoms have used the "familar things" approach, like an episode of Diff'rent Strokes (where it came back gradually, like normal) and the finale of Full House. But in the former, it could be aruged it was repeating a familiar event that triggered Mr. Drummond's memory of Arnold and Willis (threatening to ground them if they ran away), and in the latter, the Full House Chronology [[10]] describes in a footnote that it's implied an accidental repeating of an earlier experienced situation helped Michelle recover her memory. (Being in a strange place and looking for older sister Stephanie for comfort, like Michelle's first day of Kindergarten.) Or, would the "Full House" example be more an example of just using "familr things" to jog the memory, anyway, since it does seem to imply Michelle growing more comfortable with Stephanie first?Somebody or his brother (talk) 13:35, 14 February 2008 (UTC)[reply]

Matter to energy conversion using black holes

On the Mass–energy equivalence article, it says you can use black holes to convert matter to energy. What is the efficiency of this conversion? 64.236.121.129 (talk) 16:50, 14 February 2008 (UTC)[reply]

It's 100% efficient (which is why it's under the "perfect conversion" section!) The only problem is that it takes a while. (EhJJ)TALK 18:15, 14 February 2008 (UTC)[reply]
Do you know this, or are you just assuming it's 100%? I seem to recall that another book I saw in the book store say that the conversion is 60 or 70% efficient. Can anyone confirm this? Also what does the energy manifest as? Just heat? Any neutrinos? 64.236.121.129 (talk) 18:20, 14 February 2008 (UTC)[reply]
You can toss anything you like into a black hole and the Hawking radiation will consist largely of electromagnetic radiation, but a hot enough black hole will also emit neutrinos, which are effectively lost energy, and possibly heavier particles as well (which could perhaps be tossed in again). And the electromagnetic radiation can't be converted into useful work with 100% efficiency in practice (I love the image of a miniature black hole being used to boil water which drives a steam turbine). So it does seem pretty dubious to claim that this is a 100% efficient conversion. I'm afraid I don't know the ratios of different particles emitted as a function of temperature.
Also, I should point out that theoretical black holes are able to attain such high efficiencies only by violating most conservation laws. The reason you can annihilate matter with antimatter producing "pure energy" (i.e. photons) is that the conserved quantum numbers attached to a particle and its antiparticle are negatives of each other, and the quantum numbers attached to a photon are all zero, so both sides of the interaction add to zero and the conservation laws are satisfied (except for spin and energy-momentum, which have to be considered separately). With a Hawking black hole you can turn ordinary baryonic matter into photons even though this grossly violates the conservation of baryon and lepton number. I'm not sure what would happen if real black holes actually respect these conservation laws, but you might get out pretty much what you put in, which would make the whole thing rather pointless. On the other hand the bill never comes due if the black hole never evaporates, and baryon and lepton number aren't exactly conserved in general even outside black holes, so maybe this is a non-issue. -- BenRG (talk) 19:39, 14 February 2008 (UTC)[reply]
I should note that matter + antimatter doesn't entirely result in photons. As much as 50% results in useless neutrinos. —Preceding unsigned comment added by 64.236.121.129 (talk) 21:04, 14 February 2008 (UTC)[reply]

The Position and Speed of Matter

I'm not a particle physicist but there is something I cannot get my head around. Documentaries I have seen state that it is not possible to plot the position and speed of a sub-atomic particle at the same time. But if you don't know where something is, how can you measure the speed it is travelling at. If I want to know how fast my cat is running up the stairs then surely I need to where the cat is in the first place and where it ends up with the time taken inbetween! —Preceding unsigned comment added by 82.21.54.17 (talk) 17:03, 14 February 2008 (UTC)[reply]

Simple explanation is that to measure the position of an object you have to interact with it in some way, and that interaction changes the object's momentum. Now measuring the speed and position of your cat by, say, bouncing a light ray off it is not going to change its momentum very much - but that is because your cat is a macroscopic object. No matter how carefully you try to make a measurement, or how sensitive your equipment is, there is an absolute lower limit on the precision with which you can simultaneously measure an object's position and momentum. This precision limit doesn't make a significant difference to your cat, but it is important for sub-atomic particles because of their small size and mass - see uncertainty principle for more details. (Yes, yes, I know this is an enormously simplified explanation, which glosses over several factors, but it will do for now) Gandalf61 (talk) 17:42, 14 February 2008 (UTC)[reply]
I think you have oversimplified what the documentary (should have) said. I have no way of knowing what they actually said. A better phrasing might be: "not possible to plot the position and speed of a sub-atomic particle to an arbitrary degree of precision at the same time." Gandalf61's comment follows from here. JohnAspinall (talk) 19:23, 14 February 2008 (UTC)[reply]
Gandalf, the uncertainty principle is distinct from the observer effect. It is possible to create two particles so finding the velocity of one will give you the velocity of the other and finding the position of one will give you the position of the other (I'm not certain how it works. It might be that they have the same position and velocity, but you make one after the other.) You can then measure the position of one and the velocity of the other. Anon, there are ways of finding the velocity of something without using two measurements of its position, such as the Doppler effect on a photon reflected from it. — Daniel 00:51, 15 February 2008 (UTC)[reply]
As I said, I do know that my explanation is enormously simplified, and glosses over several factors, one of which is the subtle distinction between uncertainty principle and observer effect. Yes, we could get into the whole topic of quantum entanglement. But I gave an explanation that I thought was appropriate for the questioner's level of knowledge. If you think you can give a better answer to the original question, then please have at it. Gandalf61 (talk) 10:34, 15 February 2008 (UTC)[reply]
This is a good question. I don't know how the speeds of quantum objects are measured in practice, actually. But it's not so strange for something to have a well-defined speed without having a well-defined position. Wind does, for example. The uncertainty principle shows up in the theory of classical waves, so there's a closer analogy there. The speed of a quantum particle corresponds to the frequency of a wave, and the location of a particle to, well, the location of a wave (in space or in time). You might know that you can tune one instrument against another by listening for beats between the frequencies. As the frequencies get closer the time between beats gets longer, which lets you know that you're tuning in the right direction. But to be sure that the frequencies were exactly the same you would have to wait forever. The frequency difference and beat time are related by an equation that looks like the uncertainty principle. This isn't a weakness of this tuning technique, it's a universal property of waves. A wave doesn't actually have a well-defined frequency unless it has a large extent in space or time. -- BenRG (talk) 13:43, 16 February 2008 (UTC)[reply]

War

Assuming that Nuclear Weapons were not used, who would be likely to win in the event of a war between the USA and Europe? —Preceding unsigned comment added by 195.188.208.251 (talk) 17:04, 14 February 2008 (UTC)[reply]

According to Military budget of the United States, the US spends 47% of the world military budget, so dollar for dollar, the US could take on approximately the rest of the world, let alone Europe (taking the premise of a unified Europe). Also, US military technology is generally recognized to be somewhat superior to that of other countries, so I'd speculate that each US dollar spent on defense is more potent then a European dollar. Europeans do outnumber Americans by a bit more than 2:1, but the population of Europe is also older. Overall, I'd bet on the USA. --Bmk (talk) 17:33, 14 February 2008 (UTC)[reply]
The Navy alone would be crushingly decisive. Our article claims that "The United States Navy is the largest in the world with a tonnage greater than that of the next 17 largest combined" and "the world's largest carrier fleet, with 11 carriers in service". Read the article Aircraft carriers and ponder what 11 of them could do. They'd get hurt; the Brits, French, and Swedes have good air forces, and the British subs would be troublesome. But not for long. Funny, isn't it, that throughout history madmen have been dreaming of conquering the world, and now sane men actually could but won't. --Milkbreath (talk) 17:52, 14 February 2008 (UTC)[reply]
Don't forget the individual units themselves. The M1 Abrams, the AH-64, and the A-10 are all pretty frightening. Let's also recall that the USA has remotely piloted stealth bombers and those fancy artillary units that makes safe zones that troops can find with GPS. Thank God they don't go and attack random countries. Oh wait, never mind. 206.252.74.48 (talk) 17:59, 14 February 2008 (UTC)[reply]
Interesting side note: Although the USA spends the most on its military, it is only third in military spending per capita (after Israel and Singapore). See this source. --Bmk (talk) 18:11, 14 February 2008 (UTC)[reply]
Also, the US has military bases in Germany, Italy, The Netherlands, Portugal, Spain, and Turkey (Well, alot of those are NATO bases, but there's American forces there) which gives them a strategic advantage. NATO bases would cause interesting problems, although I suppose that by the time it came to war NATO will have been dissolved. Also, we don't attack random countries, only countries with alot of oil, get it right. Mad031683 (talk) 18:21, 14 February 2008 (UTC)[reply]
Hi. Oh dear. Peak oil is going to be a big problem. Why is the US spending a trillion dollars fighting partially for oil that will proably run out in a few decades?!? Thanks. ~AH1(TCU) 18:24, 14 February 2008 (UTC)[reply]
Ok, we attack countries that have oil at random. The paradox is the the vehicles used to invade the countries are mostly fueled by oil products - so soon we will fight for oil just to get oil in order to fight...for oil. 206.252.74.48 (talk) 19:28, 14 February 2008 (UTC)[reply]

Is this science? (rhetorical)87.102.114.215 (talk) 18:25, 14 February 2008 (UTC)[reply]

I don't think it is. It's also hypothetical. Not really appropriate.64.236.121.129 (talk) 18:29, 14 February 2008 (UTC)[reply]
Also it answers itself, if the supply is running out, its more worth fighting over it. Mad031683 (talk) 18:42, 14 February 2008 (UTC)[reply]

There are numerous forums where you can have this discussion to your hearts content. To get started go to YouTube and search for videos of the USS Enterprise, or M1A1 Abrahams, or whatever piece of military hardware helps you most. The enter the comments below the video and leave a message such as "USA ROKS, frenxch pussies can kiss my ass" or something similar.. Before long you will have lots of well wishers inviting you to their websites, as well as many heart warming replies from nice people around the world. Best of luck.87.102.114.215 (talk) 18:56, 14 February 2008 (UTC)[reply]

Ho ho - my web search turns up this "LOL i remember russian soldiers are forced to become prostitute by their officers according to UN LOL! " - given this information I think USA will win unless it catches AIDS from mexican woman first. Once again in peace and love to all of you.87.102.114.215 (talk) 19:11, 14 February 2008 (UTC)[reply]

I don't think the Europeans would keep those low military budgets if their existence was seriously challenged by a foreign power such as the USA. Whenever a country faces conflict, the portion of its GDP dedicated to military sky-rockets. Nowadays the US army is far stronger than those of the European Union countries combined, but the EU has a larger population and a slightly larger economy. Furthermore, Russia alone (if we let it join the European team) has a large army, something to keep in mind. To sum up, I guess that if the war began tomorrow, it would be a one-sided win for the US, but if we let the EU have 10 years for preparing itself, I wouldn't be that sure. --Taraborn (talk) 20:26, 14 February 2008 (UTC)[reply]

I don't think russia would make a big diference - once recruit was so badly beaten by his comrades he had to have his penis and testicles amputated.87.102.114.215 (talk) 21:11, 14 February 2008 (UTC)[reply]
Although if war began tomorrow the USA might have more difficulty extricating itself from the Middle East to deploy them elsewhere than various European powers. AlmostReadytoFly (talk) 23:38, 14 February 2008 (UTC)[reply]
ok that's enough isn't it. TRY SOMEWHERE ELSE.87.102.114.215 (talk) 00:35, 15 February 2008 (UTC)[reply]

Well, all the pointers say the US should win, but then Vietnam should have been a cake-walk, and you got your butt tanned big time there, and now you are getting second helpings in Iraq because the first lesson didn't sink in, so it just goes to show that "the bigger they are, the harder they fall". I think Monaco might be a safe bet. Myles325a (talk) 02:18, 15 February 2008 (UTC)[reply]

There are no winners in war...except for the ones who don't lose
Draw. The U.S. navy is far too strong for the Europeans to reach North America, but on the other hand, without nuclear weapons, the American army isn't big enough to conquer Europe, never mind the supply problems. After all, it's having trouble handling a single, mid-sized country. Clarityfiend (talk) 06:51, 15 February 2008 (UTC)[reply]

The Europeans kept low military budgets in the 1930s when they were menaced by Nazi Germany, such that the Germans were able to take over Czechoslovakia and other territory without any military oppisition from the powers such as Britain and France which had pledged to support its territorial integrity. What reason is there to believe they would arm themselves now to defend adequately against some threat from the U.S.? But as for invading, conquering and occupying Europe, I just can't see it happening, when after five years of occupation of Iraq the U.S. still finds it necessary to engage in large scale aerial bombing of Iraqi cities and cannot drive down a road without large IEDs being detonated. Edison (talk) 17:06, 15 February 2008 (UTC)[reply]

Ok, but the Iraqis didn't do anything to us. The average American wouldn't have known an Iraqi if he was waterboarding him back in 1980. Whereas the Europeans have been going out of their way to get on our last freakin' nerve ever since WWII, especially the French. I mean, look at it. The French hairdressers are on strike! How do you think that makes a red-blooded son of Uncle Sam feel? And that on top of pulling out of NATO. You just have to piss America off enough to swing public opinion in favor of the required total war mentality, and the population imbalance can be redressed pronto. Sorry, did I say all that out loud? --Milkbreath (talk) 17:17, 15 February 2008 (UTC)[reply]
You mean, all across France, with the hairdressers on strike, even as we speak, people are forced to tolerate ungroomed armpits? Ah, the humanity. - Zotz (talk) 00:44, 16 February 2008 (UTC)[reply]
Why have so many red-blooded sons of Uncle Sam forgotten that the USA owes its independence to France? Isn't the American Revolution taught in history classes in the US anymore? Pfly (talk) 07:05, 16 February 2008 (UTC)[reply]
Isn't the chronological order of the American and French Revolution taught elsewhere? It's not that Americans aren't cognizant of the help France provided by using America as a proxy in their conflict with England, it's that they've also learned the lessons in gratitude taught them by the French nation in the years since the last World War. - Zotz (talk) 07:15, 16 February 2008 (UTC)[reply]

Igniting gunpowder (smokeless powder)

If you were hit by free electrons (lightning bolt or something), while holding a rifle like an M-16. Would the gunpowder in the bullet shells ignite? 64.236.121.129 (talk) 18:28, 14 February 2008 (UTC)[reply]

Easily possible -- a lightning strike could certainly ignite gunpowder. Mythbusters did a segment on using a .22 cartridge as a fuse replacement and got it to fire (don't recall the current/voltage used, but far less energy than a lightning strike). It's hard to say anything definitively, though, as lightning does weird stuff. Would current pass through the rifle? Even if so, would the rifle's construction serve as a sufficient lightning rod? It'll likely vary from strike to strike. I'd put money on a strike on the rifle magazine causing the bullet to fire, though. — Lomn 18:40, 14 February 2008 (UTC)[reply]
Of note: M-16 shells do not use gunpowder. Also, the butt, handle, and barrel guards are plastic, not metal. The "handle" on top is not supposed to be used a handle. Still, it is possible for electricity to travel along the metal and, for some unknown reason, want to travel into the bolt mechanism and then, for some unknown reason, want to travel to the shell and ignite the propellant. It would make more sense for the electrons to travel along the outside of the rifle to the human and then to ground. -- kainaw 20:15, 14 February 2008 (UTC)[reply]
M-16 shells use smokeless powder I believe, which I mentioned in my topic. 64.236.121.129 (talk) 20:50, 14 February 2008 (UTC)[reply]
It appears that M-16s use WC844 ball powder, which is discussed as being distinct from smokeless powder in ways I don't really understand. I'd imagine it's a similar outcome in any event. — Lomn 21:25, 14 February 2008 (UTC)[reply]
What? I think it's just one of the (many) varieties of smokeless powder. Where do you see a suggestion otherwise? Friday (talk) 21:30, 14 February 2008 (UTC)[reply]
I was using this as a reference, but like I said, I didn't grok the details. It may well be a simple distinction in subtypes of smokeless powder. My confusion probably stems from reading all this with a "gunpowder=smokeless powder" mental filter, thus seeing Kainaw's comment as "M-16s don't use smokeless powder", which led to me trying to find out what they did use. Short version: my bad. — Lomn 22:01, 14 February 2008 (UTC)[reply]
It would be pretty surprising if any explosive used to propel bullets could not be ignited from an electrical spark. But the cartridge generally provides a metal shell completely surrounding the smokeless powder, gunpowder, etc. Certainly a full-fledged lightning discharge could ignite the rifle, the rifleman, and the cartridge, just from the high temperature. I have seen fairly large pieces of metal vaporized by lightning. Edison (talk) 16:59, 15 February 2008 (UTC)[reply]

Microwave radiation health effects - alternative questions

When possible health effects of microwave radiation is considered they usually look at the average energy concentration involved. What bothers me is that there is usually no mention that artificial sources of microwaves typically have lots of energy in a tiny band (relative to the frequency, not absolute). And microwave wavelengths are typically around the order of the size of humans, human organs or a human scull.

When during exposure a person does not move relative to the source of radiation such as when speaking on a cellphone held tightly against the ear, won't resonances occur (analogous to what happens when you sing in the shower) that push energy concentration in certain spots way above the average? If yes, is this taken into account with most studies and can we place a ceiling on the extent to which this effect can cause potentially dangerous hot spots?

Secondly, and perhaps this is still a controversial subject, I read about a possible HIV treatment that uses the resonant frequencies of the virus as a way to "shake them to death". I hope it's OK that I'm not mentioning the source because I really can't remember. They say the frequency is calculated at around 60GHz and that the proposed treatment won't affect normal cells which have a much lower resonant frequency.

Does anyone know in what sense this "resonance" is meant, because in order for 60GHz to resonate at that small scale I assume that there has to be some sort of wave that propagates much slower than the speed of light. And then, using this logic, roughly at what frequency DOES normal cells resonate and what would happen if they were exposed to EM radiation at this frequency? (What I'm asking is slightly different from the proposed treatment mentioned which instead proposes using a laser pulsing at that frequency).

41.241.187.40 (talk) 20:08, 14 February 2008 (UTC) Eon Zuurmond[reply]

  • Yes, some spots of your hand/head will absorb more than others when you use a mobile phone. There's lots of people doing research and modelling in this and other areas, MTHR in the UK for example. I don't think they've found anything to worry about yet.
  • I think the HIV destruction was by ultrasound not microwaves, so yes they will travel much slower. You'd have a job getting an HIV virus to absorb much microwave energy at 60 GHz anyway, you'd probably fry the surrounding cells first and end up killing the virus that way.
  • I seem to remember that there were initially some worries about the TETRA system having a pulse repetition frequency of around 17 Hz which was close to the resonant frequency of a particular cell or reaction between cells. There's probably something about this in the Stewart Report if you want to look it up. JMiall 20:21, 14 February 2008 (UTC)[reply]

can marijuana be consumed anally

and get high off of it — Preceding unsigned comment added by Boomgaylove (talkcontribs)

I would think not. The large intestine usually only absorbs liquids, so you can get drunk in this manner (one Darwin award winner died in this fashion), but not high off solid material. THC is not soluble in water either. Lastly, I highly discourage taking this path to get high, or any path. 206.252.74.48 (talk) 19:42, 14 February 2008 (UTC)[reply]
I believe the THC in marijuana undergoes a small chemical change when it is heated (decarboxylation?) which converts it to its psychoactive form. Supposedly injesting uncooked marijuana will not get you high. (This is according to a book by Ed Rosenthal). ike9898 (talk) 20:09, 14 February 2008 (UTC)[reply]
you reduce your credibility by saying "or any path". —Preceding unsigned comment added by 79.122.19.82 (talk) 20:56, 14 February 2008 (UTC)[reply]
What? I just vehemently oppose psychoactive drugs, and alcohol. My life has been the better for it. 206.252.74.48 (talk) 21:01, 14 February 2008 (UTC)[reply]
Someone who tells you marijuana can be consumed anally might just be blowing smoke up your ass. Edison (talk) 20:29, 15 February 2008 (UTC)[reply]

crystal meth

how much does an ounce of crystal meth cost? — Preceding unsigned comment added by Boomgaylove (talkcontribs)

"In the U.S. illicit methamphetamine comes in a variety of forms, at an average price of $150 per gram for pure substance" from Methamphetamine#Production_and_distribution, and ounce is ~28g87.102.114.215 (talk) 21:38, 14 February 2008 (UTC)[reply]
Note that an ounce is a LOT. I would think (based on friends I used to have) that it would take a regular user maybe a week or two to go through just a gram. --140.247.11.3 (talk) 01:04, 15 February 2008 (UTC)[reply]
The typical price of an ounce might be less than implied by the stated price per gram, since larger amounts may be sold wholesale, to allow a markup for the risk and bother of selling it to all the crack heads and risking arrest. Law enforcement people love to show some huge shipment of confiscated drugs and announce its huge "street value." A far smaller sum would likely have changed hands had it reached the drug lord it was intended for. Edison (talk) 16:50, 15 February 2008 (UTC)[reply]

Dinosaur names

Why are dinosaur names written with a capital letter and in italics? I've seen this in both Spanish and English. --Taraborn (talk) 20:07, 14 February 2008 (UTC)[reply]

See binomial nomenclature. ike9898 (talk) 20:10, 14 February 2008 (UTC)[reply]
It's standard formatting and nomenclature - kinda like for bacteria, which is done in italics. Wisdom89 (T / C) 20:11, 14 February 2008 (UTC)[reply]
Oh, kinda obvious. Thanks :) --Taraborn (talk) 20:27, 14 February 2008 (UTC)[reply]

Earthquake question

I'm working on a science project for an earthquake unit. I need to know of an earthquake that was measrured in the mercalli scale, where and when it happened. I need the answer before 5 'oclock eastern time. Please help me. QUICK! =0 —Preceding unsigned comment added by 72.221.113.46 (talk) 20:24, 14 February 2008 (UTC)[reply]

From what I can find, the Modified Mercalli Intensity Scale is still in use, this page shows the ratings for some areas of California after the 1989 Loma Prieta event. I haven't found any specific quakes using the original 1902 10-point version, but knowing it was in developed in 1902 might lead in the right direction. --LarryMac | Talk 21:11, 14 February 2008 (UTC)[reply]
Given the 1902 date, the 1906 San Francisco earthquake is an excellent contemporary example. — Lomn 21:22, 14 February 2008 (UTC)[reply]
Are we supposed to answer or honor obvious homework questions? Wisdom89 (T / C) 21:39, 14 February 2008 (UTC)[reply]
We are allowed to help.. as much as we want to.87.102.114.215 (talk) 21:54, 14 February 2008 (UTC)[reply]
Yes and no. We're explicitly encouraged not to do someone's homework for them, but also encouraged to be helpful in assisting. I figure that "name a measured earthquake" is a pretty trivial homework question if that's the whole thing -- likely any major earthquake postdating the Mercalli system has been measured by it. Do you think I've crossed that line via the post above? — Lomn 21:57, 14 February 2008 (UTC)[reply]
The poster still has there report/essay to write - so no I don't think you've overstepped the line - you were very helpful in the case of an honest and straightforward request for info. Well done.87.102.114.215 (talk) 22:16, 14 February 2008 (UTC)[reply]

mixing chemicals

What cleaning chemical is produced when you mix salt, bi-carb of soda and vinager, I saw it used on the TV for cleaning drains 121.200.39.12 (talk) 22:12, 14 February 2008 (UTC)[reply]

The salt isn't part of the main reaction, but bicarb of soda (aka sodium bicarbonate) and plain vinegar (aka acetic acid) react to form sodium acetate, carbon dioxide gas and water. It's the reaction itself, in producing small bubbles of carbon dioxide, that does a lot of the cleaning work (it's like a gaseous exfoliation), although the individual chemicals have cleaning properties of their own, which is why it's such a multi-purpose cleaning method. Confusing Manifestation(Say hi!) 22:27, 14 February 2008 (UTC)[reply]
I suppose the salt introduces an abrasive providing it's not dissolved. Julia Rossi (talk) 23:45, 14 February 2008 (UTC)[reply]
The sodium chloride will dissolve in the aqueous portion of the vinegar which is only 3 % acetic acid. So it will probably not be abrasive. —Preceding unsigned comment added by 79.76.141.105 (talk) 00:36, 15 February 2008 (UTC)[reply]
That all depends on how much vinegar there is compared to the solids. Salt isn't infintely soluble, and it and the bicarb compete with each other in terms of solubility (common ion effect for sodium ions). Any excess would remain as a solid abrasive. Then later it could be rinsed away with excess water, so there's no solid residue to scrape out of ther drain. DMacks (talk) 20:25, 15 February 2008 (UTC)[reply]

February 15

Science

how to calculate head from the body radiation in given room temperature? —Preceding unsigned comment added by 123.201.38.52 (talk) 00:37, 15 February 2008 (UTC)[reply]

Can anyone understand this question? I can't even parse the grammar. Maybe it's about black body radiation? —Keenan Pepper 03:42, 15 February 2008 (UTC)[reply]
"Head" is the hydraulic equivalent of electromotive force. If you have a dam, the farther the water can descend to the turbine the greater the "head" and hence the more work it can do at a given flow rate. I can't even begin to relate that to black body temperature, except a higher temperature also implies more energy per photon, like more energy per cubic meter of water with greater head. No idea if this is related to the questioner's intent. Maybe they meant "heat" rather than "head." Edison (talk) 16:45, 15 February 2008 (UTC)[reply]
Maybe it's about how to calculate how much of a head your beer will have, given the temperature of the room you're in, and given how much heat is radiated from your body (especially your hand, via the mug) into the beer? Or how much body heat is radiated from your head, given the room temperature? MrRedact (talk) 17:08, 15 February 2008 (UTC)[reply]
I find the beer question more interesting, but we require more information. A stout with nitrogen injected will produce much more head than a lager, for example. This chart may be of use. However I rarely see beer served at room temperature although I hear they do that in the UK. I'd imagine the OP meant heat from black body radiation but my expertise relates more to beer. Mad031683 (talk) 17:59, 15 February 2008 (UTC)[reply]
(OR) My father, an electrical engineer, had a high regard for Lucas Electric's engineering staff, but rather less for their sales and management (they would promise buyers too much; forcing the engineering & manufacturing sides of the company to push technology, leading to nice kit which would fail early). He used to say that the English don't truly prefer warm beer, but they are forced to drink beer at room temperature because they have Lucas refrigerators. -SandyJax (talk) 19:56, 15 February 2008 (UTC)[reply]

Japanese Monkey

I recently saw a monkey in Gifu Prefecture, Japan. It was up in the mountains (the Japanese Alps), sitting in the snow, eating some sort of plant (dead grass? I'm not quite sure). It was quite big, covered with light grey fur, and had a pink face. What species was it? And is it the same sort of monkey that I've heard bathes in hot springs? (there were hot springs near the place where I saw it). And while we're on it, how many species of monkey live in Japan?

202.124.215.123 (talk) 00:57, 15 February 2008 (UTC)Kagome[reply]

Try Macaque.--Stone (talk) 06:27, 15 February 2008 (UTC)[reply]
The Snow Monkey, one cool animal! (Japanese Macaque, Macaca fuscata).--Eriastrum (talk) 20:18, 16 February 2008 (UTC)[reply]

Energy

How efficient would the collision of two neutrons traveling at 100% 99% the speed of light in total conversion of mass to energy be compared to the total conversion of mass to energy in the colliding (at any speed) of a neutron and an antineutron? Zrs 12 (talk) 01:33, 15 February 2008 (UTC)[reply]

They can't travel at 100% the speed of light, as this would imply they have infinite energy. Someguy1221 (talk) 01:34, 15 February 2008 (UTC)[reply]
Hypothetically. Or would this mean when they collided that infinite energy would be released?--Zrs 12 (talk) 02:01, 15 February 2008 (UTC)[reply]
The "infinite" means that you get them to 100% of the speed of light you have to give them infinite energy. Since you can't do that, you can't get them that fast. Ariel. (talk) 02:53, 15 February 2008 (UTC)[reply]
This would mean they have infinite relativistic mass, which would mean infinite gravity. — Daniel 02:07, 15 February 2008 (UTC)[reply]
Yes, I know. Thats why I said hypothetically. Anyway, to avoid all of this, I will just change the question. Better? Zrs 12 (talk) 02:16, 15 February 2008 (UTC)[reply]
Are you sure you are asking the right question? mass to energy of two neutrons? None of the mass will be converted to energy. It will all stay as mass. A neutron can decay to a proton and electron, but that's it. It was no method of converting it's mass to energy. When they collide the energy (from velocity) will be converted to mass, and photons. If a neutron and antineutron collide then all the mass will be converted to energy, but lots of energy from velocity will also be converted to mass and photons, just like in the first situation. Ariel. (talk) 02:26, 15 February 2008 (UTC)[reply]
So colliding two neutrons at 99% the speed of light will not produce any energy at all? Zrs 12 (talk) 02:35, 15 February 2008 (UTC)[reply]
What do you mean by "produce"? The velocity of the particles is already energy. Ariel. (talk) 02:39, 15 February 2008 (UTC)[reply]
He knows that. Yes, colliding two neutrons will produce energy in the form of heat and other radiation. 64.236.121.129 (talk) 14:20, 15 February 2008 (UTC)[reply]
BTW mass and energy are the same thing, except that mass has quantum numbers. Since the only quantum numbers in your example come from the neutrons that's the only thing that matters, the speed is irrelevant. There is no "conversion" - they are both the same thing. The only thing that matters is what happens to the quantum number of the neutrons. If you get some particles emitted from the collision then you always emit them in pairs, one mass, and anti-mass. If you look at the entire system as a whole, it started as energy and they are still energy since the particles are opposites and cancel out. Perhaps you are wondering how many particles will be emitted from the two types of collisions? I'm pretty sure it's the same both ways. Ariel. (talk) 02:38, 15 February 2008 (UTC)[reply]
No. I knew ^^ that. I mean energy "produced" if you will as it is in a neuclear fission/fusion reaction. Zrs 12 (talk) 03:07, 15 February 2008 (UTC)[reply]
Um, OK. The energy produced in a nuclear reaction is due to the change in binding energy of the nucleus. That doesn't apply to two individual neutrons. Instead of keeping some energy as binding energy we release it. Like if you have a supply of stretched rubber bands. You can release the stretch and power stuff from it. But the energy in the rubber bands was already there, we just released it from inside the rubber band to outside. This doesn't apply to smashing neutrons. I still think you haven't asked the question you really intend. Why don't you write what idea you are thinking of, and I'll see if I can help you understand. Ariel. (talk) 03:17, 15 February 2008 (UTC)[reply]
No, this is the question I intended. Would the quarks in the neutron not have releasable binding energy? And on a completely different note: Why in the Schrodinger's Cat paradox can the cat not be considered an observer and collapse its own wavefunction? Maybe there is something I'm missing? Thanks, Zrs 12 (talk) 03:25, 15 February 2008 (UTC)[reply]
Excellent questions! To the first: to release binding energy you have to go to a lower binding energy state. In atoms the lowest is nickel and iron. You can't get any energy from them, because no matter which way you go (more protons or less) you need to add energy. Currently there is no known lower state for the quarks in a neutron to go to (except a proton and electron see neutron decay). But I should point out we don't know a lot about quarks. But I'm pretty sure a lower state doesn't exist, because if it did it would have shown up. But see the article on proton decay for a possible lower state for protons. To the second question: the cat is basically two cats, one dead, one alive, both at the same time. Each of them observes itself, but that doesn't affect the other. See Quantum immortality for a rather interesting possible effect. Ariel. (talk) 03:52, 15 February 2008 (UTC)[reply]
Thanks. So does this mean that we are all more than one person? Furthermore, in the Copenhagen interpretation why would the cat not be able to collapse its own wavefunction? I assume you were talking about the Many-worlds interpretation. Thanks, Zrs 12 (talk) 15:12, 15 February 2008 (UTC)[reply]
There's no reason to suppose it couldn't, but it really doesn't matter in this experiment. It makes no empirical difference when the wave function collapses, even from the cat's perspective. The only nonclassical event in the whole experiment is the decay of the radioactive atom. You can just as well suppose that the collapse happens right after the decay, long before the signal is even amplified. You can even suppose that the decay arises from classical chaos too. As an attempt to show what was new and strange about quantum mechanics, Schroedinger's cat was a complete failure. The double slit experiment is where it's at. -- BenRG (talk) 02:15, 16 February 2008 (UTC)[reply]
Agreed; what Schrodinger should have proposed was throwing cats at a wall until two of them cancelled out. Someguy1221 (talk) 03:34, 16 February 2008 (UTC)[reply]
Haha, yeah I would like to see two cats cancel out. The double slit experiment is easier to understand (at least to me) too. Ha, and this whole time I thought the whole cat thing was about when the wavefunction collapsed. Zrs 12 (talk) 17:28, 16 February 2008 (UTC)[reply]

Full-on beak-locked combat in gulls...?

As seen in this video (and others on YouTube) - what exactly is this purpose behind this extreme aggression? This tug-o-war style squabbling is much more violent and intense than the typical inter-gull bitching, bickering and chasing for food and status you normally see. I've yet to hear a satisfactory explanation for what is a fairly uncommon behaviour (I've only seen it in real life a couple of times myself - and I watch gulls a lot). I've heard several suggestions (two equally-matched male birds fighting over a female, two male birds fighting over a particularly lucrative territory, or a female bird testing the strength of a male before deciding to pair up with him) but no-one I've ever asked seems to know for certain. Any ideas, folks? This one has been bugging me for a long time. --Kurt Shaped Box (talk) 01:39, 15 February 2008 (UTC)[reply]

Kurt, by now you are an expert on gulls. You should discuss this type of question with other experts, and then (citing the appropriate references) add the answer to the gull article. I do hope that you are by now in correspondence with other gull experts. -Arch dude (talk) 03:16, 15 February 2008 (UTC)[reply]
Haha, great answer. —Keenan Pepper 03:35, 15 February 2008 (UTC)[reply]
Where the heck is the Gull Ref Desk? There would be a load more interesting questions than they have in the dying "Entertainment" section. 206.252.74.48 (talk) 13:32, 15 February 2008 (UTC)[reply]
It was deleted. See this discussion for example. Algebraist 15:28, 15 February 2008 (UTC)[reply]
Ah, thanks for that. I knew it was mentioned at one point. I never get bored of Kurt's questions, because they are always valid and thought-provoking. As for this question: some individuals are more violent than others, that is all. This is true across the animal kingdom. I'm sure you've seen some meek seagulls, cowardly seagulls, and maybe even stupid seagulls, what is there to say there aren't psychopathic ones? —Preceding unsigned comment added by 206.252.74.48 (talk) 16:51, 15 February 2008 (UTC)[reply]
Thanks for that. It's certainly possible that it's just a case of two hot-tempered, particularly violent gulls clashing. Usually, a confrontation between gulls is a case of 'one bird attacks - the other immediately runs/flies away, with lots of noise made on both sides' - that happens all the time whenever gulls are in close proximity. Yes, what we are seeing could certainly be the end result of a bird refusing to back down in the face of another.
I've found it quite difficult to locate detailed/reliable information about gull social behaviour. It doesn't seem to be a particularly well-studied field. There are many 'gull experts' on the internet who know a lot about what different species of gull *look like*, gull migratory/nesting habits and gull physical biology - and many people who like to photograph gulls, then discuss their plumage patterns at length. If, OTOH you need to know about how gulls behave and communicate, then there are many contradictory accounts and personal theories from amateur birdwatchers to choose from - but little in the way of hard science backed by reliable sources. I have been unable to find out much about the 'pecking order' in gulls, for example - other than that "yes - they have one". Someone here did once recommend a book (quite an old book, IIRC) on this subject a couple of years ago, but I have been unable to re-find the thread in question and I cannot remember anything about the title or the name of the author (if it was you, please chime in - I'd be much obliged). --Kurt Shaped Box (talk) 00:53, 16 February 2008 (UTC)[reply]
No book, but i found this nice little piece on territorial behaviour in groups of chickens, gnats, gulls, cryptic moths, gulls, the purpose of fighting and spacing[11]. Attrition seems to be the name of the game. Julia Rossi (talk) 13:29, 16 February 2008 (UTC)[reply]

Kurt, have you read the classic book on gull behavior: The Herring Gull's World by Niko Tinbergen, published way back in 1953? Tinbergen was one of the founders of the whole field of animal behavior. His book is immensely readable and shows how clever Tinbergen was in making simple field experiments that answered many questions about gull behavior. In fact, I recommend reading any of his varied books. As to why gulls get into fights, one of the reasons is surely wrangling over the dominance hierarchy.----Eriastrum (talk) 20:10, 16 February 2008 (UTC)[reply]


I aren't able to see the video, but locking beaks together when fighting doesn't necessarily imply more violence. The locked gulls would then not be pecking or pulling feathers; and they could directly determine each others strength instead of bluffing.Polypipe Wrangler (talk) 21:05, 16 February 2008 (UTC)[reply]

Black hole evaporation

When a black hole evaporates, what happens to its electric charge and angular velocity? — Daniel 02:04, 15 February 2008 (UTC)[reply]

The emitted particles have it. Look at it backward, instead of wondering what happens, nothing can happen unless electric charge, and angular velocity, and everything else is taken care of. If something prevented that, it won't evaporate in the first place. (BTW both black holes and evaporation thereof have never been seen, so it might not exist.) Ariel. (talk) 02:30, 15 February 2008 (UTC)[reply]
Aren't the emitted particles photons? They can't carry charge. — Daniel 03:36, 15 February 2008 (UTC)[reply]
You are the second person today who told me that blackholes emit photons (isn't that a contradiction it terms?). They don't, they emit a whole zoo of random particles, but not photons. They emit anything that has a matter/anti-matter pair. A photon is it's own anti particle and will not be emitted. Ariel. (talk) 03:43, 15 February 2008 (UTC)[reply]
They do emit a whole zoo of random particles, as you say, but I think photons are allowed to be in the mix. The whole "one particle of a virtual pair falls in, the other escapes" explanation is too simplistic to make this kind of prediction. Then again, the correct description of the process (quantum field theory on a curved space-time manifold) is way over my head, so you may be right. —Keenan Pepper 04:04, 15 February 2008 (UTC)[reply]
What makes you think that means they won't be emitted? A virtual particle pair can be made up of two photons and in fact photons would be a primary form of emission associated with an evaporating black hole. Dragons flight (talk) 06:43, 15 February 2008 (UTC)[reply]
Cool, that agrees with what little intuition I have about this. —Keenan Pepper 23:06, 15 February 2008 (UTC)[reply]

What exactly is a magnetic field?

All my life I have wondered, is it an OBJECT of some kind, or a process? Is it animal, vegetable or mineral? I’ve seen diagrams where it is stretched, warped, looped and tied up in knots; but what KIND of thing is it exactly? Can you spin it into a pair of pants? Can you heat or cool it? What would it look like under an atomic microscope? What the hell is it? No technical explanations please. This is just a general enquiry Myles325a (talk) 02:43, 15 February 2008 (UTC)[reply]

I'd say a magnetic field is an abstraction - it's something used to sum up or describe a physical property of different points in space. The property being described is magnetic field strength - i.e. the extent to which a moving electric charge etc would experience a force. The property is determined by whehther and how much stuff there is around that point which exerts a force on electric charges etc, and how far away they are.
The field itself has no physical existence - it merely describes how much force you experience when you go near, say, an electromagnet (if you are a charged particle, that is). The force is caused by the magnet, and it is being experienced by you. When you are "outside" a field, that just means you are so far away from whatever is causing the magnetic force, that you don't experience it. --PalaceGuard008 (Talk) 03:39, 15 February 2008 (UTC)[reply]
But what is the argument that forces are less an abstraction than fields? As I discovered trying to research a question from last week, I can't find much in Wikipedia about the philosophy of forces. --Allen (talk) 05:27, 15 February 2008 (UTC)[reply]
The nature of science being to describe rather than explain, everything is an abstraction. Force is only formally explained in the fields that deal with the nature of the forces themselves: General relativity (or quantum gravity) for gravity, quantum electrodynamics for electric and magnetic force (essentially the same thing), and quantum chromodynamics for everything else. The best way to learn this without spending eight years learning advanced physics is just to accept that particles have interactions with eachother (explained in the above links, I presume, I don't actually feel like checking), these interactions are described by an equation involving a force, and a force field describes the force on a test particle at an arbitrary position in space. So then, neither the force nor the field really exist. They're just mathematically succinct ways of describing the nearly impossible to understand interactions between particles, for the benefit of everyone only cares what the end result is (which is just about everyone). Someguy1221 (talk) 05:37, 15 February 2008 (UTC)[reply]
  • No, the nature of science is not to "describe rather than explain" as you put it. A scientific law is a description of observed phenomena, while a scientific theory is an explaination of observed phenomena. Theory is a big part of why we do science. 64.236.121.129 (talk) 14:15, 15 February 2008 (UTC)[reply]
This is getting a bit off topic, but, what exactly is the difference between explaining and describing?128.163.80.188 (talk) 18:44, 15 February 2008 (UTC)[reply]
A scientific theory is just a model used to consistently hosue mathematical descriptions for the purpose of predicting events. If you ever study the history of science, you'll notice the distinct prominence of explanation that prevailed in pre-Newtonian/Galilean physics, for example. The ancient theories of science (mostly Aristotelian), and notable later theories such as those of Descartes, endeavored to explain, well, why things do what they do. Why do solid objects fall to the ground? Why does smoke rise? These theorists came up with very detailed explanations, yet they themselves predicted nothing. Alternatively, a pioneer in modern science such as Newton produced a mathematical description of how objects interact, with the goal of predicting as yet undocumented systems of interaction. Of course there was always a larger theory behind it all (and Newton's was generally wrong). But then you get to the crux of the matter. Even though Newton's explanations were essentially always wrong, we still learn his equations to this day. And that's because his descriptions were very accurate. And for three hundred years since, theories have been overturned in favor of newer, better ones (ok, that mostly stopped once quantum was established, but plenty of physicists are still trying). The descriptions that these theories have produced have generally not been overturned (sometimes they're rendered useless or obsolete, however), and they are often not doubted even when their base theories are proven wrong. And this is all completely opposed to those earlier thoeries I mentioned, in which the descriptions stem from the explanation (and not the other way around). And so I feel quite justified in claiming that the purpose of science is to describe rather than explain. Although you can define those terms to force the validity of that statement to whatever you desire. Someguy1221 (talk) 20:08, 15 February 2008 (UTC)[reply]
Einstein in his book About Special and General Relativity Theory (I can't remember the title exactly, but it talked about the theories from a philosophical standpoint), called the gravitational and the magnetic field "something physically real". --Taraborn (talk) 07:00, 15 February 2008 (UTC)[reply]
Science is as much about explaining as it is about describing, if not more so. Unless you are a physicist who works at the very bottom of the hierarchy of abstractions, science is about describing observed phenomena and explaining apparently diverse phenomena as manifestations of a relatively small number of more fundamental phenomena. --71.162.242.55 (talk) 09:45, 15 February 2008 (UTC)[reply]
Now see my big comment above. Unless you don't care to see your science applied in your own lifetime, your only impetus to produce a novel explanation is to arive at a better description :-p Someguy1221 (talk) 20:22, 15 February 2008 (UTC)[reply]
Force is a concept that allows certain phenomena to be described succinctly. The phenomena described are real (as in actually observed), but their descriptions do not have to involve the concept of force. You can think of force as an accounting device for objects' tendencies to accelerate in certain situations. You can describe the gravitational attraction between two objects in terms of mass, acceleration, and the distance between the objects. You can also describe the attraction/repulsion between two charged particles in a similar manner. Now what if you have two objects that have both mass and electric charge? Do we need a whole new law to describe how they tend to accelerate? Fortunately the answer is no. The concept of force presupposes that the tendencies objects to accelerate as described by different "laws of physics", as we have formulated them, are combinable. The concept of force allows such combination to be done mathematically via vector summation. --71.162.242.55 (talk) 09:33, 15 February 2008 (UTC)--71.162.242.55 (talk) 09:33, 15 February 2008 (UTC)[reply]
Did somebody axe the pigeons? Julia Rossi (talk) 09:46, 15 February 2008 (UTC)[reply]
Ah, they on the other desk here{http://en.wikipedia.org/wiki/Wikipedia:Reference_desk/Humanities#What_exactly_is_a_magnetic_field.3F] —Preceding unsigned comment added by Julia Rossi (talkcontribs) 09:59, 15 February 2008 (UTC)[reply]
[I merged the duplicate threads; moved material is below.]
Well, I'm curious. In the documentary War of the Birds re their ability to navigate, pigeons were described as "seeing" the earths magnetic field to explain how they found their way home over say 250 miles (in those days) over a featureless ocean (oh, and in the dark). There's a bit here[12] in the homing pigeon article. Wondering, in what way is it visible? Julia Rossi (talk) 08:17, 15 February 2008 (UTC)[reply]
Well, we don't really see distant objects; we see light that of its own accord enters the pupils of our eyes, and infer the existence of distant objects from that. Static magnetic fields don't emit anything, so there's no way any animal could "see" distant magnetic field lines. I assume they're only aware of the direction of the field through their body, i.e. they have an internal compass. -- BenRG (talk) 11:53, 15 February 2008 (UTC)[reply]
The use of "see" here is a metaphor for "sense". It's a fairly common metaphor, do you see what I mean? JohnAspinall (talk) 15:57, 15 February 2008 (UTC)[reply]
A magnetic field is a stress in the luminiferous aether. I'm only half joking. As far as the Standard Model is concerned, everything in the world is a field, and the physics of the fundamental fields is close enough to the physics of vibrations and stresses in a solid. A static electric or magnetic field is a state of stress. If the field varies, you get sound waves that propagate though the solid; that's light. Ordinary matter is made of different kinds of vibrations in the field. Your body is a kind of soliton. -- BenRG (talk) 12:14, 15 February 2008 (UTC)[reply]
A magnetic field is a forcefield. Ofjects within the magnetic field will be attracted to the object producing it. The Earth has a magnetic field. You cannot touch or feel it. You cannot see it under a microscope (unless it was on paper and there were iron filling scattered about). You cannot weave it into pants. A magnet produces such a field. You cannot eat a magnetic field. It is not a object, but a field containing magnetic force. A magnetic object always has two poles. The poles will attract unlike poles and repel like poles. You cannot cut a magnet to isolate one pole. Was that simple enough? Hope this helps. Thanks. ~AH1(TCU) 22:41, 15 February 2008 (UTC)[reply]
Dr Karl says watch out for Norberg[13] if you are a pigeon. Julia Rossi (talk) 13:39, 16 February 2008 (UTC)[reply]

science fair

hey, i need a science fair project with all the oooohhs and aaaaaaahhhhh if you know what i mean. i know the judges are suppose to judge on the other stuff but at this fair they don't. i need help bad! please help me! —Preceding unsigned comment added by 76.14.124.175 (talk) 06:37, 15 February 2008 (UTC)[reply]

A fission reactor would definitely illicit some sort of response (perhaps including a call to the police, or if you're in the US maybe DHS), though it may also result in an unpleasant demise if not executed properly. But more to the point: I think Do your own homework may apply here. -- Consumed Crustacean (talk) 06:50, 15 February 2008 (UTC)[reply]
That's "aargh!", not "aaahh". I don't think Do your own homework applies: s/he is going to, but just wants suggestions. Never having been to a science fair, I don't have any I'm afraid.AlmostReadytoFly (talk) 08:51, 15 February 2008 (UTC)[reply]
Regardless of your project, make sure you have pretty graphs and charts. Scientists fall for that stuff all the time! (EhJJ)TALK 12:31, 15 February 2008 (UTC)[reply]
I seem to recall a story of a Boy Scout building a working fission reactor in a shed to earn an activity badge. And one of my mates built his own fusor, but it took him several months and a lot of money. You probably want something slightly simpler... the wub "?!" 12:36, 15 February 2008 (UTC)[reply]
The Boy Scout was David Hahn and he got in a lot of trouble with it. Also, that was very much pre-9/11. I wouldn't play with nuclear materials personally unless you'd like a visit from the FBI, the NRC, AND now Homeland Security. The worst part, though, is that on account of having exposed himself to that much radiation he is not allowed to do anything else with reactors for the rest of his life (or in more scientific terms, he has already maxed out his lifetime exposure rating)! --98.217.18.109 (talk) 20:37, 15 February 2008 (UTC)[reply]
How about a self built electric motor - with all the parts visible and with home wound coils etc... Is that too simple - what sort of level are you at eg age/class number?83.100.183.231 (talk) 14:19, 15 February 2008 (UTC)[reply]
If all you want is something fancy, wire up a bunch of lights to blink for no apparent reason. If you want to win the science fair, you must have a hypothesis. You must have an experiment which tests your hypothesis in a scientific, controlled way. You must use the results of those experiments to draw a conclusion about your hypothesis. Most people fail to realize that the purpose of the science fair is not to build something. It is to demonstrate a knowledge of the scientific process of hypothesis, experimentation, and conclusion. -- kainaw 14:25, 15 February 2008 (UTC)[reply]

I'm doing a project on the fluorescence of Induim Phosphide quantum dots. My teachers have said it will probably get some oohs and aahs. I ordered the stuff from Cenco Physics. It helps if you have access to a spectrometer for it but, it's not absolutely necessary. Hope this helps (I had trouble finding one too), Zrs 12 (talk) 15:17, 15 February 2008 (UTC)[reply]

I know a potential fair attender who is legally blind (20/800 - they can see but not real well, and glasses don't help) and I can appreciate the need to ask for ideas. I would be tempted to ask, myself, what can be done where he doens't have to see much - I'm thinking some sort of bridge, but would that be more engineering? You're right, having a hypothesis and being able to explain is crucial, but they do need to be able to design something, I think.
With baseball season upon us almost, I was thinking for this situation something that might intrigue you, as it's probably a bit advanced for 8th grade, which this student is. He's a huge baseball fan, and was thikning about a pitching machine that would release the ball at various speeds and manners, and explaining why balls curve, what makes a knuckleball doe what it does and the effects of even a little spin making it straight, etc. - if you have enough room, you might be able to do that.
But, with 8th grade, thta's a bit advanced, as I say.4.68.248.130 (talk) 18:01, 15 February 2008 (UTC)[reply]
How did he get the number of 20/800? As you can see with a question far above I asked how I could calculate my numbers, and came up with 20/14400, but my glasses bring me up to about 20/15. How can glasses not help with someone with much better eyesight than me? Are my numbers completely wrong? And just when I thought I solved this mystery...206.252.74.48 (talk) 19:24, 15 February 2008 (UTC)[reply]
You're assuming that he's nearsighted. He may have some other, more serious eye problem. APL (talk) 21:35, 15 February 2008 (UTC)[reply]
According to my simplistic calculation, if you can only recognize an optotype at 20 feet when a person of 20/20 vision can recognize it at 14400 ft, the optotype would have an angular magnitude of about 55°! That sounds really awful. Are you sure you did your calculation right? --71.162.242.55 (talk) 01:00, 16 February 2008 (UTC)[reply]
Most science fairs encourage you to actually use the scientific method, so simply building something neat usually doesn't get you many points, at the very least you'll need a pretense for building the neat thing. (For example, testing the durability of various building materials might give you an excellent excuse to build a trebuchet or a battering ram.) Projects involving insects are usually good crowd-pleasers. Maybe ants. APL (talk) 21:35, 15 February 2008 (UTC)[reply]
Check out this book. It may give you some ideas. --> http://www.gutenberg.org/ebooks/14664 (Get the PDF version. It's got pictures.) APL (talk) 01:53, 16 February 2008 (UTC)[reply]

RDI

What is the RDI of KJ for an 18 year old, 60 kg male? Thanks in advance.Cuban Cigar (talk) 10:05, 15 February 2008 (UTC)[reply]

According to Food_energy, it's 10,000KJ/day for a man. (I'm assuming your abbreviations are for Recommended Daily Intake and kilojoules.) AlmostReadytoFly (talk) 11:48, 15 February 2008 (UTC)[reply]
I could be wrong but I don't think there's really enough information there to give a reasonable estimate. For starters, the height will be needed (a 2 metre guy will generally have a larger RDI then a 1.5 metre guy) and your level of daily activity (if you spend all your time at the computer your RDI will be lower then if you are a triathelete in training). Nil Einne (talk) 17:30, 16 February 2008 (UTC)[reply]

Hi. Yes, this male is around 1.55-1.6 m, and has low-to -moderate activity (Ie excercises regualarly for half an hour 2-3 times a week on average)220.237.156.78 (talk) 23:10, 16 February 2008 (UTC)[reply]

Wrist watches that read your heart rate

How does this watch ([14]) measure oxygen and energy consumption? Zain Ebrahim (talk) 14:16, 15 February 2008 (UTC)[reply]

I'm guessing that might be similar to Pulse Oximetry, which uses wavelengths of light absorbed by oxygenated and deoxygenated hemoglobin to determine oxygen saturation in the blood. Wisdom89 (T / C) 19:30, 15 February 2008 (UTC)[reply]

Radiation Scope Thingy

I was going to reply to a question above with a reference to a device, I decided not to reply anyway, but I wanted to read about the device and completely forgot it's name. Searching all of Wikipedia and the internet has been fruitless. I am referring to a little scope that lets you see radiation from a small inbedded isotope. It was released as a novelty toy during the 1950s. I know the name ends in -scope, but I'm stumped as to the first part. I first heard about it on Wikipedia, so I know the article is here! 206.252.74.48 (talk) 14:43, 15 February 2008 (UTC)[reply]

Never mind, I found it, it's the Spinthariscope. 206.252.74.48 (talk) 14:55, 15 February 2008 (UTC)[reply]
You can buy them at http://unitednuclear.com, incidentally. --98.217.18.109 (talk) 20:38, 15 February 2008 (UTC)[reply]

A commerical Freeze Dryer used for making instant coffee

In you encyclopedia you have an artical on making Instant Coffee by freeze drying using a commerical freeze dryer. I would like to know as who manufactures this type of large commerical freezze dryer? —Preceding unsigned comment added by 70.240.79.154 (talk) 17:49, 15 February 2008 (UTC)[reply]

Google search result for "freeze drying equipment". --hydnjo talk 17:59, 15 February 2008 (UTC)[reply]

Nuclear bomb effects in space?

I am wondering about this because I am quite a fan of science fiction and space warfare in science fiction. My understanding of the effects of an nuclear bomb (be it conventional fission, "neutron," boosted fission, or thermonuclear) on Earth is that much of the damage is caused by the overpressure wave and thermal radiation (i.e. the 'shock-wave' and intense heat). In space, there is (virtually) no medium for an overpressure wave. Does this mean that a nuclear bomb will have less of an effect in space, or will more of its energy go into, say, thermal radiation? If it does, will the lack of convection (as I understand it, a nuclear bomb creates a very hot wind after the initial shock wave has passed) lessen the thermal effect of the bomb?

I am very familiar with the electromagnetic pulse and radiation effects so I don't need to know more about that.

Thanks! Zephyrus67 (talk) 18:32, 15 February 2008 (UTC)[reply]

NUUUKKKEESSS INNN SPAAACCCEEE! Um. With that out of the way, ... without an atmosphere, there is no blast effect. And without a way of conducting the heat, there is practically not thermal effect. However the radiation effects will be greatly enhanced if there is no atmosphere to stop or scatter it! This page has some great graphs illustrating the differences in effects. --98.217.18.109 (talk) 20:46, 15 February 2008 (UTC)[reply]
(edit conflict) I guess most people haven't had the experience of being near something that's extremely hot. I don't mean just really, really hot like a roaring wood fire, but astronomically hot. One time I was messing around with gunpowder, smokeless powder from a rifle round, and I set a little pile of it on fire with a match. It didn't blow up, not being confined, but my fingers got burned from the radiant heat. The temperature of the heat source makes a big difference. You know how on a clear day the sun actually sort of burns on your skin? Now imagine the sun was three or four time hotter, and you were only 20 miles away from it instead of 93 million. I don't understand our article on the effects of nuclear explosions. What it calls "blast" is caused by heat, the rapid expansion of material near the explosion when it is turned to gas from the heat. In space, the only material would be the bomb itself, which I imagine would turn into plasma and blow out in all directions as atomic particles. There would be no blast as such, but I can't see how the bomb would know to turn the blast energy into something else. It's all about the heat, which is the infrared portion of the electromagnetic radiation that emanates in all directions from the explosion. The hot wind you mention I guess is a firestorm. The wind is only hot because everything is on fire out a certain distance from the site of the explosion. --Milkbreath (talk) 21:03, 15 February 2008 (UTC)[reply]
"Blast" in the sense of that article is air pressure. It's what destroys most of the structures not immediately inside the fireball. The NASA link above disagrees with you about the thermal aspect—"Second, thermal radiation, as usually defined, also disappears. There is no longer any air for the blast wave to heat and much higher frequency radiation is emitted from the weapon itself." I'm imagining the difference here is that normally a lot of the actual heat from a nuclear explosion is caused by radiation scattering off of the atmosphere and releasing its energy that way—remember that most of the energy released in a fission reaction is kinetic energy, the speed of the neutrons and fission products. But maybe I'm misunderstanding something, but it seems that a lot of basic effects change when you are setting off a nuclear explosion in a vacuum. --98.217.18.109 (talk) 23:15, 15 February 2008 (UTC)[reply]
I'm in no position to argue with NASA, but if you follow the link to high altitude nuclear explosions provided by Sean down there, you'll find this: "Heat from the Bluegill Triple Prime shot, at an altitude of 50 kilometers (31 mi), was felt by personnel on the ground at Johnston Atoll, and this test caused retina burns to two personnel at ground zero who were not wearing their safety goggles." Bluegill Triple Prime was 410kt. The air is quite thin at that altitude, the top of the stratosphere. I'd like to know how an atom bomb knows whether there's air around it or not. --Milkbreath (talk) 02:53, 16 February 2008 (UTC)[reply]
If I were to guess, again, the heat was felt by people who were within an atmosphere. Lots of radiation comes out from the bomb, goes a far distance across the cold vacuum of space, and then heats up in the atmosphere. That'd be my guess. Again, I suspect the heat is caused by contact with the atmosphere, at least that's my reading of the NASA page. The bomb doesn't know if there's air around; the large amount of infrared radiation itself is probably caused by the scattering of high-energy particles in the atmosphere, is my guessing. --98.217.18.109 (talk) 14:38, 16 February 2008 (UTC)[reply]
Check out High altitude nuclear explosions and in particular the DOD report at the bottom of Starfish Prime. --Sean 21:14, 15 February 2008 (UTC)[reply]
I'm
Profound statement, that there. 81.93.102.185 (talk) 12:27, 16 February 2008 (UTC)[reply]
Ignore what I said above. Case of knowing a whole lot of stuff from years ago that is wrong. The situation is much more complex than I thought. And, 81.93.102.185, thanks. Just tell me to shut up next time, I can take it. --Milkbreath (talk) 15:35, 16 February 2008 (UTC)[reply]

Can humans hibernate?

Can humans hibernate? Are there any "how to" guides floating about on the interweb? Weasly (talk) 18:19, 15 February 2008 (UTC)[reply]

See human hibernation. Algebraist 18:44, 15 February 2008 (UTC)[reply]
No go there, looks like you'll have to put up with Christmas like the rest of us. Richard Avery (talk) 16:40, 16 February 2008 (UTC)[reply]

Poison through the ear

In William Shakespeare's Hamlet, Claudius murdered King Hamlet by pouring poison in his ears. How is this possible? If I had some liquid poured into my ears, I think I would wake up immediately, and shake the liquid out. Is there some pathway that the ears provide that allow the poison to act so quickly that this it would be impossible to respond to? Sancho 18:35, 15 February 2008 (UTC)[reply]

Maybe King Hamlet is a more sound sleeper than you? There are times when I've slept through people shaking me, they had to slap me to wake me up. Mad031683 (talk) 18:42, 15 February 2008 (UTC)[reply]
I got the impression that it may have made Claudius wake up but poison began acting immediately such that he couldn't tell anyone before he died. This would explain how his ghost knew how it had happened. — Ƶ§œš¹ [aɪm ˈfɻɛ̃ⁿdˡi] 18:51, 15 February 2008 (UTC)[reply]
I always thought it was really quite a weird way to do it. My thinking is that he wanted him poisoned while sleeping, so that rules out something he ate, and he didn't want something that appeared to be obvious foul play (no poisoned knives and swords), and in Shakespeare's time that doesn't leave a lot of options (there weren't hypodermic needles for quite a long while later). But it's still weird. I guess if it were me I'd have gone with spider bite, but that shows how much I know. --98.217.18.109 (talk) 20:49, 15 February 2008 (UTC)[reply]

I think one theory is that it's supposed to be somewhat symbolic or foreshadowing or something like that -- it parallels the poison that the ghost of Hamlet's father pours in Hamlet's ear, metaphorically, regarding Hamlet's mother, which ultimately leads to Hamlet's undoing as he tries to avenge his Oedipal outrage. --Trovatore (talk) 20:59, 15 February 2008 (UTC)[reply]

Hmm, yeah, that actually sounds pretty plausible. Poison in the ear is a great metaphor for dangerous knowledge, etc., even if it sounds like a lousy way to actually kill someone. --98.217.18.109 (talk) 23:08, 15 February 2008 (UTC)[reply]
Could smothering with the pillow have been easily detected back in that era? 206.252.74.48 (talk) 21:00, 15 February 2008 (UTC)[reply]
Any liquid (Water, hydrogen peroxide etc..) that is placed in the ear, pretty much just sits there. I'm not sure how much absorption there actually would be, if any. My guess is that it was symbolic, or for dramatic affect. Wisdom89 (T / C) 21:28, 15 February 2008 (UTC)[reply]
Presumably if the poison was bound to a skin-penetrating substance like DMSO, then it could be absorbed into the bloodstream from the ear canal as much as from the exterior skin. But I don't think any substances like that were known before modern chemistry. --Anonymous, 00:33 UTC, February 16, 2008.
Indeed, and I doubt it was meant to be interpreted literally, and the audiences would have known not to do so. Re the question of metaphorical poison vs. actual poison, this is quite enlightening:
  • In his advice to Reynaldo, Polonius explicitly develops one of the themes of Hamlet, the idea that words can be used to bend and alter the truth. He explains to Reynaldo how to ask leading questions of Laertes’ acquaintances and how to phrase questions in a way that will seem inoffensive. As with Claudius, who manipulated the royal court with his speech in Act I, scene ii, words become a tool for influencing the minds of others and controlling their perception of the truth. Remember that Claudius killed King Hamlet by pouring poison into his ear. Shakespeare continually illustrates that words can function as poison in the ear as well. As the ghost says in Act I, scene v, Claudius has poisoned “the whole ear of Denmark” with his words (I.v.36). The running imagery of ears and hearing serves as an important symbol of the power of words to manipulate the truth.
I think there's something in that for all of us. -- JackofOz (talk) 00:55, 16 February 2008 (UTC)[reply]
That's a very nice metaphor I had missed. I like this answer. Sancho 22:09, 16 February 2008 (UTC)[reply]
I guess another angle on this is that back in the 14th/15th century anatomical knowledge was not as refined as it is today and it may well been thought that the ear connected with the throat and it would be possible to administer poison thus. But having said that I go along with what Jack says, so much of Shakespeare is symbolic it would be reasonable to expect this to be. Richard Avery (talk) 16:37, 16 February 2008 (UTC)[reply]

If the king were severely allergic to bee venom he might well die of anaphalactic shock. Alcohol drinkers often are very difficult to rouse from sleep.Polypipe Wrangler (talk) 21:15, 16 February 2008 (UTC)[reply]

WTH?

You don't even list the large planet Beetlegeuse in Wikipedia. Am i out of my mind? Who's running the show there?  :) —Preceding unsigned comment added by 64.237.99.46 (talk) 21:02, 15 February 2008 (UTC)[reply]

It's spelled Betelgeuse, and it's a star, not a planet. --Trovatore (talk) 21:06, 15 February 2008 (UTC)[reply]
Well, you can 'run the show' if you want to. That's one of the neat features of Wikipedia everybody is 'running the show'. But please don't get angry with those that are 'running the show' because you spelt Betelgeuse wrongly. Richard Avery (talk) 16:30, 16 February 2008 (UTC)[reply]
what he is trying to say is: why is there no spellchecker on the wikipedia search box like there is on google!
The spellchecker is disabled due to performance issues. If you don't know how to spell a word, you can often find its article very easily by Google searching for "<your guess on the spelling> + wikipedia." Someguy1221 (talk) 23:48, 16 February 2008 (UTC)[reply]

visual acuity conversions

Hi. Please convert 0.5, 1.0, and 1.5 to the 20/x way of measuring visual acuity, as well as how many arcminutes each can resolve. This is not a homework question. Also, eyeglasses such as 100, 200, 500 degs, etc, equal how many of 20/x for the visual observer eyes? How many is that for arcminutes? Or are the degrees for nearsighted only? What is the decimal way of measuring known as? How can I convert these numbers, is there a formula or online calculator or something? I realise there are similar questions above, but I would like a clarification of these numbers. Let's say someone can see 5 arcminutes resolution naked. How much resolution with 10x binoculars, for example, or does it depend on the aperture rather than magnification? Does it depend on the Dawes or Raleigh limit as well? Also, a rather unrelated question, is the cloth used to clean glasses called lens cloth? Thanks. ~AH1(TCU) 22:34, 15 February 2008 (UTC)[reply]

I'll answer a couple of your questions (I don't know the answer to all of them). I believe the decimal specification of 'visual acuity' is a specification, in dioptres, of the correcting lens required (please note that this may be wrong). As for resolution in 10x binoculars, this will vary depending on the quality of the binoculars – the quality of the glass (or plastic, if it's a particularly cheap-and-cheerful pair) and its coating, the number of elements and their arrangement, and the like. The 10x merely specifies (as I'm sure you know), the magnification they offer. These are (some of) the reasons that prices for camera lenses (and so, presumably, also binoculars) vary from the low hundreds to many thousands of pounds. Angus Lepper(T, C, D) 01:44, 16 February 2008 (UTC)[reply]
I'm not an eyecare professional, but I think your answer about visual acuity is incorrect. Dioptre is a unit for the power of lenses, not a measure of visual acuity. In my non-expert's understanding, the "degrees" figures in the OP's question refer to the powers of corrective lenses. In the case of corrective lenses prescribed for myopia, a prescription informally referred to as "200" actually refers a lens of -2.0 dioptres power. Visual acuity, as I understand it, is an operationally defined performance measure. It's about what a subject is able to "do" under standard test conditions—not what kind of corrective lenses the subject needs. You can have separate visual acuity measures for corrected and uncorrected vision. --71.162.242.55 (talk) 03:00, 16 February 2008 (UTC)[reply]
That's quite possible, hence my original disclaimer! Apologies if it is incorrect. Angus Lepper(T, C, D) 21:32, 16 February 2008 (UTC)[reply]

EMP and safe deposit boxes

(Note: I bumped this section down to February 15 even though it was initially posted on February 14 because I asked a whole bunch of new questions on February 15 and I wanted to make sure the questions were answered before this section ended up being archived and that people didn't overlook the section just because it was in an earlier day and assume that the questions had already all been answered.) —Lowellian (reply) 02:29, 17 February 2008 (UTC)[reply]

Would a typical bank safe deposit box form an adequate Faraday cage to protect electronic contents from an electromagnetic bomb? —Lowellian (reply) 00:02, 14 February 2008 (UTC)[reply]

Well I can't actually answer the question bear in mind your chance of getting access to your bank safe deposit box during the extreme chaos created by an electromagnetic bomb may be slim Nil Einne (talk) 06:54, 14 February 2008 (UTC)[reply]
Let's say it's electronics you want to get access to once the chaos calms down, so the question still stands, and I'd still like an answer if someone can give one. Also, your response did give me an idea for another question: would modern automobiles be able to start up after an EMP, or are they so dependent on electronics that they would be unable to do so? —Lowellian (reply) 12:50, 14 February 2008 (UTC)[reply]
A modern automobile would almost certainly toast after even a moderate EMP event. As it is, the police are learning to use devices that lay in the roadway and transfer an electrical transient to your car in order to stop you; this substitues (more safely) for the spike strips they now lay across the road to deflate all your tires.
A safe deposit box made of metal would probably provide pretty good protection against EMP. It all depends on how tightly the cover seals the box and how good and uniform is the electrical contact between the cover and the box; remember that a Faraday cage only works if the openings in it (including long, narrow slots!) are substantially shorter than the wavelength of the signal(s) you're trying to block.
Atlant (talk) 13:59, 14 February 2008 (UTC)[reply]
According to a recent documentary, you can make a car start after an EMP attack by replacing the solenoid. 206.252.74.48 (talk) 14:16, 14 February 2008 (UTC)[reply]
Very funny! Remember: after a nuclear attack, look for a car that predates any electronics fancier than an 8-track player.
Atlant (talk) 15:46, 14 February 2008 (UTC)[reply]

A safety deposit box generally is a thin metal box inside a thicker metal box inside a metal or reinforced concrete vault. This provides three levels of shielding against external electromagnetic fields, and should provide a respectable amount of shielding, but I have not seen figures on the strength of the hypothetical EMP so I could not attempt to calculate the field strength inside and compare it to the threshold for damage to electronics, magnetic media, etc. I'm surprised the folks who sell personal keychain radiation monitors [15] and iodine pills [16] do not advertise EMP safes for the home, perhaps welded from 1/2 inch ferrous metal. I expect that the avionics and communication equipment on military command planes and bombers are hardened to survive EMP, and they would not get off the ground if they had to be as solid as a safety deposit box installation. Edison (talk) 17:18, 15 February 2008 (UTC)[reply]

EMP-specific safes? Does that mean that ordinary metal home safes not secure against EMP? If not, then why not? —Lowellian (reply) 19:14, 15 February 2008 (UTC)[reply]
Many home safes seem to be plastic and concrete in large part, to deter theft and protect papers from fire. Older office safes are big steel boxes and it seems they would provide a great amount of shielding against external magnetic fields. I have seen nothing so far to convince me than a much thinner metal box would be inadequate. Edison (talk) 20:36, 15 February 2008 (UTC)[reply]

Another related question: websites I have seen about using aluminum foil or trash cans to construct Faraday cages usually recommend attaching a grounding wire to the makeshift cage. If, as suggested above, safe deposit boxes and metal safes without grounding wires may form adequate Faraday cages, why would the grounding wire be necessary on these makeshift cages? And does the grounding wire actually have to be stuck into the ground, as in the earth or soil? What if you can't leave the Faraday cage outside the house, then what do you do? (Besides sticking it in the soil of a houseplant vase? *wink* ) (Sorry for all these questions, but I don't know much about electrical physics.) —Lowellian (reply) 21:36, 15 February 2008 (UTC)[reply]

Whether a metal container is grounded or not should not affect its permeability to magnetic fields. Dissipating an electrical charge, such as from lightning is a different question. If current flowed through the container from a source to a ground connection, that might actually cause a magnetic field in the interior, if the current density was higher in one side of the container than in the other side. Aluminum foil would provide negligible magnetic shielding, but couod provide shielding against electric fields. I once used a coffee can as a Faraday cage to shield a large speaker magnet from causing distorted color in a TV picture tube. I have seen chicken wire used to provide a Faraday cage around a chamber where human subjects had electrophysiological recording of brain waves during perception experiments. No idea how either of these would fare against EMP. Edison (talk) 02:49, 17 February 2008 (UTC)[reply]

February 16

James Randy and Aliens

Anyone know if James Randy believes in Aliens? Malamockq (talk) 03:09, 16 February 2008 (UTC)[reply]

From his online bio] I get the idea that he does not believe that sentient technologically superior aliens have flown to earth in UFOs. I can't speculate on whether he thinks there is life, intelligent or not, elsewhere in the universe. Sifaka talk 03:50, 16 February 2008 (UTC)[reply]
You may want to see my somewhat OT comment here Talk:Unidentified flying object#What the hell is it going to take ... (please don't reply there tho) Nil Einne (talk) 17:26, 16 February 2008 (UTC)[reply]

EMBRYOLOGY

Do you know some tacts to remember the steps of growth that ocurre in human embryo growth during the first 8 weeks? thanks —Preceding unsigned comment added by 201.220.222.140 (talk) 04:41, 16 February 2008 (UTC)[reply]

There aren't many tactics beyond just memorization. Embryology, Human embryo would help. Also search pubmed.org for reviews on human embryology. Wisdom89 (T / C) 04:45, 16 February 2008 (UTC)[reply]
You can find some embryology mnemonics here. Personally I usually have more trouble remembering the mnemonics than I do the information they're supposed to help you remember. - Nunh-huh 04:47, 16 February 2008 (UTC)[reply]
I'm in Nunh-huh's camp and despise most mnemonics (although I hate "pneumonics" even more ;-). In approaching topics that appear to need rote memorization, I usually first try to understand terms, how structures got their names, etc. Some examples from embryology might be the origin of the term morula (a bunch of cells resembling a mulberry), why it's called the ultimobranchial body ("ultimo-", last; hence, "last of the branchial pouch derivatives"), etc. Works great for me, but your mileage may vary. --David Iberri (talk) 20:08, 16 February 2008 (UTC)[reply]

physics/water barometer

the height of water in a water barometer is measured from the open atmospheric pressure end vertically to the under side of the J tube.

the question is - If the lower u loop end of the J is elongated to about sixty feet, what differance will it make to the barometer?

M.Krishnapillai —Preceding unsigned comment added by 124.43.251.138 (talk) 06:59, 16 February 2008 (UTC)[reply]

I'm not certain if I understand your question, but I believe that extending the open side of a water barometer will make no difference. Perhaps our article on barometers may help. (EhJJ)TALK 22:32, 16 February 2008 (UTC)[reply]

Cowling nozzle?

Watching mythbusters today, when they tried to make a jet pack with 2 props and cowling, they didnt get enough lift. I would've thought that to get more lift, you could focus the air more by creating more of a nozzle effect, by narrowing the diamter after the prop instead of having it the same all the way down. Is there a technical reason why they couldnt do this? —Preceding unsigned comment added by 59.100.252.244 (talk) 12:53, 16 February 2008 (UTC)[reply]

How much elecricity does an solar panal of a sq mts produce.

Hello i am thinking of making a paper presentation on a way of producing power by concentrating mirrors and i want to know how they fare with solar panals .I would also like to know about there efficiency,cost etc.--Man manoj1990 (talk) 13:09, 16 February 2008 (UTC)[reply]

The article on Solar_radiation contains a number of estimates of how much solar energy arrives at the earth. The article on photovoltaics has a discussion of efficiency. JohnAspinall (talk) 04:54, 17 February 2008 (UTC)[reply]

Positive and negative load tests

G'day, RefDeskers. I'm stuck with something. I have come across the phrases positive load test and negative load test with reference to railway sleepers. I'm not seeking to learn the methodology of such tests, this is irrelevant - what interests me is the relevance of the terms positive and negative here. How do both of these tests differ and why the differentiation? If anyone has any ideas, I'd appreciate them. Thanks and cheers! --Ouro (blah blah) 13:42, 16 February 2008 (UTC)[reply]

Try this paper http://ro.uow.edu.au/cgi/viewcontent.cgi?article=1321&context=engpapers (look at figure 1) - can this be the answer. (the loads are 'moments' ie torque..)?77.86.8.83 (talk) 13:58, 16 February 2008 (UTC)[reply]
Makes me scratch my head, this diagram you pointed me to... could anyone briefly explain further? --Ouro (blah blah) 14:49, 16 February 2008 (UTC)[reply]
A railway sleeper somewhat works like a beam in a framed building structure. Such a beam is subjected to pressure (positive) in the top part and tension (negative) in the bottom part. In a typical reinforced beam the concrety bits cope with the pressure whilst the steel reinforcement takes up the tensile forces.
I assume that sleepers, being subjected to 2 point loads at the locus of the tracks, have similar characteristics of positively and negatively stressed parts, which have to be mathematically analysed to provide steel reinforcement in the zones of tensile forces. The article from the Wollongong University seems to indicate that significant tensile stress is measurable in the top part of the middle section of the sleeper.
There is a stack of suitable articles under the lemma structural analysis. Prost, --Cookatoo.ergo.ZooM (talk) 01:53, 17 February 2008 (UTC)[reply]

Deep space travel - Voyager and Pioneer

Are the Voyger and Pioneer space probes travelling particularly fast? That is, would it be hard, using exisiting, proven technology, to make a similar space probe travel ten times as fast as these probes? ike9898 (talk) 14:33, 16 February 2008 (UTC)[reply]

"Voyager 1 is departing the Solar System at a speed of 39,000 miles per hour. Voyager 2 is departing the Solar System at a speed of 35,000 miles per hour." http://www.spacetoday.org/SolSys/Voyagers20years.html

I have literally no idea about this stuff so couldn't say with any authority if faster speeds are possible but i suspect that the pace these things are travelling is not something that is controllable/could be changed in any meaningful way by modern technology design. ny156uk (talk) 14:38, 16 February 2008 (UTC)[reply]

Oh and Pioneer details on that page..." Accelerating to a speed of 82,000 mph, Pioneer 10 passed by Jupiter on December 3, 1973. so my assumption about speed is obviously wrong. (http://www.spacetoday.org/SolSys/ThePioneers.html) ny156uk (talk) 14:42, 16 February 2008 (UTC)[reply]

(ec) Well, let's see. Voyager I is travelling at 17,2 km/s relative to the Sun, Voyager II is supposed to be travelling 10% slower, so say about 15,5 km/s, and Pioneer 11 is travelling at 11,6 km/s, and Pioneer 10 is unfortunately unreachable. Now, say we increase tenfold the speed of Voyager II, giving ca. 150 km/s. As I read, one of the Helios probes reached around 70 km/s, a current record for a man-made object. Deep Impact reached velocities in the low twenties per second. So, we're not quite there yet actually, but ion thrusters promise certain advances. Also, keep in mind that probes tend to rely not only on engines for their speed, but also gravitational slingshots. Hope I helped a bit. --Ouro (blah blah) 14:48, 16 February 2008 (UTC)[reply]

can we reabsorb water from the bladder?

Once urine is in the bladder, can the body reabsorb it if it needs it? Say, if you were dumped into the desert with a full bladder would you be better off to 'hold it' so your body could use it later? 216.77.239.212 (talk) 15:05, 16 February 2008 (UTC)[reply]

The bladder is unable to reabsorb urine. If the bladder was able to reabsorb what would be the purpose of the kidneys, how could the kidneys control the hydration of the body is the urine was reabsorbed. One of the problems of an enlarged prostate gland is the inability to pass urine at will. This causes a build up of pressure in the bladder which if unresolved will result in back pressure on the kidneys. If you are in the desert with a full bladder it would be more sensible to pass the urine into a container and possibly seek a method to distil the water out for reconsumption. I'm assuming a certain desperation here. Richard Avery (talk) 16:23, 16 February 2008 (UTC)[reply]
Stillsuits! --Ouro (blah blah) 16:53, 16 February 2008 (UTC)[reply]
However, some people have survived without water by peeing and then drinking their own urine, yes urine is drinkable but at least it is your own urine. Hope this helps. Thanks. ~AH1(TCU) 17:27, 16 February 2008 (UTC)[reply]
Though there's only a limited amount of times that will work, of course, without an external water source. --98.217.18.109 (talk) 19:24, 16 February 2008 (UTC)[reply]
Maybe you should source this as from the little knowledge I have of extreme survival it is a bad idea to drink your own urine. On the impermeability of bladders, apparently there is a rise in cases of ruptured bladders in Europe that goes with the increase of the consumtion of beer and this especially for female drinkers as they have a smaller bladder. The scenario goes as follows: drink, to drunk to feel the need to pee, fall, rupture bladder. So if you drink beer, remember to pee! 200.127.59.151 (talk) 23:38, 16 February 2008 (UTC)[reply]

(unindent) thanks for this info, I wasn't really concerned about getting drunk and deserted in the desert. I did think though that the bladder membrane might be water permeable, which I think Richard answered, afaict. thanks Tim (talk) 00:23, 17 February 2008 (UTC)[reply]

A source is needed for the claim of females having smaller bladders. I have known females who could drink far more than males without needing to void urine. Edison (talk) 02:41, 17 February 2008 (UTC)[reply]

Reverse engineering in baking

I've found a type of bun, sold commercially, that is heavenly. It's cinnamon- and apple-flavoured, and I would like to know how I may proceed to make these on my own. They are awfully expensive - £4 for 8x bitesize. Can one reversely engineer these and find the recipe? Thank you immensely. 81.93.102.185 (talk) 16:53, 16 February 2008 (UTC)[reply]

Are maybe ingredients listed on the packaging? --Ouro (blah blah) 17:21, 16 February 2008 (UTC)[reply]
Beyond apple, sugar, apple-and-cinnamon fillings (detailed description) and the next 20 ingredients and E-substances, it doesn't lend much of a hint. I am wondering if actual tests can be done on the bun to show its amount of e.g. flour, eggs, everything. 81.93.102.185 (talk) 17:39, 16 February 2008 (UTC)[reply]
I guess one could try to estimate the ingredients from the results, but it'd be way more costly than buying them every day fresh. Oh, but if you have the ingredients and e-substances you're not that bad off. --Ouro (blah blah) 17:46, 16 February 2008 (UTC)[reply]
The proportion of the most basic ingredients (flour, eggs, water, yeast) is reasonably fixed, and a simple Google search for a recipe will yield satisfactory results (different will mostly affect the texture and consistency more than the flavour). A lot of those other ingredients, particularly the E numbers, are there because it's a commercial product - they're stabilisers and preservatives that are needed because it needs to have a factory-to-you time of a week or more. That's the big difference between stuff you bake at home and baked goods you buy from the supermarket - the home baked stuff takes incredible for the first 12 hours or so, but very quickly deteriorates. I think you'll get nice results just baking a basic bun with the normal basic ingredients. You might find that it doesn't taste appley-enough; if so, try making a reduction from apple juice and using that in place of some of the water the recipe calls for. -- Finlay McWalter | Talk 17:52, 16 February 2008 (UTC)[reply]
A recipe is more than an ingredients list. I'm no expert on baking, but I believe that the heavenliness (or lack thereof) of baked goods is based at least as much on technique as on raw chemical composition. No amount of reverse engineering, or mass spectrometry (or whatever) is going to tell you how finely or coarsely the shortening was cut in, or how long the dough was kneaded, or how long it sat between kneadings, or at what rate various amounts of liquids (some of which, of course, evaporated later) were added. For that, you're either going to have to suborn an employee at the bakery, or do lots of experimenting yourself. —Steve Summit (talk) 19:43, 16 February 2008 (UTC)[reply]
If the dough and sweet mix are separated in swirls, then the sweet mix will almost certainly contain apple or (even nicer) pear concentrate. Also, using apple varieties suited to cooking helps. I used to bake these. Polypipe Wrangler (talk) 21:22, 16 February 2008 (UTC)[reply]
You could always ask the bakery for the recipe: they might not consider it a trade secret. Of course, even if they give it to you, it might turn out to require equipment you don't have at home. [And then there's this hazard. :-)] --Anonymous, 22:19 UTC, February 16, 2008.
You might wish to seek out Saigon cinnamon, since it has greater aroma than other varieties in the baking. One source listed in the article is [The Spice House]. The cinnamon is the essence of the baked good, and compared to premium freshly ground spice, the spices sold in little jars or tins in the grocery store are pale imitations. [User:Edison|Edison]] (talk) 02:34, 17 February 2008 (UTC)[reply]

Why do some people turn green when they get sick?

I have searched the internet on this subject and not seen anything of use. Recently my daughter (white, slightly olive coloured skin) was ill and she definitely had a greenish cast to her skin. Our family are curious to know what causes the green colour. She was feeling dizzy so may have had blood drain from her face to the part of her body that needed help. Why is the resulting hue green? —Preceding unsigned comment added by 124.197.36.202 (talk) 20:29, 16 February 2008 (UTC)[reply]

We discussed green skin on the Miscellaneous desk last month (here and also here), but I guess we didn't consider the illness angle much. —Steve Summit (talk) 21:53, 16 February 2008 (UTC)[reply]

February 17

Carbonyl reduced to Hydroxyl

I understand that reduction is a gain of electrons and oxidation is a loss of electrons. But I don't understand how when a carbonyl is turned into a hydroxyl it was reduced. To me the number of electrons stayed the same in the functional group before and after. There were no gains or losses of electrons. Could someone explain this to me? 128.163.224.198 (talk) 00:29, 17 February 2008 (UTC)[reply]

It's a reference to the oxidation state of carbon, specificially. Whereas previously the oxygen-bound carbon (in the case of a ketone) was considered to have an oxidation state of +2, it now has an oxidation state of 0. Someguy1221 (talk) 00:34, 17 February 2008 (UTC)[reply]

Sleeping standing up

Are there humans who sleep standing up? I think that leaning on a corner it would be feasible at least.217.168.1.250 (talk) 00:58, 17 February 2008 (UTC)[reply]

It's not possible without an adequate means of support.The Updater would like to talk to you! 01:03, 17 February 2008 (UTC) —Preceding unsigned comment added by WikiHaquinator (talkcontribs) [reply]
If I can sleep sitting up, or sitting at my desk at work, I can probably sleep leaning against something. But unlike cattle, I do not believe humans can lock their knees and sleep while standing unsupported. Micronaps would occur if a person were sleep deprived long enough, but they would jerk as they started to fall. Edison (talk) 02:33, 17 February 2008 (UTC)[reply]

Electron microscope staining

hey,

Is it possible to stain cells for observation under electron microscope? The Updater would like to talk to you! 01:02, 17 February 2008 (UTC)[reply]

It is not necessary to stain a cell for veiwing with an electron microscope. The cell must be coated with a thin layer of metal (Usually gold I think) to reflect the electrons so an image can be created. It may also be helpful to see electron microscope. --Zrs 12 (talk) 01:27, 17 February 2008 (UTC)[reply]

Variable Speed of Light

I was recently reading a scientific paper written by one of my teacher's uncles on his theory about light having a varying instead of constant speed. First, he set some units: , by Newton's equation and defining as the acceleration . Also defining gravity is in which . He then says therefore, so . Then he says Length is L, mass is F, t is time, and so . He says work is mass multiplied by Length and energy is the rate of performing work so Newtonian energy is . He then says using the relativity equation we need to define mass. Since Newtonian mass is the relativity equation is as follows: . He says when simplified this equation shows time is Length. My questions are:

  • How did he get (this appears to be just an assumption to me)
  • Is there anything wrong with his logic? and,
  • When I simplified the equation I distinguished between Einstenian and Newtonian energy and reformulated the equation to and when it was reduced I got showing (it appears to me) that or at least for the fact that Einstenian and Newtonian energy are different. Who is correct? Does in fact?

Thanks, Zrs 12 (talk) 01:54, 17 February 2008 (UTC)[reply]

ATTENTION:I accidentally put (L/t2) now it's correct as (L/t)2. Sorry. Zrs 12 (talk) 03:31, 17 February 2008 (UTC)[reply]

The theory starts out with gibberish, because if , then in dimensional analysis length equals time squared, which is nonsense. Edison (talk) 02:30, 17 February 2008 (UTC)[reply]
Anything else? (Remember, he is trying to prove that the speed of light is veriable so maybe length does equal time squared?) Zrs 12 (talk) 02:56, 17 February 2008 (UTC)[reply]
Except that he didn't prove that length equals time squared, he just sort of declared it, and this seems to form the basis of the rest of the proof. Someguy1221 (talk) 03:19, 17 February 2008 (UTC)[reply]
Ok, thanks. What about the part where I showed ? Did I do this correctly or did he? In other words, did I reduce correctly? --Zrs 12 (talk) 03:24, 17 February 2008 (UTC)[reply]

Speculative Science Question

I was watching a Twilight Zone episode,A Kind of a Stopwatch, the other day. In it, a man gains the ability to "stop time." Objects freeze in midair, people don't move, and none of it affects him. My question is this: If such a thing were possible, then wouldn't he die? Would he be able to breathe the "stopped" air molecules? Would he be able to walk through said molecules? —Preceding unsigned comment added by 192.136.22.4 (talk) 02:04, 17 February 2008 (UTC)[reply]

I can't believe I'm answering this......it's a narrative fiction, you're supposed to just accept the premise as a given, and not examine it too closely, lest you ruin the story. Nevertheless, as I recall it, the protagonist is able to move objects, even though the objects don't move under their own force. So he ought to be able to breathe, eat, move, etc. - Nunh-huh 02:11, 17 February 2008 (UTC)[reply]
Yeah just accept sci-fi. In Star Wars sound can be heard even though there is no medium through which sound waves can travel. Zrs 12 (talk) 02:15, 17 February 2008 (UTC)[reply]

Use of Opiates in the treatment of psychiatric disorders in women!

This subject appears to still be taboo but are there any current medications with an opiate base which are being used for the treatment of psychiatric disorders in women. My wife is 53 years of age and is experiencing continued anxiety attacks, obsessive compulsions, suicidal thoughts, constant worrying and other disorders that leave her in a pretty constant state of depression. She currently is taking a number of medications subscribed to her by psychiatrists. Recently she had surgery on her wrist and was prescribed Hydrocodone for pain. By her own admission the Hydrocodone appears to be very effective in keeping her psychiatric disorders on an even keel. I know that until opiates were outlawed in the early part of the last century they were very commonly used for all sorts of disorders. She mentioned the effect the Hydrocodone has on her during her visit this week with her local Psychiatrist and he said that using Hydrocodone for this type of treatment was out of the question. I'm well aware of the addicitve nature of opiates but apparently some of the drugs she is currently taking are also strong addictive medications that you do not just stop taking. So the question is are there any particular opiate based drugs used for these purposes and is this type of treatment recognized by Psychiatrist's as being effective. Any help in this area would be greatly appreciated.66.169.81.246 (talk) 02:46, 17 February 2008 (UTC)[reply]